Mitä on Olemassa oleminen?

Heitänpähän tässä itseäni paremmille nojatuoli-filosofeille kysymyksen: Määrittele, mitä on olemassa oleminen? En keksinyt tähän oikein hyvää määritelmää ja löytämäni määritelmät tuntuivat jokseenkin hölmöiltä.

Minä alan tulla siihen tulokseen, ettei olemassa ololle voi tehdä yhtä määritelmää. Pitänee kysyä: Siis missä mielessä olemassa? Esimerkiksi ajatus tai idea voi olla olemassa, vaikka itse ajatuksen mielikuvan kohdetta ei olisikaan. Mutta miten määrittelet ”aidon” olemassa olemisen?

Fyysiset asiat ovat selvästikin olemassa ainakin materialistille, mutta idealisti voisi sanoa, että nekin ovat vain ideoita tai ajatuksia yhteisessä tietoisuudessa. Samoin, entä dualistin henki-olennot? Miten määritellä olemassa olo siten, ettei siihen sisälly keksityt ajatukset (kuten satuolennot), mutta esimerkiksi kummitukset sisältyisivät, jos kummituksia siis oikeasti olisi olemassa muutenkin kuin mielikuvituksissa?

114

2056

    Vastaukset

    Anonyymi (Kirjaudu / Rekisteröidy)
    5000
    • Anonyymi

      Tämä hetki on olemassa ja minä olen olemassa. Se on ainakin varmaa. Ja jotakin näyttäisi olevan mun ulkopuolella tai musta erillistä.

      • Se hetki oli olemassa, mutta onko enää?


    • Anonyymi

      "Miten määritellä olemassa olo siten, ettei siihen sisälly keksityt ajatukset (kuten satuolennot), mutta esimerkiksi kummitukset sisältyisivät, jos kummituksia siis oikeasti olisi olemassa muutenkin kuin mielikuvituksissa?"

      Tuontyyppistä määrittelyähän se tiede on yrittänyt vaihtelevalla menestyksellä saada aikaiseksi jo varmaan ainakin muutama tuhat vuotta mutta aina tuntuu tupsahtavat jotain uutta esille mikä osoittaa olemassaolevan tarkahkon määritelmän joko vääräksi tai puutteelliseksi.

      Olemassaolon määritelmän tivaaminen on vähän sama asia kuin jos tivaisi todellisuuden tarkkaa määritelmää joka tuntuu melkein jokaisella olevan vähän erilainen ja jatkuvasti työn alla muutenkin.

      Minusta olemassaolo (tai todellisuus) ei ole mikään staattinen asia (tuolla jossakin) jonka ylipäätänsä voisi joskus ymmärtää täydellisesti vaan alati muuttuva asia johon myös me kaikki yhdessä koko ajan vaikutamme eli siinä mielessä tiede ei tule koskaan valmiiksi eikä ymmärrys täydelliseksi.

      ps. lisäaineiden käyttö näköjään tekee sinulle hyvää. Itselläni tuntuu ajattelu kuplivan parhaiten kun en koske edes ykkösolueen pariin viikkoon kun meikäläisellä kaikki tuollaiset lisäaineet lähinnä tuntuvat hidastavan ajatuksen juoksua (mikä sekin voi olla ihan hyvä asia ainakin joskus)

      B

      • Anonyymi

        Miksi pyrit olemaan rehellinen.

        Kerro heti!


    • Anonyymi

      Vastaus: olemassa olemista ei voida määritellä esittämälläsi tavalla. Se ei ole määriteltävissä sanotussa mielessä. Ts. koko kysymys on väärin asetettu.

      Tässä viittaan Martin Heideggerin filosofiaan, joka tunnetaan myös eksistentialismina. Olemassa oleminen on aina jollekin olemista, eli esillä olevilla objekteilla on oltava 'täälläolo', joka ei ole samanlaista olemista kuin ne ovat. Kaikki esillä oleva on suhteessa täälläoloon ja sen olemisen sanotaan olevan eksistentiaalista.

      Voisi sanoa, että ainoastaan eksistentiaalinen oleminen on varsinaisesti olemassa olemista "aidossa" mielessä. Toki ne objektitkin ovat välttämättä olemassa, jos eksistenssikin on olemassa, koska eksistentiaalinen oleminen on jossakin-olemista, eli toisin sanoen täälläolo sijaitsee maailmassa (=objektien keskuudessa).

      Mutta siis itse olemassa olemista ei voida määritellä, joten kysymykseesi ei voida vastata.

      • Anonyymi

        "Olemassa oleminen on aina jollekin olemista, eli esillä olevilla objekteilla on oltava 'täälläolo', joka ei ole samanlaista olemista kuin ne ovat. Kaikki esillä oleva on suhteessa täälläoloon ja sen olemisen sanotaan olevan eksistentiaalista."

        Elämme havaitsijakeskeisessä kaikkeudessa jossa on jatkuva ja vahva takaisinkytkentä ns. fysikaalisen todellisuuden ja havaitsijoiden välillä. Ts. havaitsijoiden joukko muodostaa omanlaisensa todellisuuskuplan joka resonoi heidän joukkointentionsa kanssa.

        Tällainen havaitsijakeskeinen kuvaus on minusta hyvin pitkälle varsin yhteensopiva eksistentialismin (Heidegger) ja Husserlin filosofian kanssa.

        https://fi.wikipedia.org/wiki/Intentionaalisuus

        Kvanttifysiikassa esim. John Wheelerin Participatory universe (osallistuva kaikkeus)

        https://futurism.com/john-wheelers-participatory-universe

        Hieman spekulatiivisesti voisi ehkä olettaa että erilaiset havaitsijajoukot eri aikoina ja eri paikoissa voivat tuottaa erilaisia "todellisuuskuplia" joissa tietyt asiat voivat olla huomattavasti todennäköisempiä kuin jonkun toisen ryhmän tuottamassa "todellisuuskuplassa"

        B


      • Anonyymi
        Anonyymi kirjoitti:

        "Olemassa oleminen on aina jollekin olemista, eli esillä olevilla objekteilla on oltava 'täälläolo', joka ei ole samanlaista olemista kuin ne ovat. Kaikki esillä oleva on suhteessa täälläoloon ja sen olemisen sanotaan olevan eksistentiaalista."

        Elämme havaitsijakeskeisessä kaikkeudessa jossa on jatkuva ja vahva takaisinkytkentä ns. fysikaalisen todellisuuden ja havaitsijoiden välillä. Ts. havaitsijoiden joukko muodostaa omanlaisensa todellisuuskuplan joka resonoi heidän joukkointentionsa kanssa.

        Tällainen havaitsijakeskeinen kuvaus on minusta hyvin pitkälle varsin yhteensopiva eksistentialismin (Heidegger) ja Husserlin filosofian kanssa.

        https://fi.wikipedia.org/wiki/Intentionaalisuus

        Kvanttifysiikassa esim. John Wheelerin Participatory universe (osallistuva kaikkeus)

        https://futurism.com/john-wheelers-participatory-universe

        Hieman spekulatiivisesti voisi ehkä olettaa että erilaiset havaitsijajoukot eri aikoina ja eri paikoissa voivat tuottaa erilaisia "todellisuuskuplia" joissa tietyt asiat voivat olla huomattavasti todennäköisempiä kuin jonkun toisen ryhmän tuottamassa "todellisuuskuplassa"

        B

        Heideggerin ajattelu on yhteensovitettavissa, jos halutaan, Wheelerin ajatuksiin universumista. Näin ei kuitenkaan ole Husserlin fenomenologian osalta, koska Heidegger ei jaa Husserlin idealistista lähestymistapaa todellisuuteen. Heidän keskinäistä erimielisyyttä voi kuvata niin suureksi, että Heideggeria usein ymmärretään väärin, jos olettaa hänen olevan samaa mieltä Husserlin kanssa ja siltä pohjalta lähestyy Heideggerin ajattelua.


      • Anonyymi
        Anonyymi kirjoitti:

        "Olemassa oleminen on aina jollekin olemista, eli esillä olevilla objekteilla on oltava 'täälläolo', joka ei ole samanlaista olemista kuin ne ovat. Kaikki esillä oleva on suhteessa täälläoloon ja sen olemisen sanotaan olevan eksistentiaalista."

        Elämme havaitsijakeskeisessä kaikkeudessa jossa on jatkuva ja vahva takaisinkytkentä ns. fysikaalisen todellisuuden ja havaitsijoiden välillä. Ts. havaitsijoiden joukko muodostaa omanlaisensa todellisuuskuplan joka resonoi heidän joukkointentionsa kanssa.

        Tällainen havaitsijakeskeinen kuvaus on minusta hyvin pitkälle varsin yhteensopiva eksistentialismin (Heidegger) ja Husserlin filosofian kanssa.

        https://fi.wikipedia.org/wiki/Intentionaalisuus

        Kvanttifysiikassa esim. John Wheelerin Participatory universe (osallistuva kaikkeus)

        https://futurism.com/john-wheelers-participatory-universe

        Hieman spekulatiivisesti voisi ehkä olettaa että erilaiset havaitsijajoukot eri aikoina ja eri paikoissa voivat tuottaa erilaisia "todellisuuskuplia" joissa tietyt asiat voivat olla huomattavasti todennäköisempiä kuin jonkun toisen ryhmän tuottamassa "todellisuuskuplassa"

        B

        "sen olemisen sanotaan olevan eksistentiaalista"

        Eikö existence tarkoita olemassaoloa? Yhtä hyvin voisi sanoa että eksistenssi on eksistentiaalista, tai olemassaolo on olemassaoloa. Tuollainen määritelmä on yhtä tyhjän kanssa, esim. vihreyttäkään ei voi määritellä sanomalla vihreä on vihreää, vaan sen voi määritellä valon aallonpituuksilla, jolloin vältytään määrittelyssä käyttämästä vihreän käsitettä, joka oli tarkoitus määritellä.
        Myös olemisen käsite tulisi määritellä siten, ettei määritelmässä käytetä käsitteitä olemassaolo, eksistenssi, oleminen, tms. Kyseiset käsitteet oli tarkoitus vasta määritellä, eikä niitä voi sitä ennen käyttää minkään asian määrittelemiseen.


      • Anonyymi
        Anonyymi kirjoitti:

        "sen olemisen sanotaan olevan eksistentiaalista"

        Eikö existence tarkoita olemassaoloa? Yhtä hyvin voisi sanoa että eksistenssi on eksistentiaalista, tai olemassaolo on olemassaoloa. Tuollainen määritelmä on yhtä tyhjän kanssa, esim. vihreyttäkään ei voi määritellä sanomalla vihreä on vihreää, vaan sen voi määritellä valon aallonpituuksilla, jolloin vältytään määrittelyssä käyttämästä vihreän käsitettä, joka oli tarkoitus määritellä.
        Myös olemisen käsite tulisi määritellä siten, ettei määritelmässä käytetä käsitteitä olemassaolo, eksistenssi, oleminen, tms. Kyseiset käsitteet oli tarkoitus vasta määritellä, eikä niitä voi sitä ennen käyttää minkään asian määrittelemiseen.

        Kommentoin vielä tähän, koska selvästi väärinymmärrys on suurta. Eksistentialismi ei määrittele olemassoloa. Päinvastoin olemassolo ei ole määriteltävissä. Ts. koko kysymys on väärin asetettu.

        Lainaan tuolta alempaa tästä ketjusta Belisariota: "Tulkinnat ja metakognitiot (teoriat yms) taas kuuluvat ei-olemassaolevaan maailmaan eli voivat vaikuttaa epätasapainossa olevan itseorganisoituvan systeemin (implisiittisen järjestyksen aktualisoija) käyttäytymiseen eli ovat todellisia mutta ei olemassaolevia edellä olevan määritelmän mukaan."

        Tällainen kuvailu on eksistentialistiselta kannalta puutaheinää. *Olemassaolosta' ei voida puhua olemassolon käsitteellä tai käyttäen olemassaoloa predikaattina tai millään muullakaan tavalla, joka viittaa mihinkään olemassaolemiseen. Tämä on eksistentialismin mukaan täydellisesti mahdotonta.

        Sen vuoksi 'metakognitiot' eivät kuulu sen kummemmin olemassaolevaan kuin ei-olemassaolevaan, mitä nillä määrittelyillä Belisario sitten tarkoitaakaan tässä yhteydessä.

        Haideggerin keskeinen käsite on 'täälläolo' (dasein), jonka suhteen kaikki, mitä voitaisiin luonnehtia olemassaolevaksi, tulee mahdolliseksi. Täälläolo on kaiken (eli kaiken sellaisen, mitä ylipäätään voitaisiin kuvitella olevan) fundamentaalinen edellytys.


      • Anonyymi kirjoitti:

        "sen olemisen sanotaan olevan eksistentiaalista"

        Eikö existence tarkoita olemassaoloa? Yhtä hyvin voisi sanoa että eksistenssi on eksistentiaalista, tai olemassaolo on olemassaoloa. Tuollainen määritelmä on yhtä tyhjän kanssa, esim. vihreyttäkään ei voi määritellä sanomalla vihreä on vihreää, vaan sen voi määritellä valon aallonpituuksilla, jolloin vältytään määrittelyssä käyttämästä vihreän käsitettä, joka oli tarkoitus määritellä.
        Myös olemisen käsite tulisi määritellä siten, ettei määritelmässä käytetä käsitteitä olemassaolo, eksistenssi, oleminen, tms. Kyseiset käsitteet oli tarkoitus vasta määritellä, eikä niitä voi sitä ennen käyttää minkään asian määrittelemiseen.

        ”esim. vihreyttäkään ei voi määritellä sanomalla vihreä on vihreää, vaan sen voi määritellä valon aallonpituuksilla, jolloin vältytään määrittelyssä käyttämästä vihreän käsitettä, joka oli tarkoitus määritellä.”

        Kokemuksellisten asioiden määrittely on vaikeaa. Tutkimalla todellisuutta olemme saaneet aikaan teorian, joka selittää vihreän (kvalian) liittyvän silmiin tulevaan tiettyyn valon aallonpituuteen, jonka silmien tappisolut välittävät hermostolle. Mutta tämä ei selitä vihreää, jos puhumme kvaliasta eli vihreän värin kokemuksesta. Voisiko joku meistä nähdä vihreän sinisenä ja sinisen vihreänä?

        Olemassa olon ongelma liittyy myös maailmankuvaan. Materialistisestikin ajateltuna voisimme olla aivot-purkissa ilman fyysistä todellisuutta. Maailma olisi tällöin meille täysin samanlainen kuin nyt, mutta materialistisesti se olisi illuusiota.

        Kenties joku filosofiaa enemmän lukenut voisi valaista minulle, miten olisivatko maailmassa olevat esineet olemassa, vaikka ne olisivat pelkkää simulaatiota? Kysymys saattaa olla turha - eli riippuu siitä, mitä tarkoitamme olemassa olemisella...


      • Anonyymi
        Anonyymi kirjoitti:

        Kommentoin vielä tähän, koska selvästi väärinymmärrys on suurta. Eksistentialismi ei määrittele olemassoloa. Päinvastoin olemassolo ei ole määriteltävissä. Ts. koko kysymys on väärin asetettu.

        Lainaan tuolta alempaa tästä ketjusta Belisariota: "Tulkinnat ja metakognitiot (teoriat yms) taas kuuluvat ei-olemassaolevaan maailmaan eli voivat vaikuttaa epätasapainossa olevan itseorganisoituvan systeemin (implisiittisen järjestyksen aktualisoija) käyttäytymiseen eli ovat todellisia mutta ei olemassaolevia edellä olevan määritelmän mukaan."

        Tällainen kuvailu on eksistentialistiselta kannalta puutaheinää. *Olemassaolosta' ei voida puhua olemassolon käsitteellä tai käyttäen olemassaoloa predikaattina tai millään muullakaan tavalla, joka viittaa mihinkään olemassaolemiseen. Tämä on eksistentialismin mukaan täydellisesti mahdotonta.

        Sen vuoksi 'metakognitiot' eivät kuulu sen kummemmin olemassaolevaan kuin ei-olemassaolevaan, mitä nillä määrittelyillä Belisario sitten tarkoitaakaan tässä yhteydessä.

        Haideggerin keskeinen käsite on 'täälläolo' (dasein), jonka suhteen kaikki, mitä voitaisiin luonnehtia olemassaolevaksi, tulee mahdolliseksi. Täälläolo on kaiken (eli kaiken sellaisen, mitä ylipäätään voitaisiin kuvitella olevan) fundamentaalinen edellytys.

        "Tällainen kuvailu on eksistentialistiselta kannalta puutaheinää. *Olemassaolosta' ei voida puhua olemassolon käsitteellä tai käyttäen olemassaoloa predikaattina tai millään muullakaan tavalla, joka viittaa mihinkään olemassaolemiseen. Tämä on eksistentialismin mukaan täydellisesti mahdotonta."

        Ko. olemassaolon määrittely oli lähinnä tarkoitettu vastaukseksi ProffessoriRealistille enkä edes yrittänyt ottaa siinä myös huomioon Heideggerin eksistentialismia joka olisi taas vienyt pohdinnan sellaiseen luuppiin josta tunnu olevan ulospääsyä.

        Eksistentialistisen filosofian (Heidegger & Gadamer)

        "hermeneuttinen kehä nähdään prosessina, jossa merkitysten tulkinta annetaan aina toisilla merkityksillä, jotka tulkitaan toisilla merkityksillä ja niin edelleen. Merkitysten kokonaisuudella ei ole kivijalkaa, joka antaisi sille tukevan perustan reaalitodellisuudessa."

        https://tieteentermipankki.fi/wiki/Filosofia:hermeneuttinen_kehä

        Itselleni Heideggerin eksistentialismi ei ole koskaan oikein "resonoinut" vaikka esim. Hussserlin fenomenologia on minusta varsin kiinnostavaa.

        "Sen vuoksi 'metakognitiot' eivät kuulu sen kummemmin olemassaolevaan kuin ei-olemassaolevaan, mitä nillä määrittelyillä Belisario sitten tarkoitaakaan tässä yhteydessä."

        Yritin löytää sellaisen (fysikaalisen) olemassaolon määritelmän mikä olisi mahdollisimman yksiselitteinen (ja deterministinen) ja jossa erotetaan selkeästi aktuaaliset ja potentiaaliset asiat toisistaan eli lähdetään siitä että vain aktuaalinen on olemassaolemista yksiselitteisessä muodossa ja suhteessa aikaan kaikki menneisyys on aktuaalista eli todellista ja kaikki tuleva aika (esim. aaltofunktio) taas potentiaalista eli ei vielä aktuaalista eikä olemassaolevaa.

        Belisario


      • Anonyymi kirjoitti:

        "Tällainen kuvailu on eksistentialistiselta kannalta puutaheinää. *Olemassaolosta' ei voida puhua olemassolon käsitteellä tai käyttäen olemassaoloa predikaattina tai millään muullakaan tavalla, joka viittaa mihinkään olemassaolemiseen. Tämä on eksistentialismin mukaan täydellisesti mahdotonta."

        Ko. olemassaolon määrittely oli lähinnä tarkoitettu vastaukseksi ProffessoriRealistille enkä edes yrittänyt ottaa siinä myös huomioon Heideggerin eksistentialismia joka olisi taas vienyt pohdinnan sellaiseen luuppiin josta tunnu olevan ulospääsyä.

        Eksistentialistisen filosofian (Heidegger & Gadamer)

        "hermeneuttinen kehä nähdään prosessina, jossa merkitysten tulkinta annetaan aina toisilla merkityksillä, jotka tulkitaan toisilla merkityksillä ja niin edelleen. Merkitysten kokonaisuudella ei ole kivijalkaa, joka antaisi sille tukevan perustan reaalitodellisuudessa."

        https://tieteentermipankki.fi/wiki/Filosofia:hermeneuttinen_kehä

        Itselleni Heideggerin eksistentialismi ei ole koskaan oikein "resonoinut" vaikka esim. Hussserlin fenomenologia on minusta varsin kiinnostavaa.

        "Sen vuoksi 'metakognitiot' eivät kuulu sen kummemmin olemassaolevaan kuin ei-olemassaolevaan, mitä nillä määrittelyillä Belisario sitten tarkoitaakaan tässä yhteydessä."

        Yritin löytää sellaisen (fysikaalisen) olemassaolon määritelmän mikä olisi mahdollisimman yksiselitteinen (ja deterministinen) ja jossa erotetaan selkeästi aktuaaliset ja potentiaaliset asiat toisistaan eli lähdetään siitä että vain aktuaalinen on olemassaolemista yksiselitteisessä muodossa ja suhteessa aikaan kaikki menneisyys on aktuaalista eli todellista ja kaikki tuleva aika (esim. aaltofunktio) taas potentiaalista eli ei vielä aktuaalista eikä olemassaolevaa.

        Belisario

        Nähdäkseni koko potentiaalinen ja aktuaarinen olemassa-olon ajatuksen mielekkyys romuttui Einsteinin suhteellisuusteorian myötä. Ongelma on, että jokaiselle tarkkailijalle on oma valokartionsa, eikä mitään tarkkailijasta riippumatonta nyt-hetkeä voida määritellä.


      • Anonyymi
        Järkisyitä kirjoitti:

        Nähdäkseni koko potentiaalinen ja aktuaarinen olemassa-olon ajatuksen mielekkyys romuttui Einsteinin suhteellisuusteorian myötä. Ongelma on, että jokaiselle tarkkailijalle on oma valokartionsa, eikä mitään tarkkailijasta riippumatonta nyt-hetkeä voida määritellä.

        "Nähdäkseni koko potentiaalinen ja aktuaalinen olemassa-olon ajatuksen mielekkyys romuttui Einsteinin suhteellisuusteorian myötä. Ongelma on, että jokaiselle tarkkailijalle on oma valokartionsa, eikä mitään tarkkailijasta riippumatonta nyt-hetkeä voida määritellä."

        Kvanttifysiikassa taas Schrödingerin aaltoyhtälö nimenomaan kuvaa tulevaisuuden potentiaalisuutta (ellei sitä erityisesti haluta kieltää postuloimalla jokaisen kvanttifysikaalisen vuorovaikutuksen seurauksena uusia versumeita)

        Nyt-hetkellä ei tässä ole merkitystä koska kaikki mitattu ja havaittu on aina käytännössä menneisyyttä eli fysikaaliselta kannalta tarkoin määriteltävissä olevaa nyt-hetkeä ei edes ole olemassa koska sen kesto=0.

        Eli ts. professorirealisti on mielestäni taas osittain oikeassa eli menneisyys on nimenomaan (super-)deterministinen kun taas se kaikki epämääräinen potentiaalisuus liittyy aina tulevaisuuteen ellei sitten haluta jostain syystä olettaa että kaikki mahdollinen maailmankaikkeuden historiassa on etukäteen deterministisesti määritelty jo siinä oletetussa alkuräjähdyksessä jossa teorian mukaan syntyi aika-avaruus että ns. ikuiset luonnonlait. Se pro-realistin inhoama vapaa-tahto tai vapaa valinta taas liittyy aina tuleviin tapahtumiin.

        Mitä tulee suhtiksiin niin löysin tänään Cornellin yliopiston julkaisun "On the Existence of Undistorted Progressive Waves (UPWs) of Arbitrary Speeds 0≤v<∞ in Nature" (1997) joka saattaa ainakin osittain vetää maton alta suhtiksilta muun vastaavan aineiston ohessa jota olen tarkoituksella keräillyt jo vuosia.

        https://arxiv.org/abs/hep-th/9606171

        Mikäs sen hauskempaa kuin uhrata n. 40 vuotta elämästään virallisen tieteen uskomuksien vastaisten anomalioiden ja teorioiden etsimiseen ja tutkimiseen:D

        Belisario


      • Anonyymi kirjoitti:

        "Nähdäkseni koko potentiaalinen ja aktuaalinen olemassa-olon ajatuksen mielekkyys romuttui Einsteinin suhteellisuusteorian myötä. Ongelma on, että jokaiselle tarkkailijalle on oma valokartionsa, eikä mitään tarkkailijasta riippumatonta nyt-hetkeä voida määritellä."

        Kvanttifysiikassa taas Schrödingerin aaltoyhtälö nimenomaan kuvaa tulevaisuuden potentiaalisuutta (ellei sitä erityisesti haluta kieltää postuloimalla jokaisen kvanttifysikaalisen vuorovaikutuksen seurauksena uusia versumeita)

        Nyt-hetkellä ei tässä ole merkitystä koska kaikki mitattu ja havaittu on aina käytännössä menneisyyttä eli fysikaaliselta kannalta tarkoin määriteltävissä olevaa nyt-hetkeä ei edes ole olemassa koska sen kesto=0.

        Eli ts. professorirealisti on mielestäni taas osittain oikeassa eli menneisyys on nimenomaan (super-)deterministinen kun taas se kaikki epämääräinen potentiaalisuus liittyy aina tulevaisuuteen ellei sitten haluta jostain syystä olettaa että kaikki mahdollinen maailmankaikkeuden historiassa on etukäteen deterministisesti määritelty jo siinä oletetussa alkuräjähdyksessä jossa teorian mukaan syntyi aika-avaruus että ns. ikuiset luonnonlait. Se pro-realistin inhoama vapaa-tahto tai vapaa valinta taas liittyy aina tuleviin tapahtumiin.

        Mitä tulee suhtiksiin niin löysin tänään Cornellin yliopiston julkaisun "On the Existence of Undistorted Progressive Waves (UPWs) of Arbitrary Speeds 0≤v<∞ in Nature" (1997) joka saattaa ainakin osittain vetää maton alta suhtiksilta muun vastaavan aineiston ohessa jota olen tarkoituksella keräillyt jo vuosia.

        https://arxiv.org/abs/hep-th/9606171

        Mikäs sen hauskempaa kuin uhrata n. 40 vuotta elämästään virallisen tieteen uskomuksien vastaisten anomalioiden ja teorioiden etsimiseen ja tutkimiseen:D

        Belisario

        ”Kvanttifysiikassa taas Schrödingerin aaltoyhtälö nimenomaan kuvaa tulevaisuuden potentiaalisuutta”

        Oikeastaan ei vaan se kuvaa nyt-hetkeä. Tarkoittanet, että se kuvaa hiukkasta potentiaali-aaltona, jolloin hiukkaset todellinen sijainti konkretisoituu vasta, kun hiukkanen vuoro-vaikuttaa jonkun toisen kanssa.

        Mutta tällä sinänsä ei ole mitään tekemistä suhteellisuusteorian kanssa. Sekä suhteellisuusteoria että kvanttimekaniikka ovat omia teorioitaan, jotka toimivat omissa ympäristöissään: Kvanttifysiikka mikromaailmassa ja suhteellisuusteoria makromaailmassa suurilla nopeuksilla.


      • Anonyymi
        Anonyymi kirjoitti:

        "Tällainen kuvailu on eksistentialistiselta kannalta puutaheinää. *Olemassaolosta' ei voida puhua olemassolon käsitteellä tai käyttäen olemassaoloa predikaattina tai millään muullakaan tavalla, joka viittaa mihinkään olemassaolemiseen. Tämä on eksistentialismin mukaan täydellisesti mahdotonta."

        Ko. olemassaolon määrittely oli lähinnä tarkoitettu vastaukseksi ProffessoriRealistille enkä edes yrittänyt ottaa siinä myös huomioon Heideggerin eksistentialismia joka olisi taas vienyt pohdinnan sellaiseen luuppiin josta tunnu olevan ulospääsyä.

        Eksistentialistisen filosofian (Heidegger & Gadamer)

        "hermeneuttinen kehä nähdään prosessina, jossa merkitysten tulkinta annetaan aina toisilla merkityksillä, jotka tulkitaan toisilla merkityksillä ja niin edelleen. Merkitysten kokonaisuudella ei ole kivijalkaa, joka antaisi sille tukevan perustan reaalitodellisuudessa."

        https://tieteentermipankki.fi/wiki/Filosofia:hermeneuttinen_kehä

        Itselleni Heideggerin eksistentialismi ei ole koskaan oikein "resonoinut" vaikka esim. Hussserlin fenomenologia on minusta varsin kiinnostavaa.

        "Sen vuoksi 'metakognitiot' eivät kuulu sen kummemmin olemassaolevaan kuin ei-olemassaolevaan, mitä nillä määrittelyillä Belisario sitten tarkoitaakaan tässä yhteydessä."

        Yritin löytää sellaisen (fysikaalisen) olemassaolon määritelmän mikä olisi mahdollisimman yksiselitteinen (ja deterministinen) ja jossa erotetaan selkeästi aktuaaliset ja potentiaaliset asiat toisistaan eli lähdetään siitä että vain aktuaalinen on olemassaolemista yksiselitteisessä muodossa ja suhteessa aikaan kaikki menneisyys on aktuaalista eli todellista ja kaikki tuleva aika (esim. aaltofunktio) taas potentiaalista eli ei vielä aktuaalista eikä olemassaolevaa.

        Belisario

        Belisariolle

        "Ko. olemassaolon määrittely oli lähinnä tarkoitettu vastaukseksi ProffessoriRealistille enkä edes yrittänyt ottaa siinä myös huomioon Heideggerin eksistentialismia joka olisi taas vienyt pohdinnan sellaiseen luuppiin josta tunnu olevan ulospääsyä."

        Ymmärsin tämän hyvin, mutta kommenttini oli tarkoitettu vain erään väärinymmärryksen oikaisuun, eikä siten siis ollenkaan kritiikiksi Belisariota kohtaan. Heideggerin lähestymistapaa todellisuuteen taasen voitaisiin kuvailla "ei-materialistiseksi" kuitenkin ottaen huomioon sen, ettei hän kuitenkaan ole idealistikaan siinä mielessä mitä Husserlin filosofiassa todellisuutta tulkitaan.

        Mutta oma näkemykseni on asiaan on sikäli monitulkintainen, että pidän empiirisen tieteen tuloksia todellisuudesta kiinnostavina, vaikka tiedostan tieteellisen menetelmän rajoittuneisuuden siinä mielessä, jos tarkastellaan sen tuloksia heideggerilaisen fenomenologian ankarassa tarkastelukulmassa.

        "hermeneuttinen kehä nähdään prosessina, jossa merkitysten tulkinta annetaan aina toisilla merkityksillä, jotka tulkitaan toisilla merkityksillä ja niin edelleen. Merkitysten kokonaisuudella ei ole kivijalkaa, joka antaisi sille tukevan perustan reaalitodellisuudessa."

        Mitä tulee mainittuun hermeneuttiseen kehään niin pidän selviönä, että merkitysten tasolla reaalitodellisuudella ei ole eikä edes voi olla mitään kivijalkaa, josta tietoinen ihminen (=minuus tai heideggerilaisittain 'täälläolo') voisi mitenkään saada perustaa todellisuuskäsitykselleen. Tässä lähestymistavassa joudutaan tyytymään tiedon epävarmuuteen lopullisessa mielessä ja tarkastelemaan tieteenkin tuloksia vain eräänlaisina hypoteeseinä. Mielestäni tällainen lähestymistapa ei ole kuitenkaan johda Belisarion esitämään "luuppiin, josta ei ole ulospääsyä".

        Keskusteltaessa tulisi aina ottaa huomioon, missä lähestymiskulmassa asiaa olla tarkastelemassa, eli siis toisin sanoen pitää mielessä tieteellisen tiedon rajallisuus antamatta tämän asian kuitenkaan estää keskustelua (luonnon)tieteellisen menetelmän kiinnostavista tuloksista. Vielä toisella tavalla sanoen: empiirisellä tieteellä on ansionsa, joita ei ole tarpeen kiistää, kunhan ottaa huomioon, ettei niiden tulokset ole epistemologisessa tarkastelussa välttämättä totuutta tavoittavia.


      • Anonyymi

        Olemassa oleminen
        = hengittäminen
        Laatua hengittämiseen tulee elämisellä!


    • Anonyymi

      Mitä olemassaolo tarkoittaa? Mikä on sen merkitys? Ja jos sillä ei ole olemassa täsmällistä merkitystä, kuten luulen, mutta korostan, en tiedä, se on olematon käsite, jolla itsessään ei ole yhtään olemassa oloa, koska se ei merkitse mitään. Ja jos se ei kerran merkitse mitään, kuten näin luullessani väitän, jota en kyllä epäile, mitä tällöin tarkoitetaan, kun puhutaan mielettömiä?

      Descarteskin puhui mitä sattui. Oli kuulemma olemassa siitäkin huolimatta, ettei osannut ajatella. Vai liittyikö se olemassaolo sittenkään ajatteluun, kuten Descartes ajatellessaan mitä sattui väitti epäilemättä, vaikka toisaalta korosti epäilyn tärkeyttä väitteissään.

      Onko olemassaolo sama asia kuin oleminen? Voiko kaksi eri sanaa tarkoittaa samaa asiaa ja mitä tarkoitus merkitsee?

    • Aito olemassaolo on kaikki mitä me voimme kokea viidellä aisteilla tai mitata tieteellisillä mittareilla. Kummituksia ei ole voitu havaita ja mitata tarpeeksi varmasti, joten me ei tiedetä onko niitä olemassa.

    • Anonyymi

      'Olemassa oleminen' on käsite, josta voi luoda monimutkaisia pohdiskelu- ja väittelytilanteita. Olemassa oleminen ei sinänsä ole kummoinenkaan asia, jos se perustuu primitiiviseen havainnointiin ja kokemukseen. Kun tietoisuuteen muodostuu, tai rakennetaan opiskelemalla, laaja informaatioverkosto, erilaiset toisiinsa sekä negatiivisesti että positiivisesti vaikuttavat sisällöt ja merkitykset voivat yhdessä muodostaa tilanteita, jossa koetaan, ettei olemassa olevilla tiedoilla voida luoda tyydyttävää vastausta. Seurauksena on, että etsitään uutta tietoa, jotta epätyydyttävät ja ristiriitaiset tilanteet ja asiayhteydet voidaan selvittää ja eliminoida. 'Olemassa oleminen' on olemassaolemista, mutta informaatiolla siitä luodaan aihe, jossa olemassaolemisen ymmärtäminen monimutkaistuu ja hämärtyy. Kysehän on aina lopulta siitä, miten mikäkin asia koetaan, ja tähän vaikuttaa voimakkaastikin se, miten mikäkin asia on käsitteellistetty, määritelty, informoitu.

      Mitä ns. älykkäämmäksi ihminen tulee, sitä voimakkaammaksi voi muodostua myös epätyydyttävä kokemus, koska koetaan, että nykyiset tiedot ei riitä? Kun ihminen luo lisää informaatiota, tyytymättömyys lisääntyy? Koska aina on jotain, minkä ihminen kokee tuntemattomaksi?

    • Anonyymi

      Jos ihminen saisi tietää todellisen syyn ja merkityksen olemassaololleen, niin hän alkaisi ehkä kapinoimaan sitä vastaan. Suurin osa tällä palstalla ei varmaankaan mihinkään "Suureen Suunnitelmaan" usko, vaan pitävät kaikkea ilman objektiivista arvoa/merkitystä. Itse uskon Suunnitelmaan ja vapaaseen tahtoon. Olenko ristiriitainen? Jokainen voi toteuttaa Suunnitelmaa omalla persoonallisella tavallaan. Tällä hetkellä luulen, että Suunnitelma on paluu Alkulähteen yhteyteen kokemuksia rikkaampana ja omana kokonaisena suvereenina itsenään.

      --
      heisielu

    • Anonyymi

      Nimimerkki: Professori Realisti

      Olemassaoleminen tarkoittaa "kuulumista todellisten asioiden joukkoon", eli sitä, että olemassaolevalla asialla on *vähintään yksi* yhteinen ominaisuus kaikkien muiden olemassaolevien asioiden kanssa. *Todelliset* asiat puolestaan ovat sellaisia, jotka eivät ole mahdottomia. Eli, kaikki sellaiset asiat, jotka eivät ole mahdottomia ovat olemassa, koska mahdollinen asia ei ole *oikeasti* mahdollinen, jos se ei ole myös todellinen.

      Todellisia ovat sekä sellaiset asiat, jotka ovat havaintojen sisältöjä, että sellaiset asiat, jotka ovat havaintojen edellytyksiä, vaikkakaan havaintojen sisältöjen *tulkinta* ei tietenkään välttämättä vastaa sitä kohdetta, josta havainnon kohdetta kuvaava informaatio on peräisin. Mahdottomia puolestaan ovat sellaiset asiat, jotka ovat loogisesti ristiriitaisia, eivätkä ne siis voi olla sen enempää havaintojen sisältöjä kuin niiden edellytyksiäkään.

      Eli, esim. "loogisesti ristiriidaton kummitus" on siis todellinen ja siten siis olemassa, mutta esim. "sellainen loogisesti ristiriidaton kummitus, jonka kaikki havaitsevat juuri tällä hetkellä", ei ole todellinen, kuten ei myöskään "kaikkivaltias kummitus", koska sekään ei; nimestään ja sille kuvitelluista ominaisuuksista huolimatta, pysty aikaansaamaan esim. sitä, että se tulisi tietyllä hetkellä tietyn havaitsijan toimesta *sekä* havaituksi, *että* ei-havaituksi ja ei siis voi täyttää määritelmäänsä. Esim. "yliluonnollinen kummitus" puolestaan on sellainen otus, että siitä ei voida *tietää*, että onko se olemassa vai ei, mm. koska ei voida olla varmoja siitä, että ollaanko havaittu se, vai jotakin muuta...

      Myös esim. "1:n ja 3:n välissä lukusuoralla sijaitseva kokonaisluku" on todellinen, mutta esim. "1:n ja 2:n välissä lukusuoralla sijaitseva kokonaisluku" ei ole määritelmällisesti todellinen. Luvut ja muut senkaltaiset asiat, jotka eivät itsessään kykene *sisältämään* informaatiota, vaan *ovat* vain osa havaintojen tulkintoja, voivat tulla havaituiksi noina tulkintoina vain, jos havaitsija myös havaitsee jonkin sellaisen todellisen asian, joka tulee *tulkituksi kyseisen asian kuvaukseksi*.

      Eli, tuossa tapauksessa tulkinta kylläkin on todellinen, mutta edellyttää aina myös mm. jonkin informaation tallentumisen mahdollistavan sellaisen todellisen asian, jolla on toiminnallinen yhteys koettuun tulkinnan sisältöön. Voidaan ajatella, että nämä ja paljon mutkikkaammat ja abstraktimmat oliot ovat todellisten asioiden välisten yhteyksien malleja ja ovat todellisia noiden asioiden välisinä yhteyksinä, vaikkakaan nuo mallit eivät tietenkään välttämättä kuitenkaan *kuvaa* mitään sellaisia asioita, jotka olisivat tietyllä hetkellä tietyn havaitsijan toimesta havaittavissa tai edes todellisiakaan asioita...

      Itseasiassa, koska jokainen havainto edellyttää aina kaikki edellytyksensä, niin se on mahdollinen vain, jos todellisten asioiden joukon *kaikkien* osien kaikkien ominaisuuksien arvot mahdollistavat kyseisen havainnon sisällön, eli jos ajatellaan, että todellisuus on sellainen tilakone, jonka kaikilla osilla on väistämättä vähintään välillinen toiminnallinen yhteys toisiinsa, niin välillisesti jokainen havainto edellyttää sen, että noiden kaikkien osien ominaisuuksien arvot ovat nimenomaan tietyt, vaikka onkin ilmeistä, että samansisältöisten havaintojen toistuminen on mahdollista myös useilla erilaisilla noiden ominaisuuksien arvojen yhdistelmillä...

      Todellisuuden toimintaa selitettäessä muodostettavat mallit tietenkin ovat aina käytännöllisyys- ja käsiteltävyyssyistä abstraktioita ja siten kuvaavat todellisuutta vain osittain. Ilmeisesti, mitä kompleksisemmasta havainnosta tai havaintojoukosta on kyse, niin sitä pienempi osa noista todellisten asioiden joukon osien ominaisuuksien arvojen yhdistelmistä mahdollistavat kyseisen sisällön.

      Tämä pohdinta alkaa tulla filosofisesti mielenkiintoiseksi, jos oletetaan äärettömän kompleksinen todellisuus, koska silloin esim. sekä yksinkertaisia, että kompleksisia kummituksia olisi molempia ilmeisesti ääretön määrä, jolloin kysymykseksi muodostuu se, että miksi kuitenkin noita kompleksisempia kummituksia havaitaan harvemmin ja ilmeisesti syy tuohon on se, että noiden kompleksisempien kummitusten mutkikkaampia yksityiskohtia on keskimäärin vaikeampia havaita, johtuen havaitsijan *itsensä* kognitiivista rajoitteista ja monet yksinkertaisilta *näyttävät* kummitukset voivatkin siis oikeasti olla tuota kompleksisempaa tyyppiä, jonka joukon mahtavuus on siis potentiaalisesti ääretön...

      Näin filosofinen ajattelu auttaa siis myös selkiyttämään sitä, että mitkä asiat voivat oikeasti olla olemassa ja mitkä eivät. :D

      • Anonyymi

        Kompleksisuus voi syntyä todella yksinertaista säännöistä, eikä niitä tarvitse olla edes kovin montaa. Kokeissa on tutkittu ja havaittu, että säännöt synnyttävät helposti diskreettejä sarjoja, hajaantuvia ja suppenevia, jotka voivat olla kuin piin arvo, ei löydy toistuvuutta, näyttävät satunnaisilta. Tai tuottavat fraktaalinomaisia rakenteita. Nyt se mitä maailmankaikkeuden toiminnan tutkinta on, on periaatteessa lopputuleman tutkimista ja käänteistä insinöörityötä, jolla pyritään pääsemään kiinni siihen, että millä lauserakenteella voidaan ennustaa miten käy tai miten tilanne jostain hetkestä alkaen jatkuu. Se mitä on alettu tutkimaan oikeastaan aika vastoittain, on pellin alta löytyvät toistu uudet, joista voidaan päästä kiinni varsinaisiin sääntöihin. Jotka toteuttavat asioita kuten entropia tai painovoima, tai vaikka suhteellisuus, energia, fotoni jne. Ja joista voi löytyä selityksiä ajattelulle tai kummituksille. Se on kuitenkin todettava, että mahdollisten sääntöjen simulointi hetkestä nolla tähän päivään on jännä ajatus, koska maailmankaikkeuden toteuttama laskenta on kestänyt jo jonkinaikas. Ja toisaalta ei ole varmuutta voiko systeemin sisältä päätellä kaikkia systeemin sääntöjä. Osa asioista kun voi toteutua paikallisesti riippuen siitä mitä ympäristöstä löytyy. Joten kaikkea toteumaa ei välttämättä saada edes havaituksi koska rajallinen informaation siirtymisen nopeus. Tämä ei tarkoita siis välttämättä deterministisyuttä, ehkä enemmän emergenttisyyttä.
        --planeetta


    • Anonyymi

      Oikeassa olemiseen on montakin tietä. Mutta vain, jos on oikeasti olemassaan.

      • Anonyymi

        Mitä tarkoittaa oikeasti?


    • Anonyymi

      Oikeastaan niin, olemassaolematon on jos tanssii vasenkätisen rosvon kanssa.

    • Anonyymi

      Ole massa oleminen on massana olemisen kokemista jolloin olemme alttiita painovoiman vaikutuksille ja osana fysikaalisen maailman ilmiötä jossa meillä on tietoisuuden sijainti sen sijaan että se olisi hajallaan kaikkialla. Eri näkökulmissa pystymme käymään mielekästä keskustelua siitä miltä toinen näkökulma vaikuttaa omaan verrattuna ja luomaan kokonaiskuvan kaikista näkökulmista yhden sijaan. Tietenkin kaikki on vain mahdollisuus olevasta eikä lainkaan totta mutta meillä on se illuusio kokemuksesta jonka voimme jättää taakse kun se loppuu.

    • Anonyymi

      Kvanttifysiikan mukaan on vain todennäköisyyksiä. Mitään ei ole olemassa tasan 100% todennäköisyydellä, ja toisaalta ei ole ei-olevaakaan, vaan myös ei-olevaksi ajatellut asiat ovat häviävän pienellä todennäköisyydellä olemassa.
      Ei ole mitään periaatteellista eroa asioilla jotka ovat olemassa 99,999%, 50% tai 0,0001% todennäköisyyksillä. Kaikki ei-oleva ja oleva on samaa.

      • Anonyymi

        Nobel-palkittu fyysikko Gerard t Hooft on vastikään julkaissut deterministisen kvanttifysiikan matematiikkaan tarvittavia yhtälöitä. Ne ovat yhteensopivia myös Bellin teoreeman kanssa, ja osoittavat että kvanttifysiikka on täysin determinististä eikä siinä ole mitään mystistä "puhdasta satunnaisuutta".
        https://arxiv.org/pdf/2005.06374.pdf


      • Anonyymi

        "Ei ole mitään periaatteellista eroa asioilla jotka ovat olemassa 99,999%, 50% tai 0,0001% todennäköisyyksillä. Kaikki ei-oleva ja oleva on samaa"

        Periaatteessa on. Toista kutsutaan adjektiivilla "yleinen" ja toista adjektiivilla "harvinainen".

        Lausahduksesi olevasta ja ei-olevasta johtaa erikoiseen tilanteeseen. Se tarvitsee tarkennusta, koska on niin lyhyt lause, että sen voi ymmärtää monella tavalla ja myös väärin.


      • Anonyymi
        Anonyymi kirjoitti:

        "Ei ole mitään periaatteellista eroa asioilla jotka ovat olemassa 99,999%, 50% tai 0,0001% todennäköisyyksillä. Kaikki ei-oleva ja oleva on samaa"

        Periaatteessa on. Toista kutsutaan adjektiivilla "yleinen" ja toista adjektiivilla "harvinainen".

        Lausahduksesi olevasta ja ei-olevasta johtaa erikoiseen tilanteeseen. Se tarvitsee tarkennusta, koska on niin lyhyt lause, että sen voi ymmärtää monella tavalla ja myös väärin.

        Lausahdus on mielestäni periaatteessa mielenkiintoinen ja sitä voisi pitää jopa rtotena, ellei ymmärtäisi, ettei olemassaoloa voida määritellä. Siksi juuri se onkin nonsensea, kuten kaikki muutkin yritelmät määritellä olevaa/ei-olevaa.


    • Anonyymi

      Olemassa oleminen on erilaisuutta eli ympäristöstä erottumista.

      Täysin samanlainen eli homogeeninen ympäristö on havaitsijan kannalta tyhjä vaikka se todellisuudessa koostuisikin pienenpienistä hiukkasista (paljon pienemmistä kuin tunnetut alkeishiukkaset eli Planckin mittakaavan alikvanttihiukkassita tai "esi-hiukkaista" - sub-quantum particles) jotka kaikki pyörivät (spin) samalla nopeudella samaan suuntaan eli kaikki rakenne, muoto, ajallisuus & paikallisuus olisi vain mahdollista (potentiaalista) mutta ei todellista (aktuaalista) . Tällaisen olemattomuuden meren informaatiopotentiaali on ääretön.
      Jokaisella spinnaavalla esihiukkasella on ääretön määrä vektoreita ja niitä esihiukkasia on ääretön määrä joiden vektorisumma on yhteensä nolla.

      Jos oletetaan tällaisen homogeenisen ympäristön lisäksi myös jonkinlainen potentiaalinen havaitsija tai tietoisuus niin jo sen havaitsijan yritys havaita jotain tuottaa niitä eroavaisuuksia siinä homogeenisessä todellisuudessa jolloin samanaikaisesti syntyy sekä aika että avaruus jolloin ne esi-hiukkaset joiden nopeus ja spin poikkeaa niiden ympäristön hiukkasten ominaisuuksista muuttuvat havaittaviksi, mitattavissa oleviksi ja siis samalla olemassaoleviksi.

      Nykyisessä kosmologiassa väitetään kaikkeuden syntyneen tyhjästä eli tasapainossa olevien symmetrioiden (=nollasummavektorit) tasapainon rikkoutumisesta eli jonkinlaisesta kvanttifluktuaatiosta. Epätasapaino sinänsä jo tuottaa kaiken havaittavissa olevan aineen ja energian ja ns. lämpökuolema voi tapahtua vain jos se epätasapaino lakkaa olemasta.

      Tältä kannalta tarkasteltuna kaikki 3 keskeisintä kosmologista mallia (oskilloiva, steady state & Big Bang) ovat kaikki ainakin joiltain osin yhteensopivia keskenään. Tällainen malli voidaan tulkita haluttaessa sekä teistisesti että ateistisesti riippuen siitä minkälainen ontologinen status annetaan erilaistumisprosessin alkuunpanijalle eli havaitsijalle tai sen epäsymmetriarikon alkup. aiheuttajalle.

      (En ole ylläolevaa mallia itse kehittänyt vaan se löytyy esim. Joseph P. Farrellin teoksesta The Giza Death Star Deployed s.243-244 ja Farrell taas vuorostaan on käyttänyt lähteinä mm. Thomas Beardenin kirjoituksia Maxwellin alkup. yhtälöistä eli quaternion-kaavoista joissa otettiin huomioon sähkömagneettisen systeemin sisäinen rakenne tai stressi joka ei ole mitattavissa koska se koostuu ns. nollasummavektoreista)

      B

      • Anonyymi

        B:lle propsit, että opiskelee uutta eikä vaan toista samoja aiheita vuodesta toiseen!


      • Anonyymi

        Kosmologiassa ei väitetä universumin syntyneen tyhjästä tai kvanttifluktuaatioista yms.

        Eräs hypoteesi on mainitun oletuksen sisältävä, mutta tälle ei ole olemassa mitään todisteita. Tieteessä ei ole vielä menetelmää, jolla mainittua hypoteesia voisi tutkia empiirisesti.


      • Anonyymi
        Anonyymi kirjoitti:

        Kosmologiassa ei väitetä universumin syntyneen tyhjästä tai kvanttifluktuaatioista yms.

        Eräs hypoteesi on mainitun oletuksen sisältävä, mutta tälle ei ole olemassa mitään todisteita. Tieteessä ei ole vielä menetelmää, jolla mainittua hypoteesia voisi tutkia empiirisesti.

        "Tieteessä ei ole vielä menetelmää, jolla mainittua hypoteesia voisi tutkia empiirisesti."

        Kvanttifluktuaatioita voidaan tutkia, ja onkin tutkittu ja todistettu empiirisesti. Jos otetaan kuutiometri täysin tyhjää avaruutta, on todellakin niin että se ei pysy tyhjänä, ja sinne syntyy itsestään tyhjästä hiukkasia.


      • Anonyymi
        Anonyymi kirjoitti:

        "Tieteessä ei ole vielä menetelmää, jolla mainittua hypoteesia voisi tutkia empiirisesti."

        Kvanttifluktuaatioita voidaan tutkia, ja onkin tutkittu ja todistettu empiirisesti. Jos otetaan kuutiometri täysin tyhjää avaruutta, on todellakin niin että se ei pysy tyhjänä, ja sinne syntyy itsestään tyhjästä hiukkasia.

        Kvanttifluktuaatiolla ei ole todistettu olevan mitään tekemistä universumin synnyn kanssa.


      • Anonyymi
        Anonyymi kirjoitti:

        "Tieteessä ei ole vielä menetelmää, jolla mainittua hypoteesia voisi tutkia empiirisesti."

        Kvanttifluktuaatioita voidaan tutkia, ja onkin tutkittu ja todistettu empiirisesti. Jos otetaan kuutiometri täysin tyhjää avaruutta, on todellakin niin että se ei pysy tyhjänä, ja sinne syntyy itsestään tyhjästä hiukkasia.

        Nimimerkki: Professori Realisti

        "Kvanttifluktuaatioita voidaan tutkia, ja onkin tutkittu ja todistettu empiirisesti. Jos otetaan kuutiometri täysin tyhjää avaruutta, on todellakin niin että se ei pysy tyhjänä, ja sinne syntyy itsestään tyhjästä hiukkasia."

        Luonnollisen kielen epätäsmällinen käyttö johtaa näissä ja muissakin asioissa hyvin helposti harhaan, mutta yritetään silti selittää; edes jollakin karkeustasolla, koska tuo lainattu kuvaus ei ainakaan ole todellisen tilanteen tarkka kuvaus.

        Eli, 1) tuossa tapauksessa sinne ei itseasiassa suinkaan synny *tyhjästä* hiukkasia, vaan *kvanttifluktuaatioista*; joilla taas on "omat" syynsä. 2) Kvanttifluktuaatioita *itseänsähän* ei suoranaisesti voida tutkia, vaan niiden oletettuja *vaikutuksia havaintoihin*, joiden vaikutusten havaitsemiseen tietysti liittyvät kaikki havaitsemisen yleiset ja kvanttimekaaniset rajoitteet, eli niiden havaitseminen on mahdollista vain tietyillä edellytyksillä ja on ilmeistä, että esim. tyhjyyttä ei voida havaita ja ei ole myöskään mitään keinoa varmentaa sitä, että sen falsifiointitavat eivät olisi vastaavasti rajoitteisia.

        3) Noiden hiukkasten syntymisen kannalta oleellista ei myöskään suinkaan ole oletettu avaruuden *tyhjyys*, vaan taas itse *kvanttifluktuaatiot* ja tietysti myös niiden edellytykset, joita taas välttämättä ei tietenkään tiedetä tai edes voidakaan tietää, mutta kvanttifluktuaatiot eivät edellytä tyhjyyttä, joskin niiden *oletettujen vaikutusten tunnistaminen suhteessa havaintoihin* muiden vastaavien joukosta, on tietenkin edellytys sille, että voidaan uskottavasti ylipäätään puhuakaan noista havainnoista.

        4) Täysin tyhjää todellista avaruutta ei ylipäätään voi olla olemassa, koska kaikilla *todellisuuden* osilla on väistämättä vähintään yksi yhteinen ominaisuus, mikä tarkoittaa sitä, että ne sisältävät informaatiota ja täysin tyhjä avaruus ei voisi sisältää informaatiota. 5) Vaikka *näennäisesti* tyhjä avaruus voisikin sisältää informaatiota, niin siltikään ei voitaisi todistaa empiirisesti, että siitä syntyisi hiukkasia, koska ei ole olemassa mitään keinoa tutkia sellaista todellisuuden osaa, josta ei voida vastaanottaa informaatiota, eli mahdollinen informaatio ei siis ole peräisin tyhjästä olemattomasta avaruudesta, vaan esim. ensinnäkin kvanttifluktuaatioista ja edelleen jostain muusta; todennäköisesti ns. ei-lokaaleista todellisuuden muiden osien vaikutuksista.

        6) Kvanttifluktuaatioiden osalta mitattavuuden rajoitteet todellakin ovat oleellinen seikka, eli vaikka kvanttimekaanisten ilmiöiden eräs perusominaisuus tosiaan onkin aaltoluonne, niin *todelliset* fysikaaliset aallot koostuvat *silti* osistansa, joita osia taas ei kyetä tässä tapauksessa mittauksilla havaitsemaan, vaikka noiden osien välinen vuorovaikutus onkin epäilemättä syy siihen, että mittaustulokset eivät tosiaan ole tarkasti *ennustettavissa*, vaikka kaikki kvanttimekaaniset ilmiöt pohjimmiltaan käyttäytyvätkin epäilemättä *tarkoissa* tieteellisissä todellisuuden malleissa *deterministisesti*. Eli, todellisuus ei tietenkään suinkaan muutu epämääräiseksi sen seurauksena, että sen toiminnasta ei kyetä *laatimaan* täsmällisiä malleja, vaan itseasiassa päinvastoin "epätäsmällinen todellisuus" on oksymorooni. Eli, todellisuus on väistämättä täsmällistä ja epätodellinen on epätäsmällistä...

        7) Kvanttifluktuaatiot ovat *näennäisesti* pistemäisiä tapahtumia, mutta matemaattinen piste ei ole osa fysikaalista todellisuutta, koska matemaattinen piste ei voi sisältää mm. mitään energiaa, eikä informaatiota. Eli, tuolta osin kyse on vain epätäsmällisestä matemaattisesta mallista perustuen rajoitteisiin havaintoihin.

        8) Eli, kaikenkaikkiaan, kaikki "tyhjästä syntymiseen" , "satunnaisuuteen", "pistemäisten toimijoiden aikaansaamiin mystisiin vaikutuksiin" yms. maagisiin selityksiin viittaavat havaintojen tulkinnat ovat vähintään harhaanjohtavia ja epätäsmällisiä ja osin täysin virheellisiä tieteellisinä todellisuuden kuvauksina ja kertovat siis paljon enemmän niiden laatijoista ja noiden laatijoiden kognitiivisista rajoitteista, kuin noiden kuvausten tarkoitetuista kohteista.


      • Anonyymi
        Anonyymi kirjoitti:

        Nimimerkki: Professori Realisti

        "Kvanttifluktuaatioita voidaan tutkia, ja onkin tutkittu ja todistettu empiirisesti. Jos otetaan kuutiometri täysin tyhjää avaruutta, on todellakin niin että se ei pysy tyhjänä, ja sinne syntyy itsestään tyhjästä hiukkasia."

        Luonnollisen kielen epätäsmällinen käyttö johtaa näissä ja muissakin asioissa hyvin helposti harhaan, mutta yritetään silti selittää; edes jollakin karkeustasolla, koska tuo lainattu kuvaus ei ainakaan ole todellisen tilanteen tarkka kuvaus.

        Eli, 1) tuossa tapauksessa sinne ei itseasiassa suinkaan synny *tyhjästä* hiukkasia, vaan *kvanttifluktuaatioista*; joilla taas on "omat" syynsä. 2) Kvanttifluktuaatioita *itseänsähän* ei suoranaisesti voida tutkia, vaan niiden oletettuja *vaikutuksia havaintoihin*, joiden vaikutusten havaitsemiseen tietysti liittyvät kaikki havaitsemisen yleiset ja kvanttimekaaniset rajoitteet, eli niiden havaitseminen on mahdollista vain tietyillä edellytyksillä ja on ilmeistä, että esim. tyhjyyttä ei voida havaita ja ei ole myöskään mitään keinoa varmentaa sitä, että sen falsifiointitavat eivät olisi vastaavasti rajoitteisia.

        3) Noiden hiukkasten syntymisen kannalta oleellista ei myöskään suinkaan ole oletettu avaruuden *tyhjyys*, vaan taas itse *kvanttifluktuaatiot* ja tietysti myös niiden edellytykset, joita taas välttämättä ei tietenkään tiedetä tai edes voidakaan tietää, mutta kvanttifluktuaatiot eivät edellytä tyhjyyttä, joskin niiden *oletettujen vaikutusten tunnistaminen suhteessa havaintoihin* muiden vastaavien joukosta, on tietenkin edellytys sille, että voidaan uskottavasti ylipäätään puhuakaan noista havainnoista.

        4) Täysin tyhjää todellista avaruutta ei ylipäätään voi olla olemassa, koska kaikilla *todellisuuden* osilla on väistämättä vähintään yksi yhteinen ominaisuus, mikä tarkoittaa sitä, että ne sisältävät informaatiota ja täysin tyhjä avaruus ei voisi sisältää informaatiota. 5) Vaikka *näennäisesti* tyhjä avaruus voisikin sisältää informaatiota, niin siltikään ei voitaisi todistaa empiirisesti, että siitä syntyisi hiukkasia, koska ei ole olemassa mitään keinoa tutkia sellaista todellisuuden osaa, josta ei voida vastaanottaa informaatiota, eli mahdollinen informaatio ei siis ole peräisin tyhjästä olemattomasta avaruudesta, vaan esim. ensinnäkin kvanttifluktuaatioista ja edelleen jostain muusta; todennäköisesti ns. ei-lokaaleista todellisuuden muiden osien vaikutuksista.

        6) Kvanttifluktuaatioiden osalta mitattavuuden rajoitteet todellakin ovat oleellinen seikka, eli vaikka kvanttimekaanisten ilmiöiden eräs perusominaisuus tosiaan onkin aaltoluonne, niin *todelliset* fysikaaliset aallot koostuvat *silti* osistansa, joita osia taas ei kyetä tässä tapauksessa mittauksilla havaitsemaan, vaikka noiden osien välinen vuorovaikutus onkin epäilemättä syy siihen, että mittaustulokset eivät tosiaan ole tarkasti *ennustettavissa*, vaikka kaikki kvanttimekaaniset ilmiöt pohjimmiltaan käyttäytyvätkin epäilemättä *tarkoissa* tieteellisissä todellisuuden malleissa *deterministisesti*. Eli, todellisuus ei tietenkään suinkaan muutu epämääräiseksi sen seurauksena, että sen toiminnasta ei kyetä *laatimaan* täsmällisiä malleja, vaan itseasiassa päinvastoin "epätäsmällinen todellisuus" on oksymorooni. Eli, todellisuus on väistämättä täsmällistä ja epätodellinen on epätäsmällistä...

        7) Kvanttifluktuaatiot ovat *näennäisesti* pistemäisiä tapahtumia, mutta matemaattinen piste ei ole osa fysikaalista todellisuutta, koska matemaattinen piste ei voi sisältää mm. mitään energiaa, eikä informaatiota. Eli, tuolta osin kyse on vain epätäsmällisestä matemaattisesta mallista perustuen rajoitteisiin havaintoihin.

        8) Eli, kaikenkaikkiaan, kaikki "tyhjästä syntymiseen" , "satunnaisuuteen", "pistemäisten toimijoiden aikaansaamiin mystisiin vaikutuksiin" yms. maagisiin selityksiin viittaavat havaintojen tulkinnat ovat vähintään harhaanjohtavia ja epätäsmällisiä ja osin täysin virheellisiä tieteellisinä todellisuuden kuvauksina ja kertovat siis paljon enemmän niiden laatijoista ja noiden laatijoiden kognitiivisista rajoitteista, kuin noiden kuvausten tarkoitetuista kohteista.

        Jos kvanttifluktuaatiot jotenkin riippuisivat muusta aineesta ja olisivat esim. muualta tunneloituneita hiukkasia, tällöin galaksienvälisessä avaruudessa pitäisi olla todella vähän kvanttifluktuaatioita ja maapallon läheisyydessä paljon. On paljon todennäköisempää tunneloitua 10cm tai metrinkin matka kuin jotain valovuosien matkoja. Kilometrinkin tunneloituminen on niin epätodennäköistä ettei sitä tapahdu koko universumin eliniän aikana.
        Kuitenkin kvanttifluktuaatioita on yhtä paljon kaikkialla avaruudessa, sekä galaksienvälisessä avaruudessa että maapallolla, ja niiden energia vastaa yhtä protonia kuutiometrissä avaruutta. Se on täysin itsestään syntynyttä, tyhjästä tullutta ainetta.


      • Anonyymi
        Anonyymi kirjoitti:

        Jos kvanttifluktuaatiot jotenkin riippuisivat muusta aineesta ja olisivat esim. muualta tunneloituneita hiukkasia, tällöin galaksienvälisessä avaruudessa pitäisi olla todella vähän kvanttifluktuaatioita ja maapallon läheisyydessä paljon. On paljon todennäköisempää tunneloitua 10cm tai metrinkin matka kuin jotain valovuosien matkoja. Kilometrinkin tunneloituminen on niin epätodennäköistä ettei sitä tapahdu koko universumin eliniän aikana.
        Kuitenkin kvanttifluktuaatioita on yhtä paljon kaikkialla avaruudessa, sekä galaksienvälisessä avaruudessa että maapallolla, ja niiden energia vastaa yhtä protonia kuutiometrissä avaruutta. Se on täysin itsestään syntynyttä, tyhjästä tullutta ainetta.

        Nimimerkki: Professori Realisti

        "Jos kvanttifluktuaatiot jotenkin riippuisivat muusta aineesta ja olisivat esim. muualta tunneloituneita hiukkasia, tällöin galaksienvälisessä avaruudessa pitäisi olla todella vähän kvanttifluktuaatioita ja maapallon läheisyydessä paljon."

        Ei-lokaaleja nettovaikutuksia arvioitaessa täytyy ottaa huomioon se, esim. Olbersin paradoksista tuttu seikka, että äärettömässä avaruudessa kaukaisempien kohteiden yhteisvaikutukset kuhunkin tiettyyn yksittäiseen kohteeseen olisivat lopulta itseasiassa äärettömät, vaikka niiden yksittäiset vaikutukset olisivatkin paljon vähäisemmät kuin läheisempien, mistä seuraisi se, että läheisempien kohteiden vaikutuksilla ei itse asiassa olisi edes mitään oleellista suhteellista vaikutustakaan lopputulokseen...

        Käytännössähän noin ei tuon Olbersin paradoksiin liittyvän ilmiön osalta kuitenkaan käy havaintojen osalta, *koska* valonnopeus on äärellinen, mutta toisaalta kvanttimekaaninen vuorovaikutus ilmeisesti tapahtuu viiveettä tai ainakin se on niin viiveetöntä, että viivettä ei kyetä mittaamaan. Itseasiassa on siis hyvin kyseenalaista, että ovatko havaitunkaltaiset kvanttifluktuaatioiden vaikutukset edes mahdollisiakaan ilman hyvin merkittäviä ei-lokaaleja syitä, koska "paikallisen tunneloitumisen" todennäköisyys todellakin on erittäin erittäin pieni. Käytännössän kvanttifluktuaatioiden graafiset; kaoottisen aallokon kaltaiset, kuvaukset synnyttävät sen vaikutelman, että tuon ilmiön syynä on se, että kuhunkin avaruuden "pisteeseen" kohdistuu kaikista eri suunnista tulevia hiukan voimakkuudeltaan vaihtelevia vaikutuksia.

        "Kuitenkin kvanttifluktuaatioita on yhtä paljon kaikkialla avaruudessa, sekä galaksienvälisessä avaruudessa että maapallolla, ja niiden energia vastaa yhtä protonia kuutiometrissä avaruutta."

        Kvanttifluktuaatioiden homogeenisuus; tai ainakin siis homogeenisuus mittaustarkkuuden puitteissa, on taatusti myös oma oletukseni. Eli, en usko, että "paikallisilla syillä" olisi tuon ilmiön voimakkuuteen ainakaan mittaustarkkuuden puitteissa olevaa vaikutusta. Se, mikä tässä on mielenkiintoista on se, että havaitsemiemme kvanttifluktuaatioiden keskimääräinen energiatiheys tosiaan on äärellinen, josta voidaan tietenkin päätellä, että joko 1) todellisuus on energiamäärältään äärellinen; mikä kuulostaa epäilyttävältä tai 2) jostain muusta syystä ei-lokaalisten vaikutusten voimakkuus "normalisoituu" äärelliseksi, mikä syy epäilemättä liittyy todellisuuden osien väliseen vuorovaikutukseen ja on erittäin relevantti, joskin tietenkin hankalasti tutkittava, tieteellinen tutkimuskohde...

        "Se on täysin itsestään syntynyttä, tyhjästä tullutta ainetta."

        No ei taatusti ole! Johan tuon selitin, mutta lisäksi voidaan todeta myös, että jos tyhjästä tosiaan voisi tulla "täysin itsestään" jotakin, niin mikään ei voisi estää sitä, etteikö sitä tulisi sieltä *jatkuvasti äärettömästi*, mikä taas on tietenkin täysin havaintojen vastainen tilanne ja itseasiassa myös sellainen asia, joka estäisi kaikki havainnotkin...

        Eli, jostakinhan kaiken kvanttifluktuaatioihin liittyvän energiankin täytyy tulla ja tieteen tehtävänä on tietysti selvittää tai ainakin yrittää selvittää, että *mistä* se on peräisin ja tietenkään sekään, että vaikka tiede ei koskaan tuossa onnistuisi, ei tarkoittaisi sitä, että tuo energia tulisi tyhjästä, eli yleisessä tapauksessa se, että jos jonkin syytä *ei havaita* ei tietenkään seuraa, että syytä ei silti olisi olemassa ja tuollainen tyhjästä syntymisen johtopäätös on siis kaikenkaikkiaan non sequitur.


      • Anonyymi
        Anonyymi kirjoitti:

        Nimimerkki: Professori Realisti

        "Jos kvanttifluktuaatiot jotenkin riippuisivat muusta aineesta ja olisivat esim. muualta tunneloituneita hiukkasia, tällöin galaksienvälisessä avaruudessa pitäisi olla todella vähän kvanttifluktuaatioita ja maapallon läheisyydessä paljon."

        Ei-lokaaleja nettovaikutuksia arvioitaessa täytyy ottaa huomioon se, esim. Olbersin paradoksista tuttu seikka, että äärettömässä avaruudessa kaukaisempien kohteiden yhteisvaikutukset kuhunkin tiettyyn yksittäiseen kohteeseen olisivat lopulta itseasiassa äärettömät, vaikka niiden yksittäiset vaikutukset olisivatkin paljon vähäisemmät kuin läheisempien, mistä seuraisi se, että läheisempien kohteiden vaikutuksilla ei itse asiassa olisi edes mitään oleellista suhteellista vaikutustakaan lopputulokseen...

        Käytännössähän noin ei tuon Olbersin paradoksiin liittyvän ilmiön osalta kuitenkaan käy havaintojen osalta, *koska* valonnopeus on äärellinen, mutta toisaalta kvanttimekaaninen vuorovaikutus ilmeisesti tapahtuu viiveettä tai ainakin se on niin viiveetöntä, että viivettä ei kyetä mittaamaan. Itseasiassa on siis hyvin kyseenalaista, että ovatko havaitunkaltaiset kvanttifluktuaatioiden vaikutukset edes mahdollisiakaan ilman hyvin merkittäviä ei-lokaaleja syitä, koska "paikallisen tunneloitumisen" todennäköisyys todellakin on erittäin erittäin pieni. Käytännössän kvanttifluktuaatioiden graafiset; kaoottisen aallokon kaltaiset, kuvaukset synnyttävät sen vaikutelman, että tuon ilmiön syynä on se, että kuhunkin avaruuden "pisteeseen" kohdistuu kaikista eri suunnista tulevia hiukan voimakkuudeltaan vaihtelevia vaikutuksia.

        "Kuitenkin kvanttifluktuaatioita on yhtä paljon kaikkialla avaruudessa, sekä galaksienvälisessä avaruudessa että maapallolla, ja niiden energia vastaa yhtä protonia kuutiometrissä avaruutta."

        Kvanttifluktuaatioiden homogeenisuus; tai ainakin siis homogeenisuus mittaustarkkuuden puitteissa, on taatusti myös oma oletukseni. Eli, en usko, että "paikallisilla syillä" olisi tuon ilmiön voimakkuuteen ainakaan mittaustarkkuuden puitteissa olevaa vaikutusta. Se, mikä tässä on mielenkiintoista on se, että havaitsemiemme kvanttifluktuaatioiden keskimääräinen energiatiheys tosiaan on äärellinen, josta voidaan tietenkin päätellä, että joko 1) todellisuus on energiamäärältään äärellinen; mikä kuulostaa epäilyttävältä tai 2) jostain muusta syystä ei-lokaalisten vaikutusten voimakkuus "normalisoituu" äärelliseksi, mikä syy epäilemättä liittyy todellisuuden osien väliseen vuorovaikutukseen ja on erittäin relevantti, joskin tietenkin hankalasti tutkittava, tieteellinen tutkimuskohde...

        "Se on täysin itsestään syntynyttä, tyhjästä tullutta ainetta."

        No ei taatusti ole! Johan tuon selitin, mutta lisäksi voidaan todeta myös, että jos tyhjästä tosiaan voisi tulla "täysin itsestään" jotakin, niin mikään ei voisi estää sitä, etteikö sitä tulisi sieltä *jatkuvasti äärettömästi*, mikä taas on tietenkin täysin havaintojen vastainen tilanne ja itseasiassa myös sellainen asia, joka estäisi kaikki havainnotkin...

        Eli, jostakinhan kaiken kvanttifluktuaatioihin liittyvän energiankin täytyy tulla ja tieteen tehtävänä on tietysti selvittää tai ainakin yrittää selvittää, että *mistä* se on peräisin ja tietenkään sekään, että vaikka tiede ei koskaan tuossa onnistuisi, ei tarkoittaisi sitä, että tuo energia tulisi tyhjästä, eli yleisessä tapauksessa se, että jos jonkin syytä *ei havaita* ei tietenkään seuraa, että syytä ei silti olisi olemassa ja tuollainen tyhjästä syntymisen johtopäätös on siis kaikenkaikkiaan non sequitur.

        "Eli, jostakinhan kaiken kvanttifluktuaatioihin liittyvän energiankin täytyy tulla ja tieteen tehtävänä on tietysti selvittää tai ainakin yrittää selvittää, että *mistä* se on peräisin"

        Kvanttifluktuaatiossa ilmaantuu virtuaalinen hiukkaspari ainoastaan äärimmäisen lyhyeksi aikaa ja sen jälkeen se taas katoaa ja tyhjiön energia on taas 0, kunnes ilmaantuu uusi virtuaalihiukkaspari joka taas katoaa. Energian ei tarvitse olla mistään peräisin, se voi hetkellisesti poiketa nollasta, kuhan palautuu nopeasti taas takaisin nollaan. Tosin äärimmäisen pienellä todennäköisyydellä energiaa voi syntyä pidemmäksikin aikaa, eikä hiukkaspari heti tuhoudukaan.


      • Anonyymi
        Anonyymi kirjoitti:

        "Eli, jostakinhan kaiken kvanttifluktuaatioihin liittyvän energiankin täytyy tulla ja tieteen tehtävänä on tietysti selvittää tai ainakin yrittää selvittää, että *mistä* se on peräisin"

        Kvanttifluktuaatiossa ilmaantuu virtuaalinen hiukkaspari ainoastaan äärimmäisen lyhyeksi aikaa ja sen jälkeen se taas katoaa ja tyhjiön energia on taas 0, kunnes ilmaantuu uusi virtuaalihiukkaspari joka taas katoaa. Energian ei tarvitse olla mistään peräisin, se voi hetkellisesti poiketa nollasta, kuhan palautuu nopeasti taas takaisin nollaan. Tosin äärimmäisen pienellä todennäköisyydellä energiaa voi syntyä pidemmäksikin aikaa, eikä hiukkaspari heti tuhoudukaan.

        Nimimerkki: Professori Realisti

        "Energian ei tarvitse olla mistään peräisin, se voi hetkellisesti poiketa nollasta, kuhan palautuu nopeasti taas takaisin nollaan."

        Tuo on edelleen perustelematon non sequitur, eikä edes selitäkään mitään..., eli kuulostaa aivan uskontunnustukselta.

        "Tosin äärimmäisen pienellä todennäköisyydellä energiaa voi syntyä pidemmäksikin aikaa, eikä hiukkaspari heti tuhoudukaan."

        Tieteellisen teorian tulee selittää se, että miksi todennäköisyys on juuri se, minkä sen havaitaan olevan, eikä sisältää spekulointia tyhjästä syntymisen kaltaisilla ilmeisen mahdottomilla asioilla.


      • Anonyymi
        Anonyymi kirjoitti:

        Nimimerkki: Professori Realisti

        "Energian ei tarvitse olla mistään peräisin, se voi hetkellisesti poiketa nollasta, kuhan palautuu nopeasti taas takaisin nollaan."

        Tuo on edelleen perustelematon non sequitur, eikä edes selitäkään mitään..., eli kuulostaa aivan uskontunnustukselta.

        "Tosin äärimmäisen pienellä todennäköisyydellä energiaa voi syntyä pidemmäksikin aikaa, eikä hiukkaspari heti tuhoudukaan."

        Tieteellisen teorian tulee selittää se, että miksi todennäköisyys on juuri se, minkä sen havaitaan olevan, eikä sisältää spekulointia tyhjästä syntymisen kaltaisilla ilmeisen mahdottomilla asioilla.

        Vaihtoehto on että systeemi on täysin staattinen ja kiinteä, eikä sen energiassa ole mitään fluktuaatioita. Se taas tuntuu todella epäluonnolliselta ja järjenvastaiselta/newtonilaiselta. Ainahan kvanttimekaanisissa systeemeissä on fluktuaatioita ja epätarkkuutta. Vastakkaista eli staattista tilannetta ei ole ikinä missään havaittu eikä sellaisen olemassaolosta ole yhtään viitettä.


      • Anonyymi
        Anonyymi kirjoitti:

        Vaihtoehto on että systeemi on täysin staattinen ja kiinteä, eikä sen energiassa ole mitään fluktuaatioita. Se taas tuntuu todella epäluonnolliselta ja järjenvastaiselta/newtonilaiselta. Ainahan kvanttimekaanisissa systeemeissä on fluktuaatioita ja epätarkkuutta. Vastakkaista eli staattista tilannetta ei ole ikinä missään havaittu eikä sellaisen olemassaolosta ole yhtään viitettä.

        Nimimerkki: Professori Realisti

        "Vaihtoehto on että systeemi on täysin staattinen ja kiinteä, eikä sen energiassa ole mitään fluktuaatioita. Se taas tuntuu todella epäluonnolliselta ja järjenvastaiselta/newtonilaiselta."

        Staattisuushan ei ole epämääräisyyden, vaan dynaamisuuden vastakohta, eikä tuollainen staattinen järjestelmä ole tässä ainakaan ensisijainen vaihtoehto ja voisi kaiketi olla paikkansapitävä vain, jos havaitut muutokset olisivat illuusioita, mikä kylläkin on filosofisesti hyvin mielenkiintoinen idea. Kuten jo aiemmin totesin, muutokset havainnoissa ovat selitettävissä myös todellisuuden osien ei-lokaalilla vuorovaikutuksella, mikä on tässä se ensisijainen vaihtoehto...

        "Ainahan kvanttimekaanisissa systeemeissä on fluktuaatioita ja epätarkkuutta."

        Eihän kvanttimekaanisissa, eikä missään muissakaan systeemeissä koskaan *oikeasti* voi olla mitään epätarkkuutta, vaan tuo epätarkkuus on vain niiden *kuvauksissa*. Eli, vaikka kuvaus olisikin tarkoituksellisesti laadittu *tarkasti epätarkaksi*, niin epätarkka se on silti todellisuuden kuvauksena, koska se kuvaa todellisuutta epätarkkana, vaikka epätarkka todellisuus siis on oksymorooni...

        Tietenkin tuollainen kuvaus voi tosin kuvata tiettyjä *havaintoja* riittävän tarkasti, jonkin tarkoituksen kannalta, mutta havaitsemisellahan on aina tietyt rajoitteensa, eikä havaintojen perusteella voida loogisesti päätyä sellaiseen johtopäätökseen, että todellisuus olisi epämääräinen, vaan vain esim. siihen johtopäätökseen, että *jostain syystä* havainnot eivät ole vakioisia.

        Todelliset asiat eivät koskaan voi olla epätarkkoja, koska olemassa on vain kaksi perusvaihtoehtoa, eli joko yksittäinen elementaarinen mittaus tuottaa tulokseksi 0:n tai 1:n. Mitään välimuotoa ei ole, eikä voi olla, ensinnäkin, koska pätee "tertium non datur" ja toisekseen, koska todellisuus on kvantittunut. Ja jos taas tarkastellaan yksittäisten mittaustulosten joukkoa, niin silloinkin kukin yksittäinen; eli *todellinen* mittaustuloksen arvo voi olla vain joko 0 tai 1, eikä mikään välimuoto. Jos taas mittaus tuottaa eri mittauskerroilla eri arvon, vaikka oletetaan mitattavan samaa kohdetta, niin se johtuu siitä, että mittausta ei olla suoritettu samalla ajanhetkellä tai mitataan eri kohdetta.

        Voi tietenkin silti olla niin, että emme *tiedä* sitä, että mikä jokin mittaustulos on tai, että tiedämme sen vain osittain, esim. koska sitä kuvaava informaatio on hajautunut ja käytettävissämme on siitä vain osa, mutta silti kukin tuollainen mittaustulos voi *tiedettynä* olla arvoltaan vain 0 tai 1. Tietenkin, toisaalta, jos laskemme yksittäisten mittaustulosten perusteella muita kompleksisempia asioita kuvaavia arvoja, niin ne voivat olla muutakin kuin 0 tai 1, mutta ei sekään tietenkään tarkoita sitä, että todellisuus muuttuisi maagisesti epämääräiseksi, vaan vain sitä, että kuvattava todellisuuden osa on tuossa tapauksessa kompleksisempi ja sitä kuvaavaa informaatiota on käytettävissä enemmän kuin yksi bitti.

        "Vastakkaista eli staattista tilannetta ei ole ikinä missään havaittu eikä sellaisen olemassaolosta ole yhtään viitettä."

        Eihän dynaamisuuden edellytys siis tietenkään ole epätarkkuus, vaan epätarkkuus päinvastoin tekisi dynaamisuuden mahdottomaksi, koska epätarkat asiat eivät voi olla todellisia. Dynaamisuudessa on vain kyse siitä, että todellisuuden osien väliset suhteet; tai ainakin niitä vastaaviksi oletetut havainnot, ovat eri ajanhetkillä erilaisia, mikä taas on tietenkin aivan luonnollista, dynaamisesti käyttäytyvän, eli itsensä kanssa vuorovaikuttavan, todellisuuden tapauksessa.

        Jos mittaustulos on epätarkka, niin vika on mittalaitteessa, vääränlaisen mittalaitteen käytössä kyseiseen tarkoitukseen, väärästä mittaustulosten tulkintatavasta tms. Koskaan ei olla havaittu mitään *reaalisten mittausten kohteiden* epätarkkuutta, eikä edes elementaaristen mittaustulostenkaan epätarkkuutta, koska kukin niistä on arvoltaan juuri se, mikä se sitten onkin, vaikka ei tietenkään välttämättä kuvaakaan oikeellisesti sitä kohdetta, jota mittauksen oletetaan kuvaavan, eli esim. voidaan olettaa mitattavan samaa kohdetta eri mittauskerroilla, vaikka oikeasti mitataankin eri kohdetta ja tietenkin myös, jos mitattava yksinkertaiseksi oletettu kohde onkin oikeasti kompleksinen, niin on tietenkin täysin mahdollista, että siihen kohdistuvat mittaukset tuottavat eri mittauskerroilla erilaisia arvoja, vaikka kohteessa itsessään ei olekaan mitään epämääräistä...


      • Anonyymi
        Anonyymi kirjoitti:

        Nimimerkki: Professori Realisti

        "Vaihtoehto on että systeemi on täysin staattinen ja kiinteä, eikä sen energiassa ole mitään fluktuaatioita. Se taas tuntuu todella epäluonnolliselta ja järjenvastaiselta/newtonilaiselta."

        Staattisuushan ei ole epämääräisyyden, vaan dynaamisuuden vastakohta, eikä tuollainen staattinen järjestelmä ole tässä ainakaan ensisijainen vaihtoehto ja voisi kaiketi olla paikkansapitävä vain, jos havaitut muutokset olisivat illuusioita, mikä kylläkin on filosofisesti hyvin mielenkiintoinen idea. Kuten jo aiemmin totesin, muutokset havainnoissa ovat selitettävissä myös todellisuuden osien ei-lokaalilla vuorovaikutuksella, mikä on tässä se ensisijainen vaihtoehto...

        "Ainahan kvanttimekaanisissa systeemeissä on fluktuaatioita ja epätarkkuutta."

        Eihän kvanttimekaanisissa, eikä missään muissakaan systeemeissä koskaan *oikeasti* voi olla mitään epätarkkuutta, vaan tuo epätarkkuus on vain niiden *kuvauksissa*. Eli, vaikka kuvaus olisikin tarkoituksellisesti laadittu *tarkasti epätarkaksi*, niin epätarkka se on silti todellisuuden kuvauksena, koska se kuvaa todellisuutta epätarkkana, vaikka epätarkka todellisuus siis on oksymorooni...

        Tietenkin tuollainen kuvaus voi tosin kuvata tiettyjä *havaintoja* riittävän tarkasti, jonkin tarkoituksen kannalta, mutta havaitsemisellahan on aina tietyt rajoitteensa, eikä havaintojen perusteella voida loogisesti päätyä sellaiseen johtopäätökseen, että todellisuus olisi epämääräinen, vaan vain esim. siihen johtopäätökseen, että *jostain syystä* havainnot eivät ole vakioisia.

        Todelliset asiat eivät koskaan voi olla epätarkkoja, koska olemassa on vain kaksi perusvaihtoehtoa, eli joko yksittäinen elementaarinen mittaus tuottaa tulokseksi 0:n tai 1:n. Mitään välimuotoa ei ole, eikä voi olla, ensinnäkin, koska pätee "tertium non datur" ja toisekseen, koska todellisuus on kvantittunut. Ja jos taas tarkastellaan yksittäisten mittaustulosten joukkoa, niin silloinkin kukin yksittäinen; eli *todellinen* mittaustuloksen arvo voi olla vain joko 0 tai 1, eikä mikään välimuoto. Jos taas mittaus tuottaa eri mittauskerroilla eri arvon, vaikka oletetaan mitattavan samaa kohdetta, niin se johtuu siitä, että mittausta ei olla suoritettu samalla ajanhetkellä tai mitataan eri kohdetta.

        Voi tietenkin silti olla niin, että emme *tiedä* sitä, että mikä jokin mittaustulos on tai, että tiedämme sen vain osittain, esim. koska sitä kuvaava informaatio on hajautunut ja käytettävissämme on siitä vain osa, mutta silti kukin tuollainen mittaustulos voi *tiedettynä* olla arvoltaan vain 0 tai 1. Tietenkin, toisaalta, jos laskemme yksittäisten mittaustulosten perusteella muita kompleksisempia asioita kuvaavia arvoja, niin ne voivat olla muutakin kuin 0 tai 1, mutta ei sekään tietenkään tarkoita sitä, että todellisuus muuttuisi maagisesti epämääräiseksi, vaan vain sitä, että kuvattava todellisuuden osa on tuossa tapauksessa kompleksisempi ja sitä kuvaavaa informaatiota on käytettävissä enemmän kuin yksi bitti.

        "Vastakkaista eli staattista tilannetta ei ole ikinä missään havaittu eikä sellaisen olemassaolosta ole yhtään viitettä."

        Eihän dynaamisuuden edellytys siis tietenkään ole epätarkkuus, vaan epätarkkuus päinvastoin tekisi dynaamisuuden mahdottomaksi, koska epätarkat asiat eivät voi olla todellisia. Dynaamisuudessa on vain kyse siitä, että todellisuuden osien väliset suhteet; tai ainakin niitä vastaaviksi oletetut havainnot, ovat eri ajanhetkillä erilaisia, mikä taas on tietenkin aivan luonnollista, dynaamisesti käyttäytyvän, eli itsensä kanssa vuorovaikuttavan, todellisuuden tapauksessa.

        Jos mittaustulos on epätarkka, niin vika on mittalaitteessa, vääränlaisen mittalaitteen käytössä kyseiseen tarkoitukseen, väärästä mittaustulosten tulkintatavasta tms. Koskaan ei olla havaittu mitään *reaalisten mittausten kohteiden* epätarkkuutta, eikä edes elementaaristen mittaustulostenkaan epätarkkuutta, koska kukin niistä on arvoltaan juuri se, mikä se sitten onkin, vaikka ei tietenkään välttämättä kuvaakaan oikeellisesti sitä kohdetta, jota mittauksen oletetaan kuvaavan, eli esim. voidaan olettaa mitattavan samaa kohdetta eri mittauskerroilla, vaikka oikeasti mitataankin eri kohdetta ja tietenkin myös, jos mitattava yksinkertaiseksi oletettu kohde onkin oikeasti kompleksinen, niin on tietenkin täysin mahdollista, että siihen kohdistuvat mittaukset tuottavat eri mittauskerroilla erilaisia arvoja, vaikka kohteessa itsessään ei olekaan mitään epämääräistä...

        "Eihän kvanttimekaanisissa, eikä missään muissakaan systeemeissä koskaan *oikeasti* voi olla mitään epätarkkuutta..."

        Tarkka ajan ja paikan hetki edellyttää havainnon kohteena olevan prosessin pysäyttämistä ja vain tarkalla ajan (kesto=0) ja paikan hetkellä jokin voi olla tarkasti määriteltyä.

        Voiko luonnon prosesseja edes sillä tavalla pysäyttää mielivaltaisesti? Minusta ei voi.

        Koska mittauksia ei voi suorittaa niin että niiden kesto olisi nolla eikä todellisuudessa ole muutenkaan sellaisia "pysäytyskuvia" eli tarkasti määriteltävissä olevia objekteja kuten esim. luonnollisessa kielessä (jossa oletetaan kaikki fysikaaliset objektit jotenkin staattisesti olemassaolevina) niin se epämääräisyys on lievennettävissä ainoastaan ehkä niin että tieteen objekteja aletaan tutkia nimenomaan prosessisysteemeinä eikä staattisina esineen kaltaisina objekteina eli ts. otetaan huomioon vain kokonaiset prosessit ja niiden takaisinkytkennät jonkinlaisena holistisena systeeminä ja samalla luovutaan reduktionismista.

        Atomit, molekyylit, eliöt jne. ovat siis ne ainoat konkreettiset tutkittavissa olevat prosessit kun taas ns. alkeishiukkaset ovat pelkkiä abstraktioita joita ei voi erottaa niistä prosesseista joiden osia ne ovat ilman että se konkreettisuus samalla katoaa eli ts. atomit, molekyylit, biologiset oliot lajinsa jäseninä ovat pienimpiä mahdollisia prosesseja joilla on tietyt yksiselitteiset ominaisuudet joita niiden osilla ei ole .

        Kaikki konkreettiset oliot ja niiden joukot ovat prosesseja eli niillä on ajallinen kesto ja usein monenlaisia (takaisinkykentä) luuppeja eli syklejä eikä yhtä sen syklin osaa voi tarkastella eikä empiirisesti mitata irrallaan siitä kokonaissykilistä joka taas koostuu lukuisista sisäkkäisistä ja/tai rinnakkaisista sykleistä.

        2-arvoinen (kyllä/ei) logiikka pätee binäärisissä systeemeissä ja vaikka ns. kvanttimaailma ilmeisesti onkin jollain tavalla epäjatkuva ja epälineaarinen niin siitä ei mitenkään välttämättä seuraa että se olisi myös digitaalinen tai 2-arvoisesti binäärinen.

        Ei se superdeterminismi muutu pelkästään sillä tavalla todelliseksi että joitakuita eksaktisuus ja matemaattinen tarkkuus & ehdoton determinismi suuresti miellyttää ja vastaavasti kaikenlainen epätarkkuus, tilastollinen ontologia ja valinnan vapaus eivät taas miellytä :D


        B


      • Anonyymi
        Anonyymi kirjoitti:

        Nimimerkki: Professori Realisti

        "Vaihtoehto on että systeemi on täysin staattinen ja kiinteä, eikä sen energiassa ole mitään fluktuaatioita. Se taas tuntuu todella epäluonnolliselta ja järjenvastaiselta/newtonilaiselta."

        Staattisuushan ei ole epämääräisyyden, vaan dynaamisuuden vastakohta, eikä tuollainen staattinen järjestelmä ole tässä ainakaan ensisijainen vaihtoehto ja voisi kaiketi olla paikkansapitävä vain, jos havaitut muutokset olisivat illuusioita, mikä kylläkin on filosofisesti hyvin mielenkiintoinen idea. Kuten jo aiemmin totesin, muutokset havainnoissa ovat selitettävissä myös todellisuuden osien ei-lokaalilla vuorovaikutuksella, mikä on tässä se ensisijainen vaihtoehto...

        "Ainahan kvanttimekaanisissa systeemeissä on fluktuaatioita ja epätarkkuutta."

        Eihän kvanttimekaanisissa, eikä missään muissakaan systeemeissä koskaan *oikeasti* voi olla mitään epätarkkuutta, vaan tuo epätarkkuus on vain niiden *kuvauksissa*. Eli, vaikka kuvaus olisikin tarkoituksellisesti laadittu *tarkasti epätarkaksi*, niin epätarkka se on silti todellisuuden kuvauksena, koska se kuvaa todellisuutta epätarkkana, vaikka epätarkka todellisuus siis on oksymorooni...

        Tietenkin tuollainen kuvaus voi tosin kuvata tiettyjä *havaintoja* riittävän tarkasti, jonkin tarkoituksen kannalta, mutta havaitsemisellahan on aina tietyt rajoitteensa, eikä havaintojen perusteella voida loogisesti päätyä sellaiseen johtopäätökseen, että todellisuus olisi epämääräinen, vaan vain esim. siihen johtopäätökseen, että *jostain syystä* havainnot eivät ole vakioisia.

        Todelliset asiat eivät koskaan voi olla epätarkkoja, koska olemassa on vain kaksi perusvaihtoehtoa, eli joko yksittäinen elementaarinen mittaus tuottaa tulokseksi 0:n tai 1:n. Mitään välimuotoa ei ole, eikä voi olla, ensinnäkin, koska pätee "tertium non datur" ja toisekseen, koska todellisuus on kvantittunut. Ja jos taas tarkastellaan yksittäisten mittaustulosten joukkoa, niin silloinkin kukin yksittäinen; eli *todellinen* mittaustuloksen arvo voi olla vain joko 0 tai 1, eikä mikään välimuoto. Jos taas mittaus tuottaa eri mittauskerroilla eri arvon, vaikka oletetaan mitattavan samaa kohdetta, niin se johtuu siitä, että mittausta ei olla suoritettu samalla ajanhetkellä tai mitataan eri kohdetta.

        Voi tietenkin silti olla niin, että emme *tiedä* sitä, että mikä jokin mittaustulos on tai, että tiedämme sen vain osittain, esim. koska sitä kuvaava informaatio on hajautunut ja käytettävissämme on siitä vain osa, mutta silti kukin tuollainen mittaustulos voi *tiedettynä* olla arvoltaan vain 0 tai 1. Tietenkin, toisaalta, jos laskemme yksittäisten mittaustulosten perusteella muita kompleksisempia asioita kuvaavia arvoja, niin ne voivat olla muutakin kuin 0 tai 1, mutta ei sekään tietenkään tarkoita sitä, että todellisuus muuttuisi maagisesti epämääräiseksi, vaan vain sitä, että kuvattava todellisuuden osa on tuossa tapauksessa kompleksisempi ja sitä kuvaavaa informaatiota on käytettävissä enemmän kuin yksi bitti.

        "Vastakkaista eli staattista tilannetta ei ole ikinä missään havaittu eikä sellaisen olemassaolosta ole yhtään viitettä."

        Eihän dynaamisuuden edellytys siis tietenkään ole epätarkkuus, vaan epätarkkuus päinvastoin tekisi dynaamisuuden mahdottomaksi, koska epätarkat asiat eivät voi olla todellisia. Dynaamisuudessa on vain kyse siitä, että todellisuuden osien väliset suhteet; tai ainakin niitä vastaaviksi oletetut havainnot, ovat eri ajanhetkillä erilaisia, mikä taas on tietenkin aivan luonnollista, dynaamisesti käyttäytyvän, eli itsensä kanssa vuorovaikuttavan, todellisuuden tapauksessa.

        Jos mittaustulos on epätarkka, niin vika on mittalaitteessa, vääränlaisen mittalaitteen käytössä kyseiseen tarkoitukseen, väärästä mittaustulosten tulkintatavasta tms. Koskaan ei olla havaittu mitään *reaalisten mittausten kohteiden* epätarkkuutta, eikä edes elementaaristen mittaustulostenkaan epätarkkuutta, koska kukin niistä on arvoltaan juuri se, mikä se sitten onkin, vaikka ei tietenkään välttämättä kuvaakaan oikeellisesti sitä kohdetta, jota mittauksen oletetaan kuvaavan, eli esim. voidaan olettaa mitattavan samaa kohdetta eri mittauskerroilla, vaikka oikeasti mitataankin eri kohdetta ja tietenkin myös, jos mitattava yksinkertaiseksi oletettu kohde onkin oikeasti kompleksinen, niin on tietenkin täysin mahdollista, että siihen kohdistuvat mittaukset tuottavat eri mittauskerroilla erilaisia arvoja, vaikka kohteessa itsessään ei olekaan mitään epämääräistä...

        Nyt tuli ihan täydellistä puuta heinää, jos kirjoittajan oli tarkoitus kuvata kvanttifysikaalista todellisuutta. Einstein yritti vielä vängätä piilomuuttujahypoteesillaan, ettei Heisenbergin epätarkkuusperiaate kuvaisikaan todellista todellisuutta, mutta hänen todistettiin olleen väärässä.

        Kvanttitodellisuus on pysyvästi erilainen kuin arkijärellä sitä voisi kuvata.


      • Anonyymi
        Anonyymi kirjoitti:

        "Eihän kvanttimekaanisissa, eikä missään muissakaan systeemeissä koskaan *oikeasti* voi olla mitään epätarkkuutta..."

        Tarkka ajan ja paikan hetki edellyttää havainnon kohteena olevan prosessin pysäyttämistä ja vain tarkalla ajan (kesto=0) ja paikan hetkellä jokin voi olla tarkasti määriteltyä.

        Voiko luonnon prosesseja edes sillä tavalla pysäyttää mielivaltaisesti? Minusta ei voi.

        Koska mittauksia ei voi suorittaa niin että niiden kesto olisi nolla eikä todellisuudessa ole muutenkaan sellaisia "pysäytyskuvia" eli tarkasti määriteltävissä olevia objekteja kuten esim. luonnollisessa kielessä (jossa oletetaan kaikki fysikaaliset objektit jotenkin staattisesti olemassaolevina) niin se epämääräisyys on lievennettävissä ainoastaan ehkä niin että tieteen objekteja aletaan tutkia nimenomaan prosessisysteemeinä eikä staattisina esineen kaltaisina objekteina eli ts. otetaan huomioon vain kokonaiset prosessit ja niiden takaisinkytkennät jonkinlaisena holistisena systeeminä ja samalla luovutaan reduktionismista.

        Atomit, molekyylit, eliöt jne. ovat siis ne ainoat konkreettiset tutkittavissa olevat prosessit kun taas ns. alkeishiukkaset ovat pelkkiä abstraktioita joita ei voi erottaa niistä prosesseista joiden osia ne ovat ilman että se konkreettisuus samalla katoaa eli ts. atomit, molekyylit, biologiset oliot lajinsa jäseninä ovat pienimpiä mahdollisia prosesseja joilla on tietyt yksiselitteiset ominaisuudet joita niiden osilla ei ole .

        Kaikki konkreettiset oliot ja niiden joukot ovat prosesseja eli niillä on ajallinen kesto ja usein monenlaisia (takaisinkykentä) luuppeja eli syklejä eikä yhtä sen syklin osaa voi tarkastella eikä empiirisesti mitata irrallaan siitä kokonaissykilistä joka taas koostuu lukuisista sisäkkäisistä ja/tai rinnakkaisista sykleistä.

        2-arvoinen (kyllä/ei) logiikka pätee binäärisissä systeemeissä ja vaikka ns. kvanttimaailma ilmeisesti onkin jollain tavalla epäjatkuva ja epälineaarinen niin siitä ei mitenkään välttämättä seuraa että se olisi myös digitaalinen tai 2-arvoisesti binäärinen.

        Ei se superdeterminismi muutu pelkästään sillä tavalla todelliseksi että joitakuita eksaktisuus ja matemaattinen tarkkuus & ehdoton determinismi suuresti miellyttää ja vastaavasti kaikenlainen epätarkkuus, tilastollinen ontologia ja valinnan vapaus eivät taas miellytä :D


        B

        Nimimerkki: Professori Realisti (XPR-0001)

        B: "Tarkka ajan ja paikan hetki edellyttää havainnon kohteena olevan prosessin pysäyttämistä ja vain tarkalla ajan (kesto=0) ja paikan hetkellä jokin voi olla tarkasti määriteltyä. Voiko luonnon prosesseja edes sillä tavalla pysäyttää mielivaltaisesti? Minusta ei voi."

        [Osa 1/2] Olen samaa mieltä siitä, että luonnon prosesseja ei voida pysäyttää, eli mittaukset antavat vain tavallaan pysäytyskuvia todellisuudesta, eli pysäytys- *siinä merkityksessä*, että vähänkään laajemman prosessin omainaisuuksia ei mitenkään saada mitattua tarkasti; etenkään yksittäisellä mittauksella ja tietenkin mittauksia rajoittaa myös se, että todellisuus ei salli suorittaakaan kuin vain tietynlaiset mittaukset, mikä on asia, jota tieteentekijät eivät ymmärrettävistä syistä yleensä halua korostaa...

        "2-arvoinen (kyllä/ei) logiikka pätee binäärisissä systeemeissä ja vaikka ns. kvanttimaailma ilmeisesti onkin jollain tavalla epäjatkuva ja epälineaarinen niin siitä ei mitenkään välttämättä seuraa että se olisi myös digitaalinen tai 2-arvoisesti binäärinen."

        Kvantti-ilmiöt eivät ole niin outoja, kuin miltä ne ehkä voivat joistakin vaikuttaa, koska pohjimmiltaan niissä on kyse vain kvanttimekaanisten aaltojen keskinäisestä interferenssistä. Tietenkin, jos noita interferoivia aaltoja on paljon; liittyen tiettyyn mitattavaan kohteeseen, niin tilanne voi olla erittäin *kompleksinen*, mutta ei siinä silti oikeasti mitään epämääräistä ole. Toisaalta, esim. kvanttilogiikkapiireillä voidaan toteuttaa kaikki Boolean funktiot ja toisaalta joidenkin kvanttilogiikkapiirien tulostotuustaulut voivat tosiaan sisältää myös muita arvoja kuin 0:n ja 1:n, mutta tuossa on kyse siitä, että noiden interferoivien aaltojen nettovaikutus on; noiden piirien osalta sellainen, joka vaihtelee ajan funktiona ja noudattaa isomman prosessointimäärän osalta keskimäärin tietynlaista todennäköisyysjakaumaa. Kantani on se, että tuo on mahdollista vain juuri sen ansiosta, että tarkasteltavan järjestelmän *konkreettinen kompleksisuus* on riittävä ja jos/kun se on riittävä, niin tuollaiset todennäköisyysjakaumat ovat selitettävissä nimenomaan ja vain konkreettisten järjestelmien vuorovaikutuksena.
        https://en.wikipedia.org/wiki/Quantum_logic_gate

        "Atomit, molekyylit, eliöt jne. ovat siis ne ainoat konkreettiset tutkittavissa olevat prosessit kun taas ns. alkeishiukkaset ovat pelkkiä abstraktioita joita ei voi erottaa niistä prosesseista joiden osia ne ovat ilman että se konkreettisuus samalla katoaa eli ts. atomit, molekyylit, biologiset oliot lajinsa jäseninä ovat pienimpiä mahdollisia prosesseja joilla on tietyt yksiselitteiset ominaisuudet joita niiden osilla ei ole."

        Voi olla, että emme ole useimmista noista ja aiemmin mainitsemistasi asioista eri mieltä, joskaan en ole varma siitä, että ymmärrämmekö nuo asiat aivan samalla tavalla. Tuon alkeishiukkasten yms. "ei-konkreettisuuden" osalta tilanne kuitenkin on se, että jos jokin asia on edellytys jollekin todellisella vaikutukselle, eli sellaiselle, joka on edellytys todellisuuden tilan muutokselle, niin silloin se ei voi olla abstraktio.

        Koska siis esim. kvanttitietokoneissa alkeishiukkaset; tietyssä kohta selitettävässä merkityksessä, vuorovaikuttavat keskenään ja tuottavat sellaisia laskennallisia tuloksia, joita keskimäärin syntyisi ilman tuollaista laitetta vain hyvin tai äärimmäisen epätodennäköisesti, niin sellainen oletus, että nuo alkeishiukkaset eivät *tietyssä merkityksessä* olisi konkreettisia, ei ole uskottava. Tosin, tietenkin nekin ovat todellisia siinä merkityksessä *vain abstraktiona*, että itse asiassa vain niiden osat tai "ei-lokaaliset vastineet", jotka vuorovaikuttavat keskenään kvanttimekaanisesti, eli juuri noiden kvanttimekaanisten aaltojen interferenssin kautta, vaikuttavat konkreettisilta, eli siis ovat eräs keskeinen edellytys noiden tulosten tuottamiselle.


      • Anonyymi
        Anonyymi kirjoitti:

        "Eihän kvanttimekaanisissa, eikä missään muissakaan systeemeissä koskaan *oikeasti* voi olla mitään epätarkkuutta..."

        Tarkka ajan ja paikan hetki edellyttää havainnon kohteena olevan prosessin pysäyttämistä ja vain tarkalla ajan (kesto=0) ja paikan hetkellä jokin voi olla tarkasti määriteltyä.

        Voiko luonnon prosesseja edes sillä tavalla pysäyttää mielivaltaisesti? Minusta ei voi.

        Koska mittauksia ei voi suorittaa niin että niiden kesto olisi nolla eikä todellisuudessa ole muutenkaan sellaisia "pysäytyskuvia" eli tarkasti määriteltävissä olevia objekteja kuten esim. luonnollisessa kielessä (jossa oletetaan kaikki fysikaaliset objektit jotenkin staattisesti olemassaolevina) niin se epämääräisyys on lievennettävissä ainoastaan ehkä niin että tieteen objekteja aletaan tutkia nimenomaan prosessisysteemeinä eikä staattisina esineen kaltaisina objekteina eli ts. otetaan huomioon vain kokonaiset prosessit ja niiden takaisinkytkennät jonkinlaisena holistisena systeeminä ja samalla luovutaan reduktionismista.

        Atomit, molekyylit, eliöt jne. ovat siis ne ainoat konkreettiset tutkittavissa olevat prosessit kun taas ns. alkeishiukkaset ovat pelkkiä abstraktioita joita ei voi erottaa niistä prosesseista joiden osia ne ovat ilman että se konkreettisuus samalla katoaa eli ts. atomit, molekyylit, biologiset oliot lajinsa jäseninä ovat pienimpiä mahdollisia prosesseja joilla on tietyt yksiselitteiset ominaisuudet joita niiden osilla ei ole .

        Kaikki konkreettiset oliot ja niiden joukot ovat prosesseja eli niillä on ajallinen kesto ja usein monenlaisia (takaisinkykentä) luuppeja eli syklejä eikä yhtä sen syklin osaa voi tarkastella eikä empiirisesti mitata irrallaan siitä kokonaissykilistä joka taas koostuu lukuisista sisäkkäisistä ja/tai rinnakkaisista sykleistä.

        2-arvoinen (kyllä/ei) logiikka pätee binäärisissä systeemeissä ja vaikka ns. kvanttimaailma ilmeisesti onkin jollain tavalla epäjatkuva ja epälineaarinen niin siitä ei mitenkään välttämättä seuraa että se olisi myös digitaalinen tai 2-arvoisesti binäärinen.

        Ei se superdeterminismi muutu pelkästään sillä tavalla todelliseksi että joitakuita eksaktisuus ja matemaattinen tarkkuus & ehdoton determinismi suuresti miellyttää ja vastaavasti kaikenlainen epätarkkuus, tilastollinen ontologia ja valinnan vapaus eivät taas miellytä :D


        B

        Nimimerkki: Professori Realisti (XPR-0002)

        B: "Ei se superdeterminismi muutu pelkästään sillä tavalla todelliseksi että joitakuita eksaktisuus ja matemaattinen tarkkuus & ehdoton determinismi suuresti miellyttää ja vastaavasti kaikenlainen epätarkkuus, tilastollinen ontologia ja valinnan vapaus eivät taas miellytä :D"

        [Osa 2/2] Superdeterminismi on looginen seuraus siitä, että todellisuus ei voi olla epämääräinen. Miksikö todellisuus ei voi olla epämääräinen? Koska epämääräisellä asialla ei voi olla mitään *vaikutuksia*, koska jokainen todellinen vaikutus on väistämättä tietynlainen ja *jokainen* mittaustulos todentaa tämän ja jos/kun epämääräiseksi oletettu asia tuottaa nimenomaan aina tietynlaisen vaikutuksen, niin se ei voi olla epämääräinen... Asia aivan erikseen on se, että voidaanko kaikki todellisia asioita *siltikään* mallintaa tarkasti ja matemaattisesti ja tuolta osinhan kantani on se, että ei voida, joskaan se ei tietenkään tarkoita sitä, etteikö täsmällisyys olisi yleisessä katsannossa hyve kommunikaation tarkoitetun kaltaisen onnistumisen eräänä edellytyksenä. Mielestäni ongelma näiden asioiden ymmärtämisessä on monilla siinä, että sekoitetaan epistemologinen epävarmuus ja ontologinen epämääräisyys keskenään, vaikka en ovat kaksi aivan eri asiaa.

        Mitä taas tuohon "valinnan vapauteen" tulee, niin oleellistahan ei ole se, että miellyttääkö se vai ei, vaan se, että siinä ei ole mitään järkeä, se ei ole mitenkään mahdollista, eikä siitä olisi mitään hyötyä, vaikka se voisikin olla mahdollista. Ihmisillä on riittävästi sekä ongelmia, että kykyjä ilmankin, että väitetään, että heillä olisi sellaisia kykyjä, joita heillä ei mitenkään voi olla ja tuo "valinnan vapaus" on tyyppiesimerkki noista mahdottomista kyvyistä ja itseasiassa keskittyminen tuollaisiin kuviteltuihin kykyihin vähentää mahdollisuuksia keskittyä todellisiin kykyihin... Tämänkin asian osalta luonnollisen kielen käyttö on todennäköinen syy suureen osaan hämmennyksestä. Eli, *todellinen* valinnan vapaus on täysin mahdoton asia, mutta se mitä "valinnan vapaudella" usein tarkoitetaan onkin sitä, että esim. tietty henkilö ei tiedosta tietyllä hetkellä olevansa osa todellisuutta ja kuvittelee esim. voivansa painaa äänestysnappulaa sekä siten, että tuloksena olisi sekä "Jaa" että "Ei". Kuitenkaan tuollaista ei koskaan ole tapahtunut, eikä mitenkään voi tapahtua... ja vaikka käytössä olisi kolmas painike "Ehkä", niin silloinkin kukin todellinen äänestystulos on aina *jokin* noista, eikä niiden yhdistelmä... Tietenkin lisäongelmana tässä myös on se, että poliittisille päättäjille, markkinavoimille jne. on tyypillistä pyrkimys manipuloida ihmisiä siten, että nämä *kuvittelisivat tekevänsä päätöksiään itsenäisesti*, vaikka oikeasti he eivät siis mitenkään voikaan tehdä niitä itsenäisesti...

        Kirjoituksesi vaikuttaa hyvin paljon nimimerkin Belisario tyyliseltä ja kysynkin, että voitko varmentaa, että olet Belisario? Tuolla asialla ei tietenkään ole mitään vaikutusta esitettävien argumenttien pätevyyteen, mutta olisi hyödyllistä tietää, että kenen kanssa keskustelee, koska esim. minä ja Belisario olemme keskustelleet aiemminkin jo niin paljon, että tietyt asiat voidaan olettaa tiedetyiksi ja siten hiukan oikaista selitysten määrässä. Luonnollisella kielellä tyypillisen höperösti ilmaisten, "jos olet Belisario, niin keskustelusta voi muodostua laadukas", mutta tietenkin *oikeasti* keskustelun laatutaso voi olla vain se, mikä se sitten onkin, eli tuokin näennäinen epämääräisyys on luonteeltaan vain epistemologista epävarmuutta. :D


      • Anonyymi
        Anonyymi kirjoitti:

        Nyt tuli ihan täydellistä puuta heinää, jos kirjoittajan oli tarkoitus kuvata kvanttifysikaalista todellisuutta. Einstein yritti vielä vängätä piilomuuttujahypoteesillaan, ettei Heisenbergin epätarkkuusperiaate kuvaisikaan todellista todellisuutta, mutta hänen todistettiin olleen väärässä.

        Kvanttitodellisuus on pysyvästi erilainen kuin arkijärellä sitä voisi kuvata.

        Nimimerkki: Professori Realisti (XPR-0003)

        Anonyymi: "Einstein yritti vielä vängätä piilomuuttujahypoteesillaan, ettei Heisenbergin epätarkkuusperiaate kuvaisikaan todellista todellisuutta, mutta hänen todistettiin olleen väärässä."

        Einsteinin aikana (1879-1955) ei vielä oltu kehitetty kvanttimekaniikan monimaailmatulkintaa (1957-), joka on siististi täysin deterministinen...
        https://en.wikipedia.org/wiki/Many-worlds_interpretation


      • Anonyymi
        Anonyymi kirjoitti:

        Nimimerkki: Professori Realisti (XPR-0003)

        Anonyymi: "Einstein yritti vielä vängätä piilomuuttujahypoteesillaan, ettei Heisenbergin epätarkkuusperiaate kuvaisikaan todellista todellisuutta, mutta hänen todistettiin olleen väärässä."

        Einsteinin aikana (1879-1955) ei vielä oltu kehitetty kvanttimekaniikan monimaailmatulkintaa (1957-), joka on siististi täysin deterministinen...
        https://en.wikipedia.org/wiki/Many-worlds_interpretation

        Mwi -tulkinnassa mittaustulos määräytyy täysin satunnaisesti. Esim. yksittäinen elektroni kaksoisrakokokeessa, mwi:n mukaan elektronin paikka vastakkaisella filmillä on puhtaan satunnainen.


      • Anonyymi
        Anonyymi kirjoitti:

        Nimimerkki: Professori Realisti (XPR-0001)

        B: "Tarkka ajan ja paikan hetki edellyttää havainnon kohteena olevan prosessin pysäyttämistä ja vain tarkalla ajan (kesto=0) ja paikan hetkellä jokin voi olla tarkasti määriteltyä. Voiko luonnon prosesseja edes sillä tavalla pysäyttää mielivaltaisesti? Minusta ei voi."

        [Osa 1/2] Olen samaa mieltä siitä, että luonnon prosesseja ei voida pysäyttää, eli mittaukset antavat vain tavallaan pysäytyskuvia todellisuudesta, eli pysäytys- *siinä merkityksessä*, että vähänkään laajemman prosessin omainaisuuksia ei mitenkään saada mitattua tarkasti; etenkään yksittäisellä mittauksella ja tietenkin mittauksia rajoittaa myös se, että todellisuus ei salli suorittaakaan kuin vain tietynlaiset mittaukset, mikä on asia, jota tieteentekijät eivät ymmärrettävistä syistä yleensä halua korostaa...

        "2-arvoinen (kyllä/ei) logiikka pätee binäärisissä systeemeissä ja vaikka ns. kvanttimaailma ilmeisesti onkin jollain tavalla epäjatkuva ja epälineaarinen niin siitä ei mitenkään välttämättä seuraa että se olisi myös digitaalinen tai 2-arvoisesti binäärinen."

        Kvantti-ilmiöt eivät ole niin outoja, kuin miltä ne ehkä voivat joistakin vaikuttaa, koska pohjimmiltaan niissä on kyse vain kvanttimekaanisten aaltojen keskinäisestä interferenssistä. Tietenkin, jos noita interferoivia aaltoja on paljon; liittyen tiettyyn mitattavaan kohteeseen, niin tilanne voi olla erittäin *kompleksinen*, mutta ei siinä silti oikeasti mitään epämääräistä ole. Toisaalta, esim. kvanttilogiikkapiireillä voidaan toteuttaa kaikki Boolean funktiot ja toisaalta joidenkin kvanttilogiikkapiirien tulostotuustaulut voivat tosiaan sisältää myös muita arvoja kuin 0:n ja 1:n, mutta tuossa on kyse siitä, että noiden interferoivien aaltojen nettovaikutus on; noiden piirien osalta sellainen, joka vaihtelee ajan funktiona ja noudattaa isomman prosessointimäärän osalta keskimäärin tietynlaista todennäköisyysjakaumaa. Kantani on se, että tuo on mahdollista vain juuri sen ansiosta, että tarkasteltavan järjestelmän *konkreettinen kompleksisuus* on riittävä ja jos/kun se on riittävä, niin tuollaiset todennäköisyysjakaumat ovat selitettävissä nimenomaan ja vain konkreettisten järjestelmien vuorovaikutuksena.
        https://en.wikipedia.org/wiki/Quantum_logic_gate

        "Atomit, molekyylit, eliöt jne. ovat siis ne ainoat konkreettiset tutkittavissa olevat prosessit kun taas ns. alkeishiukkaset ovat pelkkiä abstraktioita joita ei voi erottaa niistä prosesseista joiden osia ne ovat ilman että se konkreettisuus samalla katoaa eli ts. atomit, molekyylit, biologiset oliot lajinsa jäseninä ovat pienimpiä mahdollisia prosesseja joilla on tietyt yksiselitteiset ominaisuudet joita niiden osilla ei ole."

        Voi olla, että emme ole useimmista noista ja aiemmin mainitsemistasi asioista eri mieltä, joskaan en ole varma siitä, että ymmärrämmekö nuo asiat aivan samalla tavalla. Tuon alkeishiukkasten yms. "ei-konkreettisuuden" osalta tilanne kuitenkin on se, että jos jokin asia on edellytys jollekin todellisella vaikutukselle, eli sellaiselle, joka on edellytys todellisuuden tilan muutokselle, niin silloin se ei voi olla abstraktio.

        Koska siis esim. kvanttitietokoneissa alkeishiukkaset; tietyssä kohta selitettävässä merkityksessä, vuorovaikuttavat keskenään ja tuottavat sellaisia laskennallisia tuloksia, joita keskimäärin syntyisi ilman tuollaista laitetta vain hyvin tai äärimmäisen epätodennäköisesti, niin sellainen oletus, että nuo alkeishiukkaset eivät *tietyssä merkityksessä* olisi konkreettisia, ei ole uskottava. Tosin, tietenkin nekin ovat todellisia siinä merkityksessä *vain abstraktiona*, että itse asiassa vain niiden osat tai "ei-lokaaliset vastineet", jotka vuorovaikuttavat keskenään kvanttimekaanisesti, eli juuri noiden kvanttimekaanisten aaltojen interferenssin kautta, vaikuttavat konkreettisilta, eli siis ovat eräs keskeinen edellytys noiden tulosten tuottamiselle.

        "[Osa 1/2] Olen samaa mieltä siitä, että luonnon prosesseja ei voida pysäyttää...."

        Tarkoitin lähinnä sitä että koska se tutkittava kohde on aina jatkuvan muutoksen tilassa varsinkin atomitason mittakaavassa niin se "pysäytyskuva" ei ole koskaan selkeärajainen vaan lähinnä melko sumea vähän samaan tyyliin kun valokuvattaessa makromaailman nopeasti liikkuvaa kohdetta eli esim. atomin sisällä elektronit näkyvät jonkinlaisena sumuna eivätkä jonkinlaisina selkeästi erottuvina pieninä "biljardipalloina".

        Se tarkkuus, binäärisyys ja määräytyneisyys sekä myös jossain mielessä se determinismi seuraa lähinnä matematiikan ja logiikan käytöstä eikä sitä välttämättä ainakaan samassa muodossa ole edes löydettävissä luonnosta. Helposti sotketaan ja liitetään käytetyn kielen sisäänrakennettuja ominaisuuksia ja oletuksia siihen havaintoon ja liitetään tutkittavaan kohteeseen sellaisia ominaisuuksia joita siinä ei välttämättä ole. Sitten helposti päädytäänkin tutkimaan oikeasti sitä käytettyä kieltä ja syntyy helposti kuvitelmia että se tutkittava kohde on sen kielen ominaisuuksien mukainen tai jopa rakentunut siitä kielestä (kuten Max Tegmarkin matematiikkaontologiassa)

        "Tuon alkeishiukkasten yms. "ei-konkreettisuuden" osalta ... "

        Nuo alkeishiukkaset voidaan minusta käsittää myös matemaattisen mallin (hiukkasfysiikan standardimalli) parametreina ja empiiriseltä kannalta tilastollisia poikkeamina joilla ei välttämättä ole edes mitään merkittävää ontologiaa sen hiukkaskiihdyttimen ulkopuolella vaikka niille on annettukin nimet ja ne joissain grafiikoissa esitetään pieninä biljardipalloina. Kun se fysiikan taustaoletus on reduktionismi niin tietysti uskotaan että kaikki todellisuus koostuu niistä ja määräytyy niiden mukaan. Makromaailman kannalta ne kuitenkin ovat ehkä yhtä epäolennaisia kuin kuvaruudun yksittäiset pikselit.

        B(elisario)


      • Anonyymi
        Anonyymi kirjoitti:

        Nimimerkki: Professori Realisti (XPR-0002)

        B: "Ei se superdeterminismi muutu pelkästään sillä tavalla todelliseksi että joitakuita eksaktisuus ja matemaattinen tarkkuus & ehdoton determinismi suuresti miellyttää ja vastaavasti kaikenlainen epätarkkuus, tilastollinen ontologia ja valinnan vapaus eivät taas miellytä :D"

        [Osa 2/2] Superdeterminismi on looginen seuraus siitä, että todellisuus ei voi olla epämääräinen. Miksikö todellisuus ei voi olla epämääräinen? Koska epämääräisellä asialla ei voi olla mitään *vaikutuksia*, koska jokainen todellinen vaikutus on väistämättä tietynlainen ja *jokainen* mittaustulos todentaa tämän ja jos/kun epämääräiseksi oletettu asia tuottaa nimenomaan aina tietynlaisen vaikutuksen, niin se ei voi olla epämääräinen... Asia aivan erikseen on se, että voidaanko kaikki todellisia asioita *siltikään* mallintaa tarkasti ja matemaattisesti ja tuolta osinhan kantani on se, että ei voida, joskaan se ei tietenkään tarkoita sitä, etteikö täsmällisyys olisi yleisessä katsannossa hyve kommunikaation tarkoitetun kaltaisen onnistumisen eräänä edellytyksenä. Mielestäni ongelma näiden asioiden ymmärtämisessä on monilla siinä, että sekoitetaan epistemologinen epävarmuus ja ontologinen epämääräisyys keskenään, vaikka en ovat kaksi aivan eri asiaa.

        Mitä taas tuohon "valinnan vapauteen" tulee, niin oleellistahan ei ole se, että miellyttääkö se vai ei, vaan se, että siinä ei ole mitään järkeä, se ei ole mitenkään mahdollista, eikä siitä olisi mitään hyötyä, vaikka se voisikin olla mahdollista. Ihmisillä on riittävästi sekä ongelmia, että kykyjä ilmankin, että väitetään, että heillä olisi sellaisia kykyjä, joita heillä ei mitenkään voi olla ja tuo "valinnan vapaus" on tyyppiesimerkki noista mahdottomista kyvyistä ja itseasiassa keskittyminen tuollaisiin kuviteltuihin kykyihin vähentää mahdollisuuksia keskittyä todellisiin kykyihin... Tämänkin asian osalta luonnollisen kielen käyttö on todennäköinen syy suureen osaan hämmennyksestä. Eli, *todellinen* valinnan vapaus on täysin mahdoton asia, mutta se mitä "valinnan vapaudella" usein tarkoitetaan onkin sitä, että esim. tietty henkilö ei tiedosta tietyllä hetkellä olevansa osa todellisuutta ja kuvittelee esim. voivansa painaa äänestysnappulaa sekä siten, että tuloksena olisi sekä "Jaa" että "Ei". Kuitenkaan tuollaista ei koskaan ole tapahtunut, eikä mitenkään voi tapahtua... ja vaikka käytössä olisi kolmas painike "Ehkä", niin silloinkin kukin todellinen äänestystulos on aina *jokin* noista, eikä niiden yhdistelmä... Tietenkin lisäongelmana tässä myös on se, että poliittisille päättäjille, markkinavoimille jne. on tyypillistä pyrkimys manipuloida ihmisiä siten, että nämä *kuvittelisivat tekevänsä päätöksiään itsenäisesti*, vaikka oikeasti he eivät siis mitenkään voikaan tehdä niitä itsenäisesti...

        Kirjoituksesi vaikuttaa hyvin paljon nimimerkin Belisario tyyliseltä ja kysynkin, että voitko varmentaa, että olet Belisario? Tuolla asialla ei tietenkään ole mitään vaikutusta esitettävien argumenttien pätevyyteen, mutta olisi hyödyllistä tietää, että kenen kanssa keskustelee, koska esim. minä ja Belisario olemme keskustelleet aiemminkin jo niin paljon, että tietyt asiat voidaan olettaa tiedetyiksi ja siten hiukan oikaista selitysten määrässä. Luonnollisella kielellä tyypillisen höperösti ilmaisten, "jos olet Belisario, niin keskustelusta voi muodostua laadukas", mutta tietenkin *oikeasti* keskustelun laatutaso voi olla vain se, mikä se sitten onkin, eli tuokin näennäinen epämääräisyys on luonteeltaan vain epistemologista epävarmuutta. :D

        "Koska epämääräisellä asialla ei voi olla mitään *vaikutuksia*, koska jokainen todellinen vaikutus on väistämättä tietynlainen ja *jokainen* mittaustulos todentaa tämän ja jos/kun epämääräiseksi oletettu asia tuottaa nimenomaan aina tietynlaisen vaikutuksen, niin se ei voi olla epämääräinen"

        Kyllä minusta systeemi voi olla esim. määräytymättömässä tilassa potentiaalisuutensa takia ja siksi että vain osa sen systeemin ominaisuuksista on aktualisoitunut kunakin hetkenä. Tietysti jos kielletään todennäköisyysjakauman olemassaolo ja oletetaan että kaikki mahdollisuudet ovat aina täysin deterministisesti aktualisoituneita kukin omassa multiversumin maailmassaan niin tietenkin se väistämätön johtopäätös sen tyyppisistä oletuksista on se superdeterminismi.

        "Mitä taas tuohon "valinnan vapauteen" tulee, niin oleellistahan ei ole se, että miellyttääkö se vai ei, vaan se, että siinä ei ole mitään järkeä, se ei ole mitenkään mahdollista, eikä siitä olisi mitään hyötyä, vaikka se voisikin olla mahdollista."
        Kyllä esim. kokeentekijällä pitää olla jonkinlainen valinnanvapaus ja jos ei ole niin ei se tiedekään olisi edes mahdollista eikä mielekästä.

        "että esim. tietty henkilö ei tiedosta tietyllä hetkellä olevansa osa todellisuutta ja kuvittelee esim. voivansa painaa äänestysnappulaa sekä siten, että tuloksena olisi sekä "Jaa" että "Ei". "

        Heh. Esim. eduskunnassa voi jäädä pois äänestyksestä eli ei valitse kumpaakaan ja se on todennäköisesti hyvin usein täysin harkittu valinta. Joissain maissa kansalaisilla on äänestyspakko sakon uhalla jolloin voi sitten piirellä sinne äänestyslappuun vaikka Aku Ankan kuvan jos mikään tarjolla olevista ehdokkaista ei ole mieleen. Sekin on valinta.

        Vaikka se valitsija onkin osa todellisuutta niin tietoiset valitsijat ovat kuitenkin siinä määrin autonomisia että pystyvät tekemään suhteellisen itsenäisiä ja mielekkäitä ratkaisuja.

        Tätä tahdonvapaushumppaa olemme veivanneet aikaisemmin jo kyllästymiseen asti joten erimielisyys tulee varmaan edelleenkin säilymään sen ja varmaan myös sen superdeterminismin suhteen jonka takia joudut kieltämään samalla sen valinnanmahdollisuuden.

        B(elisario)ja jatkossa pelkkä B niin säästyy vähän vaivaa.... :D


      • Anonyymi
        Anonyymi kirjoitti:

        "[Osa 1/2] Olen samaa mieltä siitä, että luonnon prosesseja ei voida pysäyttää...."

        Tarkoitin lähinnä sitä että koska se tutkittava kohde on aina jatkuvan muutoksen tilassa varsinkin atomitason mittakaavassa niin se "pysäytyskuva" ei ole koskaan selkeärajainen vaan lähinnä melko sumea vähän samaan tyyliin kun valokuvattaessa makromaailman nopeasti liikkuvaa kohdetta eli esim. atomin sisällä elektronit näkyvät jonkinlaisena sumuna eivätkä jonkinlaisina selkeästi erottuvina pieninä "biljardipalloina".

        Se tarkkuus, binäärisyys ja määräytyneisyys sekä myös jossain mielessä se determinismi seuraa lähinnä matematiikan ja logiikan käytöstä eikä sitä välttämättä ainakaan samassa muodossa ole edes löydettävissä luonnosta. Helposti sotketaan ja liitetään käytetyn kielen sisäänrakennettuja ominaisuuksia ja oletuksia siihen havaintoon ja liitetään tutkittavaan kohteeseen sellaisia ominaisuuksia joita siinä ei välttämättä ole. Sitten helposti päädytäänkin tutkimaan oikeasti sitä käytettyä kieltä ja syntyy helposti kuvitelmia että se tutkittava kohde on sen kielen ominaisuuksien mukainen tai jopa rakentunut siitä kielestä (kuten Max Tegmarkin matematiikkaontologiassa)

        "Tuon alkeishiukkasten yms. "ei-konkreettisuuden" osalta ... "

        Nuo alkeishiukkaset voidaan minusta käsittää myös matemaattisen mallin (hiukkasfysiikan standardimalli) parametreina ja empiiriseltä kannalta tilastollisia poikkeamina joilla ei välttämättä ole edes mitään merkittävää ontologiaa sen hiukkaskiihdyttimen ulkopuolella vaikka niille on annettukin nimet ja ne joissain grafiikoissa esitetään pieninä biljardipalloina. Kun se fysiikan taustaoletus on reduktionismi niin tietysti uskotaan että kaikki todellisuus koostuu niistä ja määräytyy niiden mukaan. Makromaailman kannalta ne kuitenkin ovat ehkä yhtä epäolennaisia kuin kuvaruudun yksittäiset pikselit.

        B(elisario)

        "Tarkoitin lähinnä sitä että koska se tutkittava kohde on aina jatkuvan muutoksen tilassa varsinkin atomitason mittakaavassa niin se "pysäytyskuva" ei ole koskaan selkeärajainen vaan lähinnä melko sumea vähän samaan tyyliin kun valokuvattaessa makromaailman nopeasti liikkuvaa kohdetta "

        Kvanttimekaniikassa ei edes ole parametria t (paitsi ensimmäisten peruskurssien yhtälöissä) vaan aikaoperaattori, jolla hiukkasen ajanhetken jakauma saadaan samaan tapaan kuin paikan jakauma ja muut fysikaaliset suureet.
        Jos otetaan pysytyskuva jonkin tietyn hiukkasen aikajanalta, niin kaikilla muilla hiukkaislla on epätarkat aikajakaumat ja paikkajakaumat, ja ne ovat eri todennäköisyyksillä eri paikoissa ja ajoissa. Eli vaikka tietyn hiukkasen ajanhetki tunnettaisiin täysin tarkasti, se tekee kokonaiskuvasta kaikkea muuta kuin tarkan.


      • Anonyymi
        Anonyymi kirjoitti:

        Nimimerkki: Professori Realisti (XPR-0003)

        Anonyymi: "Einstein yritti vielä vängätä piilomuuttujahypoteesillaan, ettei Heisenbergin epätarkkuusperiaate kuvaisikaan todellista todellisuutta, mutta hänen todistettiin olleen väärässä."

        Einsteinin aikana (1879-1955) ei vielä oltu kehitetty kvanttimekaniikan monimaailmatulkintaa (1957-), joka on siististi täysin deterministinen...
        https://en.wikipedia.org/wiki/Many-worlds_interpretation

        Ja monimaailmatulkintako mahdollistaa determinismin? Oikeasti, voiko olla näin lapsellisen naiiveja ihmisiä, että pitää monimaailmatulkintaa mitenkään uskottavana?


      • Anonyymi
        Anonyymi kirjoitti:

        Ja monimaailmatulkintako mahdollistaa determinismin? Oikeasti, voiko olla näin lapsellisen naiiveja ihmisiä, että pitää monimaailmatulkintaa mitenkään uskottavana?

        Nimimerkki: Professori Realisti (XPR-0004)

        Anonyymi: "Ja monimaailmatulkintako mahdollistaa determinismin? Oikeasti, voiko olla näin lapsellisen naiiveja ihmisiä, että pitää monimaailmatulkintaa mitenkään uskottavana?"

        Tietenkään *tulkinta* ei mahdollista determinismiä, vaan *todellisuus*, joka on aivan oikein *kuvattu* tuossa MWI-tulkinnassa ja kyllä MWI on täysin deterministinen... Argumentointisi menee nyt lapsellisen "ad hominem":in puolelle, koska on hyvin tunnettu tosiasia, että MWI selittää *empiiriset havainnot* *aivan yhtä hyvin* kuin kilpailevat kvanttimekaaniset tulkinnatkin ja lisäksi *ilman*, että joudutaan turvautumaan epätieteelliseen ja maagiseen epämääräisyyteen, jota esim. Einstein ei siis aikoinaan hyväksynyt...

        Ei-deterministiset tulkinnat voi poissulkea jo sillä perusteella, että ontologisesti epämääräisillä ja siten epätodellisilla asioilla ei voi olla reaalisia vaikutuksia; kuten tuolla aiemmin selitin. Epämääräisyydet yms. ovat vain abstraktioiden, eivätkä todellisten fysikaalisten asioiden ominaisuuksia. Loogisesti "potentiaalisesti todellisten tapahtumien" ja "todellisten tapahtumien" joukot ovat myös väistämättä yhtenevät, mikä tarkoittaa sitä, että kaikkien mahdollisten tapahtumien täytyy myös aktualisoitua; aivan kuten MWI:n mukaan tapahtuukin...

        Itseasiassa ei-deterministiset tulkinnat vaikuttavat myös occamilaisittain kilpailukyvyttömiltä ja käytännön tasolla epätieteellisiltä umpikujilta, joskin on tietenkin totta, että MWI:n falsifikaatiokriteerin kehittäminen on erittäin haasteellista. Ei-determinismiä ei kuitenkaan voida mitenkään pitää uskottavana; kunhan ensin edes ymmärretään, että mitä se *oikeasti* tarkoittaisi...

        Sillä taas, että esim. kuinka suosittu tietty teoria tai tulkinta on, ei ole merkitystä sen *paikkansapitävyyden* kannalta. Olettaisin, että noin 25% nyky-fyysikoista pitää MWI:tä oikeana tulkintana. MWI ei ilmeisesti ole suosittu lähinnä sen vuoksi, koska se ei ole suosittu, eli fyysikotkin kavahtavat vähemmistöön kuulumista ja akateemisesti on myös paljon helpompaa valita "perässäjuoksijoiden" rooli, joka on muutenkin aina osa akateemista kulttuuria. Olettaisin, että etenkin huippufyysikoista yli 25% pitää nykyään MWI:ta parhaana tulkintana, mutta ei em. syiden vuoksi lähde sanomaan sitä julkisesti, koska heillä ei sitä kautta olisi oikeastaan akateemisesti paljonkaan voitettavaa ja toisaalta olisi paljon hävittävää em. syistä.

        Esim. Albert Einstein ei siis hyväksynyt ei-determinismiä ja uskalsi sen sanoa, koska hänen asemansa oli aikoinaan niin vahva kuin se oli, mutta Bell:in teoreema pätee ja hänen aikanaan ei ollut keinoja ratkaista tuota dilemmaa. Moderneista fyysikoista esim. David Deutsch pitää monimaailmoja laskennallisilla perusteilla täysin reaalisina, Stephen Hawking piti MWI:ta paikkansapitävänä, Andrei Linde on kehittänyt ikuisen kosmisen inflaation mallin liittyen säieteoriaan ja Max Tegmark uskoo jopa siihenkin, että kaikki matemaattiset asiat ovat olemassa myös fyysisesti...

        Lähinnä lapsellista on se, että niitä Bell:in teoreeman "porsaanreikiä" on yritetty hyödyntää niinkin vähän kuin on, eli esim. *täysin perusteeton* ja *järjenvastainen* oletus tutkijan valinnanvapaudesta; mm. tieteellisiin kokeisiin liittyen, on nielty sen kummemmin kakistelematta ja edes asiaa kyseenalaistamatta; Gerard t'Hooft:ia ja joitakin muita harvoja edistyksellisiä lukuunottamatta, joskaan t'Hooft:kaan ei ole niin edistyksellinen, että olisi yhdistänyt superdeterminismin monimaailmatulkintaan...
        https://en.wikipedia.org/wiki/Loopholes_in_Bell_test_experiments


      • Anonyymi
        Anonyymi kirjoitti:

        "[Osa 1/2] Olen samaa mieltä siitä, että luonnon prosesseja ei voida pysäyttää...."

        Tarkoitin lähinnä sitä että koska se tutkittava kohde on aina jatkuvan muutoksen tilassa varsinkin atomitason mittakaavassa niin se "pysäytyskuva" ei ole koskaan selkeärajainen vaan lähinnä melko sumea vähän samaan tyyliin kun valokuvattaessa makromaailman nopeasti liikkuvaa kohdetta eli esim. atomin sisällä elektronit näkyvät jonkinlaisena sumuna eivätkä jonkinlaisina selkeästi erottuvina pieninä "biljardipalloina".

        Se tarkkuus, binäärisyys ja määräytyneisyys sekä myös jossain mielessä se determinismi seuraa lähinnä matematiikan ja logiikan käytöstä eikä sitä välttämättä ainakaan samassa muodossa ole edes löydettävissä luonnosta. Helposti sotketaan ja liitetään käytetyn kielen sisäänrakennettuja ominaisuuksia ja oletuksia siihen havaintoon ja liitetään tutkittavaan kohteeseen sellaisia ominaisuuksia joita siinä ei välttämättä ole. Sitten helposti päädytäänkin tutkimaan oikeasti sitä käytettyä kieltä ja syntyy helposti kuvitelmia että se tutkittava kohde on sen kielen ominaisuuksien mukainen tai jopa rakentunut siitä kielestä (kuten Max Tegmarkin matematiikkaontologiassa)

        "Tuon alkeishiukkasten yms. "ei-konkreettisuuden" osalta ... "

        Nuo alkeishiukkaset voidaan minusta käsittää myös matemaattisen mallin (hiukkasfysiikan standardimalli) parametreina ja empiiriseltä kannalta tilastollisia poikkeamina joilla ei välttämättä ole edes mitään merkittävää ontologiaa sen hiukkaskiihdyttimen ulkopuolella vaikka niille on annettukin nimet ja ne joissain grafiikoissa esitetään pieninä biljardipalloina. Kun se fysiikan taustaoletus on reduktionismi niin tietysti uskotaan että kaikki todellisuus koostuu niistä ja määräytyy niiden mukaan. Makromaailman kannalta ne kuitenkin ovat ehkä yhtä epäolennaisia kuin kuvaruudun yksittäiset pikselit.

        B(elisario)

        Nimimerkki: Professori Realisti (XPR-0005)

        Belisario: "Tarkoitin lähinnä sitä että koska se tutkittava kohde on aina jatkuvan muutoksen tilassa varsinkin atomitason mittakaavassa niin se "pysäytyskuva" ei ole koskaan selkeärajainen vaan lähinnä melko sumea vähän samaan tyyliin kun valokuvattaessa makromaailman nopeasti liikkuvaa kohdetta eli esim. atomin sisällä elektronit näkyvät jonkinlaisena sumuna eivätkä jonkinlaisina selkeästi erottuvina pieninä "biljardipalloina"."

        Esim. yksittäistä elektronia *vastaavaksi tulkittavien havaintojen*; osaltahan pätee se, että noita havaintoja täytyy tehdä runsaasti, jotta tuo "sumu" alkaa hahmottua. Eli, se ei hahmotu *yksittäisen* "pysäytyskuvan" perusteella, jossa elektronilla *näyttää* olevan tietty sijainti tiettynä ajan hetkenä; mittaustarkkuuden/mahdollisuuksien puitteissa. Tuo havainto ei siis tosiaan kuitenkaan kuvaa todellista tilannetta, liittyen epämääräisyysperiaatteeseen, eli siis johtuen "rajallisesta mahdollisesta mittaustarkkuudesta".

        Tuo rajoitteellisuus taas ei tässä tarkoita sitä, että tuo mittaustarkkuus olisi pelkästään teknisistä syistä rajoittunutta, vaan se ilmeisesti on rajoittunut myös todellisuuden osien keskinäisiin korrelaatioihin liittyen, eli siis niiden keskinäisestä vuorovaikutuksesta johtuen... Jos ollaan vielä vähän tarkempia, niin tuo em. "yksittäisen elektronin havaintotapahtumakin" liittyy aina sen ns. lokaaliin ympäristöön, eli siis sitä lähellä oleviksi *tulkittaviin* hiukkasiin ja tietenkin myös niiden järjestelmien tiloihin, jotka *mahdollistavat* nuo nimenomaiset havainnot ja niitä kuvaavien tietojen käsittelyn.

        Oletan, että tuon "hiukkas-aalto -dualismin" syy liittyy siihen, että esim. tarkkailtavalla kohteella itsellään on sellainen todellinen kompleksisuusaste, joka omalta osaltaan mahdollistaa tuon epämääräisyyden vaikutelman syntymisen. Eli, elektronit eivät tietyssä merkityksessä ole "yksinkertaisia biljardipalloja", vaan; abstraktisti ilmaisten, sellaisia osistansa koostuvia "aaltoja", joiden aaltojen rakenneosista suurin osa on mittausten ulottumattomissa, mikä taas on täysin odotusten mukainen tilanne, jos/kun todellisuuden osien ominaisuuksien arvot korreloivat väistämättä keskenään.

        Koska pistemäisillä kohteilla ei ylipäätään voi olla fysikaalisia ominaisuuksia, niin selitysten kannalta ehkä elegantein keino päästä eteenpäin tässä dilemmassa on se, että noiden "mittaustarkkuuden rajoilla" olevien havaintojen kohteiden *tulkitaan* olevan todellisuuden osien välisiä vuorovaikutuksia.

        MWI:hin toisaalta päädytään mm. siitä syystä, että tuo havaintojen kompleksisuuden selittämiseen tarvittava reaalinen kompleksisuus, ei ole uskottavasti aikaansaatavissa ns. paikallisesti, koska se edellyttää noiden vuorovaikutusten seurausten havaittavuutta ja siten ilmeisesti erittäin suuren määrän sellaisia fysikaalisia rakenteita, jotka mahdollistavat informaation tallentumisen, mikä taas ei liene mahdollista, jos noilla rakenteilla ei ole nollaa suurempi lepomassa, mikä taas ei ole mahdollista, jos tuo massa sijaitsisi paikallisesti sellaisella etäisyydellä havaitsijasta, jolla nuo rakenteet ylipäätään olisivat havaittavissa *silloin* kun havainnot tehdään.

        "Se tarkkuus, binäärisyys ja määräytyneisyys sekä myös jossain mielessä se determinismi seuraa lähinnä matematiikan ja logiikan käytöstä eikä sitä välttämättä ainakaan samassa muodossa ole edes löydettävissä luonnosta."

        On ilmeinen virhepäätelmä olettaa, että tosiaan väistämättä aina rajallisesta todellisuuden ominaisuuksien arvojen havainto/mittaus -kyvystä, seuraisi se, että *todellisuus itsessään* ei silti olisi yksikäsitteinen ja yksikäsitteisesti käyttäytyvä. Kantani on se, että "epäyksikäsitteisesti" käyttäytyvä todellisuus on toisaalta looginen mahdottomuus ja tuollaisen olettaminen noiden em. havaintojen perusteella toisaalta on siis non sequitur.

        "Helposti sotketaan ja liitetään käytetyn kielen sisäänrakennettuja ominaisuuksia ja oletuksia siihen havaintoon ja liitetään tutkittavaan kohteeseen sellaisia ominaisuuksia joita siinä ei välttämättä ole."

        Aivan ja koska voimme siis loogisesti päätellä, että *todelliset* asiat eivät voi olla epämääräisiä; vaikka niitä koskevat havainnot voidaankin *esittää* stokastisesti, niin todellisuuden "leimaaminen epämääräiseksi", on juuri esimerkki tuollaisesta "kielellisen mopon käsistä karkaamisesta johtuvasta" seurauksesta... :D


      • Anonyymi
        Anonyymi kirjoitti:

        "[Osa 1/2] Olen samaa mieltä siitä, että luonnon prosesseja ei voida pysäyttää...."

        Tarkoitin lähinnä sitä että koska se tutkittava kohde on aina jatkuvan muutoksen tilassa varsinkin atomitason mittakaavassa niin se "pysäytyskuva" ei ole koskaan selkeärajainen vaan lähinnä melko sumea vähän samaan tyyliin kun valokuvattaessa makromaailman nopeasti liikkuvaa kohdetta eli esim. atomin sisällä elektronit näkyvät jonkinlaisena sumuna eivätkä jonkinlaisina selkeästi erottuvina pieninä "biljardipalloina".

        Se tarkkuus, binäärisyys ja määräytyneisyys sekä myös jossain mielessä se determinismi seuraa lähinnä matematiikan ja logiikan käytöstä eikä sitä välttämättä ainakaan samassa muodossa ole edes löydettävissä luonnosta. Helposti sotketaan ja liitetään käytetyn kielen sisäänrakennettuja ominaisuuksia ja oletuksia siihen havaintoon ja liitetään tutkittavaan kohteeseen sellaisia ominaisuuksia joita siinä ei välttämättä ole. Sitten helposti päädytäänkin tutkimaan oikeasti sitä käytettyä kieltä ja syntyy helposti kuvitelmia että se tutkittava kohde on sen kielen ominaisuuksien mukainen tai jopa rakentunut siitä kielestä (kuten Max Tegmarkin matematiikkaontologiassa)

        "Tuon alkeishiukkasten yms. "ei-konkreettisuuden" osalta ... "

        Nuo alkeishiukkaset voidaan minusta käsittää myös matemaattisen mallin (hiukkasfysiikan standardimalli) parametreina ja empiiriseltä kannalta tilastollisia poikkeamina joilla ei välttämättä ole edes mitään merkittävää ontologiaa sen hiukkaskiihdyttimen ulkopuolella vaikka niille on annettukin nimet ja ne joissain grafiikoissa esitetään pieninä biljardipalloina. Kun se fysiikan taustaoletus on reduktionismi niin tietysti uskotaan että kaikki todellisuus koostuu niistä ja määräytyy niiden mukaan. Makromaailman kannalta ne kuitenkin ovat ehkä yhtä epäolennaisia kuin kuvaruudun yksittäiset pikselit.

        B(elisario)

        Nimimerkki: Professori Realisti (XPR-0006)

        Belisario: "Sitten helposti päädytäänkin tutkimaan oikeasti sitä käytettyä kieltä ja syntyy helposti kuvitelmia että se tutkittava kohde on sen kielen ominaisuuksien mukainen tai jopa rakentunut siitä kielestä (kuten Max Tegmarkin matematiikkaontologiassa)"

        Matemaattiset asiat toisaalta eivät voi sisältää informaatiota, eivätkä siis voi olla fyysisiä, vaan matemaattisten asioiden olemassaolo edellyttää fysikaalisia rakenteita ja matemaattiset asiat ovat siis *fysikaalisten rakenteiden tulkintoja*.

        Tegmark ilmeisesti olettaa, että nuo kaikki fysikaaliset edellytykset ovat olemassa; kuten niiden varmaan täytyykin olla olemassa, mutta mielestäni on siis selvä non sequitur olettaa, että nuo fysikaaliset edellytykset olisivat matemaattisia, eli kyse on tosiaan vain siiitä, että fysikaalinen todellisuus *mahdollistaa* sen, että sitä voidaan *kuvata* matemaattisesti, joskaan en siis usko, että sitä käytännössä kyetään edes kuvaamaankaan *täydellisesti*; tai edes läheskään täydellisestikään, matemaattisesti, eikä vähemmän täsmällisten esitystapojen käyttökään tässä lisää kuvausten oikeellisuutta, mutta tuo kuvaamiskyvyn rajoitteisuus ei siis missään tapauksessa tarkoita sitä, että todellisuus *itsessään* olisi ontologisesti epämääräinen... Matemaattisen kuvausten hyödyllisyys liittyy käsiteltävyyteen.

        "Nuo alkeishiukkaset voidaan minusta käsittää myös matemaattisen mallin (hiukkasfysiikan standardimalli) parametreina ja empiiriseltä kannalta tilastollisia poikkeamina joilla ei välttämättä ole edes mitään merkittävää ontologiaa sen hiukkaskiihdyttimen ulkopuolella vaikka niille on annettukin nimet ja ne joissain grafiikoissa esitetään pieninä biljardipalloina. Kun se fysiikan taustaoletus on reduktionismi niin tietysti uskotaan että kaikki todellisuus koostuu niistä ja määräytyy niiden mukaan. Makromaailman kannalta ne kuitenkin ovat ehkä yhtä epäolennaisia kuin kuvaruudun yksittäiset pikselit."

        Tämä on aika mutkikas asia. Eli, reduktionismin tavoiteltavuuden perussyy liittyy *käsiteltävyyteen*, eli jotta todellisuuden malli voisi olla jossakin merkityksessä hyödyllinen, niin sen *täytyy* olla käsiteltävissä oleva ja koska ilmeisesti todellisuus itsessään kuitenkaan *ei* ole käsiteltävissä oleva, niin todellisuuden mallit tosiaan ovat aina abstraktioita, mutta ilmeisesti kuitenkin jotkin niistä ovat silti joidenkin todellisuuden aspektien osalta riittävän hyviä, jotta niitä voidaan käyttää esim. todellisuuden käyttäytymisen ennustamiseen käytännön tavoitteiden kannalta aivan riittävän luotettavasti; tietyillä rajoitteilla.

        Eli, esim. jos tavoitteena on tuottaa sellainen laskennallinen laite; kvanttitietokone, jolla noiden em. mallien käsiteltävyytttä voitaisiin parantaa, niin on hyödyllistä mallintaa todellisuuden käyttäytymistä siten, että ajatellaan, että tuo laskenta kuvataan; abstraktilla tasolla, johtuvaksi alkeishiukkasten vuorovaikutuksesta, vaikka tuo vuorovaikutus ei tietenkään "tapahdukaan tyhjiössä", vaan koko todellisuus tosiaan välillisesti *kontribuoi* tuohon laskentaan ja on itse asiassa myös sen *edellytyskin*, koska kuitenkin näyttää siltä, että tuollainenkin abstraktio on riittävä, jotta se tuottaa kontribuutiota mm. tuollaisen laitteen konstruoinnin mahdollistamisen kannalta...


      • Anonyymi
        Anonyymi kirjoitti:

        "Koska epämääräisellä asialla ei voi olla mitään *vaikutuksia*, koska jokainen todellinen vaikutus on väistämättä tietynlainen ja *jokainen* mittaustulos todentaa tämän ja jos/kun epämääräiseksi oletettu asia tuottaa nimenomaan aina tietynlaisen vaikutuksen, niin se ei voi olla epämääräinen"

        Kyllä minusta systeemi voi olla esim. määräytymättömässä tilassa potentiaalisuutensa takia ja siksi että vain osa sen systeemin ominaisuuksista on aktualisoitunut kunakin hetkenä. Tietysti jos kielletään todennäköisyysjakauman olemassaolo ja oletetaan että kaikki mahdollisuudet ovat aina täysin deterministisesti aktualisoituneita kukin omassa multiversumin maailmassaan niin tietenkin se väistämätön johtopäätös sen tyyppisistä oletuksista on se superdeterminismi.

        "Mitä taas tuohon "valinnan vapauteen" tulee, niin oleellistahan ei ole se, että miellyttääkö se vai ei, vaan se, että siinä ei ole mitään järkeä, se ei ole mitenkään mahdollista, eikä siitä olisi mitään hyötyä, vaikka se voisikin olla mahdollista."
        Kyllä esim. kokeentekijällä pitää olla jonkinlainen valinnanvapaus ja jos ei ole niin ei se tiedekään olisi edes mahdollista eikä mielekästä.

        "että esim. tietty henkilö ei tiedosta tietyllä hetkellä olevansa osa todellisuutta ja kuvittelee esim. voivansa painaa äänestysnappulaa sekä siten, että tuloksena olisi sekä "Jaa" että "Ei". "

        Heh. Esim. eduskunnassa voi jäädä pois äänestyksestä eli ei valitse kumpaakaan ja se on todennäköisesti hyvin usein täysin harkittu valinta. Joissain maissa kansalaisilla on äänestyspakko sakon uhalla jolloin voi sitten piirellä sinne äänestyslappuun vaikka Aku Ankan kuvan jos mikään tarjolla olevista ehdokkaista ei ole mieleen. Sekin on valinta.

        Vaikka se valitsija onkin osa todellisuutta niin tietoiset valitsijat ovat kuitenkin siinä määrin autonomisia että pystyvät tekemään suhteellisen itsenäisiä ja mielekkäitä ratkaisuja.

        Tätä tahdonvapaushumppaa olemme veivanneet aikaisemmin jo kyllästymiseen asti joten erimielisyys tulee varmaan edelleenkin säilymään sen ja varmaan myös sen superdeterminismin suhteen jonka takia joudut kieltämään samalla sen valinnanmahdollisuuden.

        B(elisario)ja jatkossa pelkkä B niin säästyy vähän vaivaa.... :D

        Nimimerkki: Professori Realisti (XPR-0007)

        Belisario: "Kyllä minusta systeemi voi olla esim. määräytymättömässä tilassa potentiaalisuutensa takia ja siksi että vain osa sen systeemin ominaisuuksista on aktualisoitunut kunakin hetkenä."

        Tuo liittyy semantiikkaan ja epistemologiaan, eli tuollainen "potentiaalisuus" on mahdollista *siinä merkityksessä*, että jos ko. systeemillä on *sisäistä reaalista rakennetta*, niin tuon rakenteen tilan voidaan tulkita olevan eräs niistä edellytyksistä, jotka mahdollistavat havaitunkaltaisen systeemin käyttäytymisen.

        Eli, tuo "potentiaalisuus" on vain abstrakti ilmaus, eikä todellista potentiaalisuutta taatusti voi olla olemassa ilman reaalisia sen mahdollistavia tekijöitä. Tarkoitan siis sitä, että vaikka todellisiin vaikutuksiin liittyvät havainnot tosiaan vaihtelevatkin, niin *kukin todellinen vaikutus* on silti väistämättä olemassa nimenomaan juuri tietynlaisena, eikä itsensä kumoavana olemattomuutena...

        "Tietysti jos kielletään todennäköisyysjakauman olemassaolo ja oletetaan että kaikki mahdollisuudet ovat aina täysin deterministisesti aktualisoituneita kukin omassa multiversumin maailmassaan niin tietenkin se väistämätön johtopäätös sen tyyppisistä oletuksista on se superdeterminismi."

        Todennäköisyysjakaumahan on olemassa *havaintojen kuvauksena*. Kantani on se, että *kaikki* todelliset mahdollisuudet ja "potentiaalit" tosiaan väistämättä aktualisoituvat. Superdeterminismiin päädytään myös esim. *sitäkin* kautta, että mitkään yksittäiset havainnot eivät siis koskaan ole oikeasti epämääräisiä. Eli, nuo kvantti-ilmiöiden näennäiset epämääräisyydetkin ovat vain yksittäisistä havainnoista koostuvien havaintosarjojen *tulkintoja*.

        "Kyllä esim. kokeentekijällä pitää olla jonkinlainen valinnanvapaus ja jos ei ole niin ei se tiedekään olisi edes mahdollista eikä mielekästä."

        Kokeentekijällä ei tietenkään ole minkäänlaista todellista valinnanvapautta ja *silti* tiede tuottaa tuloksia. Ehkä haluat perustella tarkemmin noita käsittämättömiä oletuksiasi, niin perustelen sitten tarkemmin noita omia johtopäätöksiäni.

        "Heh. Esim. eduskunnassa voi jäädä pois äänestyksestä eli ei valitse kumpaakaan ja se on todennäköisesti hyvin usein täysin harkittu valinta. Joissain maissa kansalaisilla on äänestyspakko sakon uhalla jolloin voi sitten piirellä sinne äänestyslappuun vaikka Aku Ankan kuvan jos mikään tarjolla olevista ehdokkaista ei ole mieleen. Sekin on valinta."

        Oleellistahan tässä ei ole *näennäisesti tarjolla olevien vaihtoehtojen lukumäärä*, vaan se, että ne kaikki eivät havaintojen mukaan voi toteutua *siinä kontekstissa*, joissa niitä havainnoidaan. Eli, MWI:n mukaan ne kaikki tosiaan toteutuvat, mutta ne *eivät* voi toteutua tässä universumissa, vaan ne toteutuvat niissä muissa universumeissa; tai vaihtoehtoisesti tämän universumin *äärimmäisen* kaukaisissa osissa...

        "Tätä tahdonvapaushumppaa olemme veivanneet aikaisemmin jo kyllästymiseen asti joten erimielisyys tulee varmaan edelleenkin säilymään sen ja varmaan myös sen superdeterminismin suhteen jonka takia joudut kieltämään samalla sen valinnanmahdollisuuden."

        Tuo "kieltämisenikinhän" on siinä merkityksessä vain näennäistä, että kaikki "valinnat" tosiaan ovat pakotettuja, koska vain pakottavat syyt voivat saada aikaan todellisia vaikutuksia. Tuon "valinta-teeman" osalta debattimme antina voisi ehkä ideaalitilanteessa olla ainakin se, että kumpikin saisi muotoiltua teesinsä ja niiden taustaoletukset mahdollisimman selkeiksi ja yksikäsitteisiksi.

        "B(elisario)ja jatkossa pelkkä B niin säästyy vähän vaivaa.... :D"

        Hyvä, että olet mukana keskusteluissa, sillä se selvästi korreloi positiivisesti niiden keskimääräisen laadun kanssa. Hyvin lyhyisiin tunnisteisiin liittyy se tunnettu ongelma, että jos haluaa esim. tsekata, että mitä olet kirjoittanut viime aikoina, niin tuo "B" tuottaa myös ns. "false positive" -tapauksia... (esim. Brexit...), joskin ymmärrän kyllä tietysti toisaalta myös sen, että tietynlaisesta "esitysteknisestä epämääräisyydestä" on mm. se hyöty, että on vähemmän todennäköistä, että kommenttien sisältö muotoutuu kirjoittajaan liittyvien ennakkokäsitysten perusteella.


      • Anonyymi
        Anonyymi kirjoitti:

        Nimimerkki: Professori Realisti (XPR-0005)

        Belisario: "Tarkoitin lähinnä sitä että koska se tutkittava kohde on aina jatkuvan muutoksen tilassa varsinkin atomitason mittakaavassa niin se "pysäytyskuva" ei ole koskaan selkeärajainen vaan lähinnä melko sumea vähän samaan tyyliin kun valokuvattaessa makromaailman nopeasti liikkuvaa kohdetta eli esim. atomin sisällä elektronit näkyvät jonkinlaisena sumuna eivätkä jonkinlaisina selkeästi erottuvina pieninä "biljardipalloina"."

        Esim. yksittäistä elektronia *vastaavaksi tulkittavien havaintojen*; osaltahan pätee se, että noita havaintoja täytyy tehdä runsaasti, jotta tuo "sumu" alkaa hahmottua. Eli, se ei hahmotu *yksittäisen* "pysäytyskuvan" perusteella, jossa elektronilla *näyttää* olevan tietty sijainti tiettynä ajan hetkenä; mittaustarkkuuden/mahdollisuuksien puitteissa. Tuo havainto ei siis tosiaan kuitenkaan kuvaa todellista tilannetta, liittyen epämääräisyysperiaatteeseen, eli siis johtuen "rajallisesta mahdollisesta mittaustarkkuudesta".

        Tuo rajoitteellisuus taas ei tässä tarkoita sitä, että tuo mittaustarkkuus olisi pelkästään teknisistä syistä rajoittunutta, vaan se ilmeisesti on rajoittunut myös todellisuuden osien keskinäisiin korrelaatioihin liittyen, eli siis niiden keskinäisestä vuorovaikutuksesta johtuen... Jos ollaan vielä vähän tarkempia, niin tuo em. "yksittäisen elektronin havaintotapahtumakin" liittyy aina sen ns. lokaaliin ympäristöön, eli siis sitä lähellä oleviksi *tulkittaviin* hiukkasiin ja tietenkin myös niiden järjestelmien tiloihin, jotka *mahdollistavat* nuo nimenomaiset havainnot ja niitä kuvaavien tietojen käsittelyn.

        Oletan, että tuon "hiukkas-aalto -dualismin" syy liittyy siihen, että esim. tarkkailtavalla kohteella itsellään on sellainen todellinen kompleksisuusaste, joka omalta osaltaan mahdollistaa tuon epämääräisyyden vaikutelman syntymisen. Eli, elektronit eivät tietyssä merkityksessä ole "yksinkertaisia biljardipalloja", vaan; abstraktisti ilmaisten, sellaisia osistansa koostuvia "aaltoja", joiden aaltojen rakenneosista suurin osa on mittausten ulottumattomissa, mikä taas on täysin odotusten mukainen tilanne, jos/kun todellisuuden osien ominaisuuksien arvot korreloivat väistämättä keskenään.

        Koska pistemäisillä kohteilla ei ylipäätään voi olla fysikaalisia ominaisuuksia, niin selitysten kannalta ehkä elegantein keino päästä eteenpäin tässä dilemmassa on se, että noiden "mittaustarkkuuden rajoilla" olevien havaintojen kohteiden *tulkitaan* olevan todellisuuden osien välisiä vuorovaikutuksia.

        MWI:hin toisaalta päädytään mm. siitä syystä, että tuo havaintojen kompleksisuuden selittämiseen tarvittava reaalinen kompleksisuus, ei ole uskottavasti aikaansaatavissa ns. paikallisesti, koska se edellyttää noiden vuorovaikutusten seurausten havaittavuutta ja siten ilmeisesti erittäin suuren määrän sellaisia fysikaalisia rakenteita, jotka mahdollistavat informaation tallentumisen, mikä taas ei liene mahdollista, jos noilla rakenteilla ei ole nollaa suurempi lepomassa, mikä taas ei ole mahdollista, jos tuo massa sijaitsisi paikallisesti sellaisella etäisyydellä havaitsijasta, jolla nuo rakenteet ylipäätään olisivat havaittavissa *silloin* kun havainnot tehdään.

        "Se tarkkuus, binäärisyys ja määräytyneisyys sekä myös jossain mielessä se determinismi seuraa lähinnä matematiikan ja logiikan käytöstä eikä sitä välttämättä ainakaan samassa muodossa ole edes löydettävissä luonnosta."

        On ilmeinen virhepäätelmä olettaa, että tosiaan väistämättä aina rajallisesta todellisuuden ominaisuuksien arvojen havainto/mittaus -kyvystä, seuraisi se, että *todellisuus itsessään* ei silti olisi yksikäsitteinen ja yksikäsitteisesti käyttäytyvä. Kantani on se, että "epäyksikäsitteisesti" käyttäytyvä todellisuus on toisaalta looginen mahdottomuus ja tuollaisen olettaminen noiden em. havaintojen perusteella toisaalta on siis non sequitur.

        "Helposti sotketaan ja liitetään käytetyn kielen sisäänrakennettuja ominaisuuksia ja oletuksia siihen havaintoon ja liitetään tutkittavaan kohteeseen sellaisia ominaisuuksia joita siinä ei välttämättä ole."

        Aivan ja koska voimme siis loogisesti päätellä, että *todelliset* asiat eivät voi olla epämääräisiä; vaikka niitä koskevat havainnot voidaankin *esittää* stokastisesti, niin todellisuuden "leimaaminen epämääräiseksi", on juuri esimerkki tuollaisesta "kielellisen mopon käsistä karkaamisesta johtuvasta" seurauksesta... :D

        XPR-0005:

        "Esim. yksittäistä elektronia *vastaavaksi tulkittavien havaintojen*; osaltahan pätee se.... "

        Nyt sitten joudutaan siirtymään keskustelemaan asioista joiden ilmaiseminen ja ymmärtäminen tarkoitetussa merkityksessä on huomattavasti hankalampaa koska ne edellyttävät ainakin jossain määrin yhteismitallista todellisuuden hahmotusta keskustelijoiden välillä ja sekä hyvin poikkitieteellistä että erityisesti poikkiontologista analyysiä jossa mukaan tulee melkein kaikki kuviteltavissa olevat ns. filosofiset tai metafyysiset erimielisyydet mitä erilaisten ontologioiden välillä keskusteluissa voi esiintyä.

        Keskustelu väkisinkin sitten rönsyilee mutta varmaankin silti pysyttelee ketjun otsikon aiheessa joka on "ihanteellisen" väljä tällaiseen.

        Minä käytin tuota ilmaisua "pysäytyskuva" lähinnä enemmän vertauskuvallisessa tai analogisessa merkityksessä kuin jonkin konkreettisen yksittäisen mittaustapahtuman merkityksessä.

        Minä lähden siltä kannalta että esim. se mitä kutsumme mieleksi, tietoisuudeksi tai vielä tarkemmin ilmaistuna ainutlaatuiseksi ja yksilölliseksi persoonallisuudeksi on aina paljon enemmän kuin "osiensa summa" eli esim. ihmisyksilön tapauksessa vaikka fysiikan kannalta ihmisen kehon elimien atomit ja molekyylit jatkuvasti vaihtuvat koko fyysisen elämän ajan niin se itse yksilö tai persoonallisuus pysyy kuitenkin samana vähän samaan tyyliin kuin sama tietokoneohjelmisto erilaisissa tietokoneissa on edelleenkin se sama ohjelmisto (sama logiikka, samat toiminnot) vaikka niiden fysikaalinen ilmentyminen ja laitteisto voi olla erilainen ja moneessa laitteistossa samaan aikaan.

        Tästä tietokoneen ja ohjelmiston vertauskuvasta ei tietenkään voi eikä kannata olettaa sitä että esim. ihminen olisi tietokone ja tietoisuus toimintoineen vain ohjelmisto vaan kyseessä on vain loogis-analoginen päättely jossa tunnetun ja pohjimmiltaan tuntemattoman järjestelmän havaituista rakenne- ja toimintayhtäläisyyksistä voidaan siirtyä tuntemattoman systeemin tutkimiseen niin että samalla informaatiota sekä katoaa (koska ne systeemit eivät ole identtisiä) että lisääntyy (koska ne yhteiset ominaisuudet tarjoavat uuden rajapinnan tai uusia käsitteellisiä työkaluja mahdolliselle jatkotutkimuksille).

        Todellisuuden ontologian "valikoimaan" ainakin minun mielestäni kaikkein olennaisimpina osina kuuluvat tällaiset holistiset systeemit jotka eivät rakennu näkyvistä ja mitattavista komponenteistaan (esim. alkeishiukkaset,atomit & molekyylit) "ylöspäin" konkreettisiksi havainto-objekteiksi (konkreettinen eliöyksilö kuten tietty ihminen, koira, kissa ,puu, huonekasvi, kivi, järvi. vuori, planeetta,aurinko,galaksi jne yms.) jotka ovat jotenkin yksilöllisiä ja ainutlaatuisia ja minusta määrittelevät niiden osiensa käyttäytymistä joilla taas ole mitään yksilöllisyyttä (kuten ne molekyylit, atomit & alkeishiukkaset).

        Ne yksittäiset atomit, molekyylit & alkeishiukkaset kokonaisuudestaan erotettuna voivat minusta ainakin periaatteessa olla juuri sellaisia kuin ne ovat yksiselitteisesti riippumatta havaintojen puutteellisuudesta mutta jonkun laajemman yksilöllisen systeemin osina eivät kuitenkaan määrittele sen kokonaissysteemin toimintaa eli esim. minun pikkusormeni kynnen molekyyli tai atomi ei millään tavalla määrittele eikä determinoi minun toimintaani eikä myöskään niiden atomien ja molekyylien kokonaisuus sitä välttämättä tee koska siinä mitä kutsun pikkusormekseni vaihtuu fysikaalisen ainesosat jatkuvasti ja vuosikymmenien kuluttua siinä ei välttämättä ole enää ainuttakaan samaa atomia kuin oli esim. syntymähetkelläni ja samaa koskee kaikkia muitakin kehoni elimiä.

        "koska voimme siis loogisesti päätellä, että *todelliset* asiat eivät voi olla epämääräisiä; ...:D"

        Voisi ehkä ottaa myös huomioon sellainen mahdollisuus että kaikkien olemassaolevien asioiden "todellisuus" tai "konkreettisuus" ei ole aina samantasoista vaan ikäänkuin hierarkkinen ontologia joissa sen ontologisen hierarkian "ylätasot" (ns. "yksilölliset" oliot) determinoivat ja organisoivat niitä alemman ontologisen tason ("epäyksilölliset" atomit, alkeishiukkaset & molekyylit) ainetta jolloin se alataso voi olla tietyssä mielessä deterministinen ja tarkoin määriteltävissä eli "aktuaalinen" kun taas se "ylempi" taso aistihavaintojen kannalta näyttäytyy aina jotenkin epämääräisenä ja potentiaalisena.

        Ontologisen hierarkian ylätaso= implisiittinen järjestys joka määrittelee alemman tason säännönmukaisuudet

        Ontologisen hierakian alataso= eksplisiittinen järjestys joka käyttäytyy pääsääntöisesti sen ylemmän tason määrittelmien "luonnonlakien" mukaan.

        Minua jostain syystä miellyttävät ja kiinnostavat kaikenlaiset avoimet ja vähän epämääräisemmät systeemit ja ideat joista voi kehittyä jotain uutta ja mielenkiintoista juuri nimenomaan sen takia että ne eivät ole etukäteen lukkoon lyötyjä vaan ainakin osittain ominaisuuksiltaan vielä tuntemattomia ja edellyttävät ehkä uusia analogioita.

        Belisario

        (jatkuu ehkä huomenna....)


      • Anonyymi
        Anonyymi kirjoitti:

        Nimimerkki: Professori Realisti (XPR-0006)

        Belisario: "Sitten helposti päädytäänkin tutkimaan oikeasti sitä käytettyä kieltä ja syntyy helposti kuvitelmia että se tutkittava kohde on sen kielen ominaisuuksien mukainen tai jopa rakentunut siitä kielestä (kuten Max Tegmarkin matematiikkaontologiassa)"

        Matemaattiset asiat toisaalta eivät voi sisältää informaatiota, eivätkä siis voi olla fyysisiä, vaan matemaattisten asioiden olemassaolo edellyttää fysikaalisia rakenteita ja matemaattiset asiat ovat siis *fysikaalisten rakenteiden tulkintoja*.

        Tegmark ilmeisesti olettaa, että nuo kaikki fysikaaliset edellytykset ovat olemassa; kuten niiden varmaan täytyykin olla olemassa, mutta mielestäni on siis selvä non sequitur olettaa, että nuo fysikaaliset edellytykset olisivat matemaattisia, eli kyse on tosiaan vain siiitä, että fysikaalinen todellisuus *mahdollistaa* sen, että sitä voidaan *kuvata* matemaattisesti, joskaan en siis usko, että sitä käytännössä kyetään edes kuvaamaankaan *täydellisesti*; tai edes läheskään täydellisestikään, matemaattisesti, eikä vähemmän täsmällisten esitystapojen käyttökään tässä lisää kuvausten oikeellisuutta, mutta tuo kuvaamiskyvyn rajoitteisuus ei siis missään tapauksessa tarkoita sitä, että todellisuus *itsessään* olisi ontologisesti epämääräinen... Matemaattisen kuvausten hyödyllisyys liittyy käsiteltävyyteen.

        "Nuo alkeishiukkaset voidaan minusta käsittää myös matemaattisen mallin (hiukkasfysiikan standardimalli) parametreina ja empiiriseltä kannalta tilastollisia poikkeamina joilla ei välttämättä ole edes mitään merkittävää ontologiaa sen hiukkaskiihdyttimen ulkopuolella vaikka niille on annettukin nimet ja ne joissain grafiikoissa esitetään pieninä biljardipalloina. Kun se fysiikan taustaoletus on reduktionismi niin tietysti uskotaan että kaikki todellisuus koostuu niistä ja määräytyy niiden mukaan. Makromaailman kannalta ne kuitenkin ovat ehkä yhtä epäolennaisia kuin kuvaruudun yksittäiset pikselit."

        Tämä on aika mutkikas asia. Eli, reduktionismin tavoiteltavuuden perussyy liittyy *käsiteltävyyteen*, eli jotta todellisuuden malli voisi olla jossakin merkityksessä hyödyllinen, niin sen *täytyy* olla käsiteltävissä oleva ja koska ilmeisesti todellisuus itsessään kuitenkaan *ei* ole käsiteltävissä oleva, niin todellisuuden mallit tosiaan ovat aina abstraktioita, mutta ilmeisesti kuitenkin jotkin niistä ovat silti joidenkin todellisuuden aspektien osalta riittävän hyviä, jotta niitä voidaan käyttää esim. todellisuuden käyttäytymisen ennustamiseen käytännön tavoitteiden kannalta aivan riittävän luotettavasti; tietyillä rajoitteilla.

        Eli, esim. jos tavoitteena on tuottaa sellainen laskennallinen laite; kvanttitietokone, jolla noiden em. mallien käsiteltävyytttä voitaisiin parantaa, niin on hyödyllistä mallintaa todellisuuden käyttäytymistä siten, että ajatellaan, että tuo laskenta kuvataan; abstraktilla tasolla, johtuvaksi alkeishiukkasten vuorovaikutuksesta, vaikka tuo vuorovaikutus ei tietenkään "tapahdukaan tyhjiössä", vaan koko todellisuus tosiaan välillisesti *kontribuoi* tuohon laskentaan ja on itse asiassa myös sen *edellytyskin*, koska kuitenkin näyttää siltä, että tuollainenkin abstraktio on riittävä, jotta se tuottaa kontribuutiota mm. tuollaisen laitteen konstruoinnin mahdollistamisen kannalta...

        XPR-0006:
        "Matemaattiset asiat toisaalta eivät voi sisältää informaatiota, .. "

        Matematiikka on (myös) kieli ja aivan kuten luonnollisen kielen tapauksessa matematiikalla voi kuvata todellisia tai myös fysikaalisia (fysikaalisuus laajasti ymmärrettynä) rakenteita mutta samalla tavoin kuten luonnollisen kielen tapauksessa voidaan yhtä hyvin tuottaa fiktiota jolla ei ole välttämättä ainakaan eksaktia vastinetta fysikaalisessa maailmassa vaikka voikin olla silti suhteellisen toimiva todellisuuden kuvaus vertauskuvan tai analogian tasolla eli fiktion avulla voi esittää myös sellaisia (uusia) ideoita joita ei ole mahdollista (jollain hetkellä) ilmaista vielä riittävällä tieteellisellä tarkkuudella ja "voimalla".

        Matematiikkaa on hyvin monenlaista eivätkä kaikki matematiikan lajit ole johdettavissa yhteisistä aksioomista. Minusta matemaattisten asioiden olemassaolo ei siis mitenkään välttämättä edellytä niitä vastaavien fysikaalisten rakenteiden olemassaoloa (jos ymmärsin oikein ilmaisusi merkityksen).

        On olemassa myös sellaista matematiikkaa joka ei ole pelkästään laskennallista eli luonteltaan kvantitatiivista vaan pystyy myös käsittelemään järjestelmien erilaisia laadullisia ominaisuuksia (esim. topologia jota voi käyttää kuvaamaan hyvinkin epälineaarisia asioita kuten esim. tietoisuuden rakenteita ja toimintoja tutkiva psykologia (Kurt Lewin Field theory) tai jota voi käyttää analysoimaan ehkä jopa ns. uskonnollisten tai metafyysisten oppien rakenteellisia merkityksiä suht. täsmällisesti).

        Jossain muinaisissa metafyysisissä järjestelmissä numeroilla (lähinnä 0-12) oli lähinnä symbolisia "funktioon" liittyviä ominaisuuksia hieman topologiseen tyyliin (esim. muinainen Egypti, Sumer yms.)

        esim. jotenkin tähän tyyliin

        https://en.wikipedia.org/wiki/Systematics_–_study_of_multi-term_systems

        https://www.systematics.org/

        Yleisesti ottaen mitä täsmällisempi ja aksiomaattisempi se kuvauskieli on niin sitä suljetumpi systeemi se samalla myös on eli se ei pysty käsittelemään sellaisia asioita joita sen systeemin aksioomeissa ei ole määritelty todellisiksi ja olemassaoleviksi.

        Analogioiden ja vertauskuvien avulla päättely taas on jo luonnostaan avointa eli mahdollistaa tutkittavan asian tunnetuista aspekteista siirtymisen niiden tuntemattomiin aspekteihin ja ehkä sellaisenkin ajattelun voisi jotenkin formalisoida eksaktimmaksi esim. Leibzinin ja Chomskyn ideoiden ja jonkinlaisen topologian pohjalta jolloin tarkka minkä tahansa tilanteen missä tahansa viitekehyksessä analysointi olisi mahdollista riippumatta siitä minkä tieteiden reviireihin ne asiat perinteisesti kuuluvat.

        "...Matemaattisen kuvausten hyödyllisyys liittyy käsiteltävyyteen..."

        Niin ja tekniikkaan eli matematisoitu deterministinen ja reduktionistinen tiede palvelee nimenomaan tekniikan kehitystä ja uusi tekniikka taas takaisinkytkentänä ja uusina havaintovälineinä vahvistaa sellaista tiedettä mutta silti luonnollinen kieli varsinkin yhdistettynä erikoisalojen jargoniin kykenee pääsääntöisesti ilmaisemaan monipuolisemmin vaikkakin ehkä vähäisemmällä tarkkuudella todellisuutta ainakin ns. laadullisten aspektien suhteen joita ei ainakaan vielä tieteelliseltä kannalta kyetä kunnolla ymmärtämään.

        Belisario (jatkoa lähipäivinä)


      • Anonyymi
        Anonyymi kirjoitti:

        Nimimerkki: Professori Realisti (XPR-0007)

        Belisario: "Kyllä minusta systeemi voi olla esim. määräytymättömässä tilassa potentiaalisuutensa takia ja siksi että vain osa sen systeemin ominaisuuksista on aktualisoitunut kunakin hetkenä."

        Tuo liittyy semantiikkaan ja epistemologiaan, eli tuollainen "potentiaalisuus" on mahdollista *siinä merkityksessä*, että jos ko. systeemillä on *sisäistä reaalista rakennetta*, niin tuon rakenteen tilan voidaan tulkita olevan eräs niistä edellytyksistä, jotka mahdollistavat havaitunkaltaisen systeemin käyttäytymisen.

        Eli, tuo "potentiaalisuus" on vain abstrakti ilmaus, eikä todellista potentiaalisuutta taatusti voi olla olemassa ilman reaalisia sen mahdollistavia tekijöitä. Tarkoitan siis sitä, että vaikka todellisiin vaikutuksiin liittyvät havainnot tosiaan vaihtelevatkin, niin *kukin todellinen vaikutus* on silti väistämättä olemassa nimenomaan juuri tietynlaisena, eikä itsensä kumoavana olemattomuutena...

        "Tietysti jos kielletään todennäköisyysjakauman olemassaolo ja oletetaan että kaikki mahdollisuudet ovat aina täysin deterministisesti aktualisoituneita kukin omassa multiversumin maailmassaan niin tietenkin se väistämätön johtopäätös sen tyyppisistä oletuksista on se superdeterminismi."

        Todennäköisyysjakaumahan on olemassa *havaintojen kuvauksena*. Kantani on se, että *kaikki* todelliset mahdollisuudet ja "potentiaalit" tosiaan väistämättä aktualisoituvat. Superdeterminismiin päädytään myös esim. *sitäkin* kautta, että mitkään yksittäiset havainnot eivät siis koskaan ole oikeasti epämääräisiä. Eli, nuo kvantti-ilmiöiden näennäiset epämääräisyydetkin ovat vain yksittäisistä havainnoista koostuvien havaintosarjojen *tulkintoja*.

        "Kyllä esim. kokeentekijällä pitää olla jonkinlainen valinnanvapaus ja jos ei ole niin ei se tiedekään olisi edes mahdollista eikä mielekästä."

        Kokeentekijällä ei tietenkään ole minkäänlaista todellista valinnanvapautta ja *silti* tiede tuottaa tuloksia. Ehkä haluat perustella tarkemmin noita käsittämättömiä oletuksiasi, niin perustelen sitten tarkemmin noita omia johtopäätöksiäni.

        "Heh. Esim. eduskunnassa voi jäädä pois äänestyksestä eli ei valitse kumpaakaan ja se on todennäköisesti hyvin usein täysin harkittu valinta. Joissain maissa kansalaisilla on äänestyspakko sakon uhalla jolloin voi sitten piirellä sinne äänestyslappuun vaikka Aku Ankan kuvan jos mikään tarjolla olevista ehdokkaista ei ole mieleen. Sekin on valinta."

        Oleellistahan tässä ei ole *näennäisesti tarjolla olevien vaihtoehtojen lukumäärä*, vaan se, että ne kaikki eivät havaintojen mukaan voi toteutua *siinä kontekstissa*, joissa niitä havainnoidaan. Eli, MWI:n mukaan ne kaikki tosiaan toteutuvat, mutta ne *eivät* voi toteutua tässä universumissa, vaan ne toteutuvat niissä muissa universumeissa; tai vaihtoehtoisesti tämän universumin *äärimmäisen* kaukaisissa osissa...

        "Tätä tahdonvapaushumppaa olemme veivanneet aikaisemmin jo kyllästymiseen asti joten erimielisyys tulee varmaan edelleenkin säilymään sen ja varmaan myös sen superdeterminismin suhteen jonka takia joudut kieltämään samalla sen valinnanmahdollisuuden."

        Tuo "kieltämisenikinhän" on siinä merkityksessä vain näennäistä, että kaikki "valinnat" tosiaan ovat pakotettuja, koska vain pakottavat syyt voivat saada aikaan todellisia vaikutuksia. Tuon "valinta-teeman" osalta debattimme antina voisi ehkä ideaalitilanteessa olla ainakin se, että kumpikin saisi muotoiltua teesinsä ja niiden taustaoletukset mahdollisimman selkeiksi ja yksikäsitteisiksi.

        "B(elisario)ja jatkossa pelkkä B niin säästyy vähän vaivaa.... :D"

        Hyvä, että olet mukana keskusteluissa, sillä se selvästi korreloi positiivisesti niiden keskimääräisen laadun kanssa. Hyvin lyhyisiin tunnisteisiin liittyy se tunnettu ongelma, että jos haluaa esim. tsekata, että mitä olet kirjoittanut viime aikoina, niin tuo "B" tuottaa myös ns. "false positive" -tapauksia... (esim. Brexit...), joskin ymmärrän kyllä tietysti toisaalta myös sen, että tietynlaisesta "esitysteknisestä epämääräisyydestä" on mm. se hyöty, että on vähemmän todennäköistä, että kommenttien sisältö muotoutuu kirjoittajaan liittyvien ennakkokäsitysten perusteella.

        XPR-0007:

        "Tuo liittyy semantiikkaan ja epistemologiaan, .... "

        Minusta olet turhan sitoutunut jonkinlaiseen koneparadigmaan jossa kaikki potentiaalisuus johtuu ainoastaan fysikaalisesta, kompleksisesta, eksplisiittisestä ja mekaanisesta järjestyksestä kun taas asian voisi myös hahmottaa täysin toisin esim. sillä periaatteella että perusmaterian ominaisuus on aina jollain tavalla kaoottinen ja satunnainen ja kaikki mahdollinen järjestys syntyy nimenomaan tietoisuuden tai korkeamman tason implisiittisen järjestyksen tuottamana eli ts. nonlokaali tietoisuus tuottaa aineellisen maailman järjestyksen jonkinlaisen lomittumisen tai analogisen epälineaarisen prosessin kautta (vrt. sympathetic magic renesanssiajan alkemistien teoksissa)

        Tekniikan suhde tietoisuuteen ja intention vaikutus erilaisiin satunnaisprosesseihin:

        http://pearlab.icrl.org/

        Intentio aineen järjestyksen luojana:

        https://tillerinstitute.com/

        Huomasin äsken J.P. Farrellin kotisivulla mielenkiintoisen blogin joka oli julkaistu tänään samasta aiheesta josta kirjoitin tänä aamuna tänne.

        https://gizadeathstar.com/2020/07/randonauting-attractors-voids-and-some-high-octane/

        "Eli, tuo "potentiaalisuus" on vain abstrakti ilmaus..."

        Ei vaan se on aivan yhtä konkreettinen ellei jopa todellisempi kuin se mitä kutsumme aineelliseksi todellisuudeksi. Metafyysisillä ontologisilla uskomuksilla kuten mekanistinen reduktionistinen materialismi on taipumus supistaa koettu todellisuus niin pieneksi että sitä kyetään jotenkin käsitteellisesti ja teorettisesti hahmottamaan "tarkasti" ja siten tuotetaan vain hyvin pelkistetty ja abstrakti kuvaus todellisuudesta:

        Toinen vaihtoehto on sitten olla ontologisesti sitoutumaton tai käyttää useampia näennäisesti toisiinsa nähden ristiriitaisia ontologioita samanaikaisesti ja tyytyä jossain mielessä epätarkempaan kuvaukseen joka kuitenkin pyrkii kattamaan myös sellaiset vaihtoehtoiset mallit jotka ovat toistaiseksi hyvinkin spekulatiivisia mutta samalla se kokonaiskuvaus ei ennenaikaisesti sulje pois sellaisia oudoltakaan tuntuvia mahdollisuuksia jotka vaikuttavat pintapuolisesti tarkasteltuna täysin huuhaalta.

        Jos tarkastellaan esim. tieteen historiaa niin hyvin harva ellei kukaan 100 v sitten vaikuttanut tutkija olisi osannut ennustaan nykyistä tekniikkaa ja nykyistä tietämystä.

        Kannattaa siis välttää kaikkea dogmaattisuutta ja sellaisia kuvitelmia että kaikki olennaiset perusasiat tieteessä muka jo tunnetaan ja että jäljellä on enää vain tarkkuuden kasvattaminen.

        Kaikenkaikkiaan minusta näytttää siltä että todellisuus on paljon monimuotoisempi ja oudompi kuin superdeterminismi ja kvanttifysiikan deterministinen monimaailmatulkinta mahdollistaa eikä sellainen malli ole minusta edes kovin kiinnostava koska se johtaa minusta aika selkeään umpikujaan.

        Minun näkemykseni voi tuntua sinusta täysin käsittämättömältä mutta minä kylllä varsin hyvin ymmärrän sinun näkemyksesi ja se muistuttaa minusta jonkinlaista uskonnollista ja hyvin dogmaattista fundamentalismia esim. tuon superdeterminismin ja deterministisen monimaailmatulkinnan suhteen vaikka muita vähintään yhtä hyvin toimivia vaihtoehtoisia malleja ja tulkintoja on runsaasti ja todennäköisesti se lopullinen totuus aiheesta on sellainen ettei kumpikaan meistä kunnolla osaa sitä edes kuvitella mielessään ja itse taas pyrin välttämään kaikkea uskomista mikä mahdollistaa hyvinkin outoihin ja esoteerisiin vaihtoehtoihin perehtymisen ilman että omat uskomukset alkaisivat jotenkin aiheittaa älyllistä tökkimistä tai ähkyä. Minusta ei kannata sitoutua mihinkään malliin tutkimusten ollessa vielä monessa asiassa täysin alkutekijöissään.

        "Hyvä, että olet mukana keskusteluissa, sillä se selvästi korreloi positiivisesti niiden keskimääräisen laadun kanssa. ..."

        Ehkä en välttämättä halua olla jatkuvasti bongattavissa :D.

        Joskus kirjoittelen täysin ilman nikkiä ja käytän sitä vain jos syntyy pidempää keskustelua jonkun kanssa. En myöskään oikein ymmärrä miksi pidät minun kirjoitteluani jotenkin laadukkaana vaikka ilmeisesti olet usein täysin vastakkaista mieltä asioista.

        Aikaisemmin vuosia sitten käytin vaihtuvia nikkejä joista "Mandrake"(n haamu) lienee yleisin ja se lienee lähinnä tarkoitettu haukkumanimeksi käsittelmieni melko esoteeristen ja valtavirrasta poikkeavien kirjoituksieni vuoksi ja siksi koska se nimi muistuttaa skepojen pääinhokin R. Sheldraken nimeä.

        Mandrake on vanha sarjakuvahahmo josta löytyy youtubesta hauskoja vanhoja tv-sarjojen edeltäjiä (ns. serials joita näytettiin leffateattereissa ennen television aikaa 1940-50 luvuilla)

        Esim. Mandrake the Magician-1939

        https://youtu.be/ZYYoAhvXLp4?list=PL6x36jJhLF3FCqrkEiCTGpieKZcVmhU6g

        Belisario


      • Anonyymi
        Anonyymi kirjoitti:

        XPR-0005:

        "Esim. yksittäistä elektronia *vastaavaksi tulkittavien havaintojen*; osaltahan pätee se.... "

        Nyt sitten joudutaan siirtymään keskustelemaan asioista joiden ilmaiseminen ja ymmärtäminen tarkoitetussa merkityksessä on huomattavasti hankalampaa koska ne edellyttävät ainakin jossain määrin yhteismitallista todellisuuden hahmotusta keskustelijoiden välillä ja sekä hyvin poikkitieteellistä että erityisesti poikkiontologista analyysiä jossa mukaan tulee melkein kaikki kuviteltavissa olevat ns. filosofiset tai metafyysiset erimielisyydet mitä erilaisten ontologioiden välillä keskusteluissa voi esiintyä.

        Keskustelu väkisinkin sitten rönsyilee mutta varmaankin silti pysyttelee ketjun otsikon aiheessa joka on "ihanteellisen" väljä tällaiseen.

        Minä käytin tuota ilmaisua "pysäytyskuva" lähinnä enemmän vertauskuvallisessa tai analogisessa merkityksessä kuin jonkin konkreettisen yksittäisen mittaustapahtuman merkityksessä.

        Minä lähden siltä kannalta että esim. se mitä kutsumme mieleksi, tietoisuudeksi tai vielä tarkemmin ilmaistuna ainutlaatuiseksi ja yksilölliseksi persoonallisuudeksi on aina paljon enemmän kuin "osiensa summa" eli esim. ihmisyksilön tapauksessa vaikka fysiikan kannalta ihmisen kehon elimien atomit ja molekyylit jatkuvasti vaihtuvat koko fyysisen elämän ajan niin se itse yksilö tai persoonallisuus pysyy kuitenkin samana vähän samaan tyyliin kuin sama tietokoneohjelmisto erilaisissa tietokoneissa on edelleenkin se sama ohjelmisto (sama logiikka, samat toiminnot) vaikka niiden fysikaalinen ilmentyminen ja laitteisto voi olla erilainen ja moneessa laitteistossa samaan aikaan.

        Tästä tietokoneen ja ohjelmiston vertauskuvasta ei tietenkään voi eikä kannata olettaa sitä että esim. ihminen olisi tietokone ja tietoisuus toimintoineen vain ohjelmisto vaan kyseessä on vain loogis-analoginen päättely jossa tunnetun ja pohjimmiltaan tuntemattoman järjestelmän havaituista rakenne- ja toimintayhtäläisyyksistä voidaan siirtyä tuntemattoman systeemin tutkimiseen niin että samalla informaatiota sekä katoaa (koska ne systeemit eivät ole identtisiä) että lisääntyy (koska ne yhteiset ominaisuudet tarjoavat uuden rajapinnan tai uusia käsitteellisiä työkaluja mahdolliselle jatkotutkimuksille).

        Todellisuuden ontologian "valikoimaan" ainakin minun mielestäni kaikkein olennaisimpina osina kuuluvat tällaiset holistiset systeemit jotka eivät rakennu näkyvistä ja mitattavista komponenteistaan (esim. alkeishiukkaset,atomit & molekyylit) "ylöspäin" konkreettisiksi havainto-objekteiksi (konkreettinen eliöyksilö kuten tietty ihminen, koira, kissa ,puu, huonekasvi, kivi, järvi. vuori, planeetta,aurinko,galaksi jne yms.) jotka ovat jotenkin yksilöllisiä ja ainutlaatuisia ja minusta määrittelevät niiden osiensa käyttäytymistä joilla taas ole mitään yksilöllisyyttä (kuten ne molekyylit, atomit & alkeishiukkaset).

        Ne yksittäiset atomit, molekyylit & alkeishiukkaset kokonaisuudestaan erotettuna voivat minusta ainakin periaatteessa olla juuri sellaisia kuin ne ovat yksiselitteisesti riippumatta havaintojen puutteellisuudesta mutta jonkun laajemman yksilöllisen systeemin osina eivät kuitenkaan määrittele sen kokonaissysteemin toimintaa eli esim. minun pikkusormeni kynnen molekyyli tai atomi ei millään tavalla määrittele eikä determinoi minun toimintaani eikä myöskään niiden atomien ja molekyylien kokonaisuus sitä välttämättä tee koska siinä mitä kutsun pikkusormekseni vaihtuu fysikaalisen ainesosat jatkuvasti ja vuosikymmenien kuluttua siinä ei välttämättä ole enää ainuttakaan samaa atomia kuin oli esim. syntymähetkelläni ja samaa koskee kaikkia muitakin kehoni elimiä.

        "koska voimme siis loogisesti päätellä, että *todelliset* asiat eivät voi olla epämääräisiä; ...:D"

        Voisi ehkä ottaa myös huomioon sellainen mahdollisuus että kaikkien olemassaolevien asioiden "todellisuus" tai "konkreettisuus" ei ole aina samantasoista vaan ikäänkuin hierarkkinen ontologia joissa sen ontologisen hierarkian "ylätasot" (ns. "yksilölliset" oliot) determinoivat ja organisoivat niitä alemman ontologisen tason ("epäyksilölliset" atomit, alkeishiukkaset & molekyylit) ainetta jolloin se alataso voi olla tietyssä mielessä deterministinen ja tarkoin määriteltävissä eli "aktuaalinen" kun taas se "ylempi" taso aistihavaintojen kannalta näyttäytyy aina jotenkin epämääräisenä ja potentiaalisena.

        Ontologisen hierarkian ylätaso= implisiittinen järjestys joka määrittelee alemman tason säännönmukaisuudet

        Ontologisen hierakian alataso= eksplisiittinen järjestys joka käyttäytyy pääsääntöisesti sen ylemmän tason määrittelmien "luonnonlakien" mukaan.

        Minua jostain syystä miellyttävät ja kiinnostavat kaikenlaiset avoimet ja vähän epämääräisemmät systeemit ja ideat joista voi kehittyä jotain uutta ja mielenkiintoista juuri nimenomaan sen takia että ne eivät ole etukäteen lukkoon lyötyjä vaan ainakin osittain ominaisuuksiltaan vielä tuntemattomia ja edellyttävät ehkä uusia analogioita.

        Belisario

        (jatkuu ehkä huomenna....)

        Nimimerkki: Professori Realisti (XPR-0008)

        Belisario: "Nyt sitten [...] mukaan tulee melkein kaikki kuviteltavissa olevat ns. filosofiset tai metafyysiset erimielisyydet mitä erilaisten ontologioiden välillä keskusteluissa voi esiintyä."

        Kiitos "kannustuksesta". :D

        "Minä lähden siltä kannalta että esim. se mitä kutsumme mieleksi, tietoisuudeksi tai vielä tarkemmin ilmaistuna ainutlaatuiseksi ja yksilölliseksi persoonallisuudeksi on aina paljon enemmän kuin "osiensa summa""

        Ymmärtänen hyvin mitä noilla tarkoitat ja tuon tässä vain esille joitakin sekalaisia kommenttejani. Ensinnäkin, minusta *kaikki* on tavallaan "enemmän kuin osiensa summa", *siinä merkityksessä*, että noiden osien ja myös muiden osien välillä on vuorovaikutusta, mutta en siis usko, että *reaalinen* vuorovaikutus voisi olla epämääräistä... Toisaalta, esim. ihmisen tapauksessa, kyllähän muutoksia tapahtuu myös persoonallisuuden tasolla, *jos* todellisuus niihin pakottaa, eli ihmisen mielessä on keskimäärin joitakin melko pysyviä osia, mutta lopultahan kaikki kuitenkin muuttuu. Ilmeisesti ihmisen muistin jatkuva toiminta on edellytys sille, että kokemuksellisten sisältöjen välillä koetaan olevan jatkuvuutta.

        "joilla taas ole mitään yksilöllisyyttä (kuten ne molekyylit, atomit & alkeishiukkaset)."

        Ymmärtänen hyvin, mitä tarkoittanet sanoa, mutta sanoisin lisäksi, että *niilläkin* on tavallaan jonkinlainen identiteetti; siinä merkityksessä, että niillä on tietty vuorovaikutus ajan funktiona muiden alkeishiukkasten jne. kanssa ja toisaalta ihmistenkin yksilöllisyys on ajallisesti rajattua.

        "eli esim. minun pikkusormeni kynnen molekyyli tai atomi ei millään tavalla määrittele eikä determinoi minun toimintaani"

        En ole tuosta samaa mieltä, *juuri koska* todellisuus on vuorovaikutusverkosto, eli mielestäni kohtelet nyt tuota atomia "epäreilun epämystifioivasti"; vastoin yleistä filosofiaasi, sillä filosofiasi mukaisesti olisi johdonmukaista olla poissulkematta sitä "epämääräistä mahdollisuutta", että se määrittääkin kaiken toimintasi. Ymmärrän, että yrität vakavissasi selittää tuota asiaa ja kiitos siitä, mutta kyllä se toisaalta kuulostaa siis aika hassultakin, ottaen huomioon mystifioivan filosofiasi, eli otat tuossa kuitenkin tietyn todellisuuden osan tarkasteluun ja päätät, että sillä ei voi olla mystisiä ominaisuuksia tai edes *mitään* vaikutustakaan toimintaasi. :D

        "Voisi ehkä ottaa myös huomioon sellainen mahdollisuus että kaikkien olemassaolevien asioiden "todellisuus" tai "konkreettisuus" ei ole aina samantasoista vaan ikäänkuin hierarkkinen ontologia joissa sen ontologisen hierarkian "ylätasot" (ns. "yksilölliset" oliot) determinoivat ja organisoivat niitä alemman ontologisen tason ("epäyksilölliset" atomit, alkeishiukkaset & molekyylit) ainetta jolloin se alataso voi olla tietyssä mielessä deterministinen ja tarkoin määriteltävissä eli "aktuaalinen" kun taas se "ylempi" taso aistihavaintojen kannalta näyttäytyy aina jotenkin epämääräisenä ja potentiaalisena."

        Ymmärtänen hyvin, mitä tuolla tarkoitat, mutta itse en usko, että mikään "todellisuuden taso" tai osa olisi oikeasti muita "determinoivammassa roolissa", vaan tuo vaikutelma on vain seurausta tavasta mallintaa todellisuutta. On tietenkin aivan luonnollista, että kompleksisemmilla järjestelmillä voidaan mieltää olevan enemmän ominaisuuksia kuin vähemmän kompleksisilla. En kuitenkaan usko, että todellisuudesta voidaan poistaa mitään osia; muuttamatta todellisuutta, tai edes siihenkään, että niitä ylipäätäänkään voitaisiin poistaa...

        "Minua jostain syystä miellyttävät ja kiinnostavat kaikenlaiset avoimet ja vähän epämääräisemmät systeemit ja ideat joista voi kehittyä jotain uutta ja mielenkiintoista juuri nimenomaan sen takia että ne eivät ole etukäteen lukkoon lyötyjä vaan ainakin osittain ominaisuuksiltaan vielä tuntemattomia ja edellyttävät ehkä uusia analogioita."

        Tuntematon tosiaan kiehtoo ihmisiä ja hyvä niin, mutta jos toisaalta pyritään aina pitämään "kaikki kannat ns. avoimina", niin ilmeisesti hintana on se, että ei ehditä edetä niin pitkälle johonkin *tiettyyn* suuntaan; tutkimaan uusia tuntemattomia asioita, kuin mikä muuten olisi mahdollista. Kykysi ylläpitää epämääräistä kantaa on myös epätavallinen ja hyvin harvoilla käsittääkseni on tuollaiseen edellytyksiä ja/tai taipumustakaan.


      • Anonyymi
        Anonyymi kirjoitti:

        XPR-0006:
        "Matemaattiset asiat toisaalta eivät voi sisältää informaatiota, .. "

        Matematiikka on (myös) kieli ja aivan kuten luonnollisen kielen tapauksessa matematiikalla voi kuvata todellisia tai myös fysikaalisia (fysikaalisuus laajasti ymmärrettynä) rakenteita mutta samalla tavoin kuten luonnollisen kielen tapauksessa voidaan yhtä hyvin tuottaa fiktiota jolla ei ole välttämättä ainakaan eksaktia vastinetta fysikaalisessa maailmassa vaikka voikin olla silti suhteellisen toimiva todellisuuden kuvaus vertauskuvan tai analogian tasolla eli fiktion avulla voi esittää myös sellaisia (uusia) ideoita joita ei ole mahdollista (jollain hetkellä) ilmaista vielä riittävällä tieteellisellä tarkkuudella ja "voimalla".

        Matematiikkaa on hyvin monenlaista eivätkä kaikki matematiikan lajit ole johdettavissa yhteisistä aksioomista. Minusta matemaattisten asioiden olemassaolo ei siis mitenkään välttämättä edellytä niitä vastaavien fysikaalisten rakenteiden olemassaoloa (jos ymmärsin oikein ilmaisusi merkityksen).

        On olemassa myös sellaista matematiikkaa joka ei ole pelkästään laskennallista eli luonteltaan kvantitatiivista vaan pystyy myös käsittelemään järjestelmien erilaisia laadullisia ominaisuuksia (esim. topologia jota voi käyttää kuvaamaan hyvinkin epälineaarisia asioita kuten esim. tietoisuuden rakenteita ja toimintoja tutkiva psykologia (Kurt Lewin Field theory) tai jota voi käyttää analysoimaan ehkä jopa ns. uskonnollisten tai metafyysisten oppien rakenteellisia merkityksiä suht. täsmällisesti).

        Jossain muinaisissa metafyysisissä järjestelmissä numeroilla (lähinnä 0-12) oli lähinnä symbolisia "funktioon" liittyviä ominaisuuksia hieman topologiseen tyyliin (esim. muinainen Egypti, Sumer yms.)

        esim. jotenkin tähän tyyliin

        https://en.wikipedia.org/wiki/Systematics_–_study_of_multi-term_systems

        https://www.systematics.org/

        Yleisesti ottaen mitä täsmällisempi ja aksiomaattisempi se kuvauskieli on niin sitä suljetumpi systeemi se samalla myös on eli se ei pysty käsittelemään sellaisia asioita joita sen systeemin aksioomeissa ei ole määritelty todellisiksi ja olemassaoleviksi.

        Analogioiden ja vertauskuvien avulla päättely taas on jo luonnostaan avointa eli mahdollistaa tutkittavan asian tunnetuista aspekteista siirtymisen niiden tuntemattomiin aspekteihin ja ehkä sellaisenkin ajattelun voisi jotenkin formalisoida eksaktimmaksi esim. Leibzinin ja Chomskyn ideoiden ja jonkinlaisen topologian pohjalta jolloin tarkka minkä tahansa tilanteen missä tahansa viitekehyksessä analysointi olisi mahdollista riippumatta siitä minkä tieteiden reviireihin ne asiat perinteisesti kuuluvat.

        "...Matemaattisen kuvausten hyödyllisyys liittyy käsiteltävyyteen..."

        Niin ja tekniikkaan eli matematisoitu deterministinen ja reduktionistinen tiede palvelee nimenomaan tekniikan kehitystä ja uusi tekniikka taas takaisinkytkentänä ja uusina havaintovälineinä vahvistaa sellaista tiedettä mutta silti luonnollinen kieli varsinkin yhdistettynä erikoisalojen jargoniin kykenee pääsääntöisesti ilmaisemaan monipuolisemmin vaikkakin ehkä vähäisemmällä tarkkuudella todellisuutta ainakin ns. laadullisten aspektien suhteen joita ei ainakaan vielä tieteelliseltä kannalta kyetä kunnolla ymmärtämään.

        Belisario (jatkoa lähipäivinä)

        Nimimerkki: Professori Realisti (XPR-0009)

        "Minusta matemaattisten asioiden olemassaolo ei siis mitenkään välttämättä edellytä niitä vastaavien fysikaalisten rakenteiden olemassaoloa (jos ymmärsin oikein ilmaisusi merkityksen)".

        Matematiikkaan liittyvät näkökantamme eivät näyttäisi juurikaan eroavan toisistaan. Nykyinen kantani ei ole sama kuin Tegmark:in, eli minusta tuollaista *vastaavuutta* ei tarvita, mutta fysikaaliset *edellytykset* silti tarvitaan, eli olisi mieletöntä puhua sellaisista matemaattisista rakenteista, joiden kuvaukset eivät voisi realisoitua. Tegmark lienee analysoinut sitä, että mitä tuollainen realisoituminen *edellyttää* paljonkin, mutta en ole ehtinyt tutustua siihen enempää, vaikka aihe tietysti kiinnostaakin...

        "Yleisesti ottaen mitä täsmällisempi ja aksiomaattisempi se kuvauskieli on niin sitä suljetumpi systeemi se samalla myös on eli se ei pysty käsittelemään sellaisia asioita joita sen systeemin aksioomeissa ei ole määritelty todellisiksi ja olemassaoleviksi."

        Päteekö myös omasta mielestäsi tältä osin ns. "nollasummapeli", eli että ei ole mitään keinoa lisätä "tarkoitetulla tavalla kommunikoitavissa olevan informaation määrää", lisäämättä viestin pituutta? Eli, joko ollaan täsmällisiä ja jotain jää viittaamatta tai ollaan epämääräisiä ja mahdollistetaan ei-tarkoitetut viestin tulkinnat.

        "Analogioiden ja vertauskuvien avulla päättely taas on jo luonnostaan avointa eli mahdollistaa tutkittavan asian tunnetuista aspekteista siirtymisen niiden tuntemattomiin aspekteihin ja ehkä sellaisenkin ajattelun voisi jotenkin formalisoida eksaktimmaksi esim. Leibzinin ja Chomskyn ideoiden ja jonkinlaisen topologian pohjalta jolloin tarkka minkä tahansa tilanteen missä tahansa viitekehyksessä analysointi olisi mahdollista riippumatta siitä minkä tieteiden reviireihin ne asiat perinteisesti kuuluvat."

        Tuo kuulostaa varsin mielenkiintoiselta idealta.


      • Anonyymi
        Anonyymi kirjoitti:

        XPR-0007:

        "Tuo liittyy semantiikkaan ja epistemologiaan, .... "

        Minusta olet turhan sitoutunut jonkinlaiseen koneparadigmaan jossa kaikki potentiaalisuus johtuu ainoastaan fysikaalisesta, kompleksisesta, eksplisiittisestä ja mekaanisesta järjestyksestä kun taas asian voisi myös hahmottaa täysin toisin esim. sillä periaatteella että perusmaterian ominaisuus on aina jollain tavalla kaoottinen ja satunnainen ja kaikki mahdollinen järjestys syntyy nimenomaan tietoisuuden tai korkeamman tason implisiittisen järjestyksen tuottamana eli ts. nonlokaali tietoisuus tuottaa aineellisen maailman järjestyksen jonkinlaisen lomittumisen tai analogisen epälineaarisen prosessin kautta (vrt. sympathetic magic renesanssiajan alkemistien teoksissa)

        Tekniikan suhde tietoisuuteen ja intention vaikutus erilaisiin satunnaisprosesseihin:

        http://pearlab.icrl.org/

        Intentio aineen järjestyksen luojana:

        https://tillerinstitute.com/

        Huomasin äsken J.P. Farrellin kotisivulla mielenkiintoisen blogin joka oli julkaistu tänään samasta aiheesta josta kirjoitin tänä aamuna tänne.

        https://gizadeathstar.com/2020/07/randonauting-attractors-voids-and-some-high-octane/

        "Eli, tuo "potentiaalisuus" on vain abstrakti ilmaus..."

        Ei vaan se on aivan yhtä konkreettinen ellei jopa todellisempi kuin se mitä kutsumme aineelliseksi todellisuudeksi. Metafyysisillä ontologisilla uskomuksilla kuten mekanistinen reduktionistinen materialismi on taipumus supistaa koettu todellisuus niin pieneksi että sitä kyetään jotenkin käsitteellisesti ja teorettisesti hahmottamaan "tarkasti" ja siten tuotetaan vain hyvin pelkistetty ja abstrakti kuvaus todellisuudesta:

        Toinen vaihtoehto on sitten olla ontologisesti sitoutumaton tai käyttää useampia näennäisesti toisiinsa nähden ristiriitaisia ontologioita samanaikaisesti ja tyytyä jossain mielessä epätarkempaan kuvaukseen joka kuitenkin pyrkii kattamaan myös sellaiset vaihtoehtoiset mallit jotka ovat toistaiseksi hyvinkin spekulatiivisia mutta samalla se kokonaiskuvaus ei ennenaikaisesti sulje pois sellaisia oudoltakaan tuntuvia mahdollisuuksia jotka vaikuttavat pintapuolisesti tarkasteltuna täysin huuhaalta.

        Jos tarkastellaan esim. tieteen historiaa niin hyvin harva ellei kukaan 100 v sitten vaikuttanut tutkija olisi osannut ennustaan nykyistä tekniikkaa ja nykyistä tietämystä.

        Kannattaa siis välttää kaikkea dogmaattisuutta ja sellaisia kuvitelmia että kaikki olennaiset perusasiat tieteessä muka jo tunnetaan ja että jäljellä on enää vain tarkkuuden kasvattaminen.

        Kaikenkaikkiaan minusta näytttää siltä että todellisuus on paljon monimuotoisempi ja oudompi kuin superdeterminismi ja kvanttifysiikan deterministinen monimaailmatulkinta mahdollistaa eikä sellainen malli ole minusta edes kovin kiinnostava koska se johtaa minusta aika selkeään umpikujaan.

        Minun näkemykseni voi tuntua sinusta täysin käsittämättömältä mutta minä kylllä varsin hyvin ymmärrän sinun näkemyksesi ja se muistuttaa minusta jonkinlaista uskonnollista ja hyvin dogmaattista fundamentalismia esim. tuon superdeterminismin ja deterministisen monimaailmatulkinnan suhteen vaikka muita vähintään yhtä hyvin toimivia vaihtoehtoisia malleja ja tulkintoja on runsaasti ja todennäköisesti se lopullinen totuus aiheesta on sellainen ettei kumpikaan meistä kunnolla osaa sitä edes kuvitella mielessään ja itse taas pyrin välttämään kaikkea uskomista mikä mahdollistaa hyvinkin outoihin ja esoteerisiin vaihtoehtoihin perehtymisen ilman että omat uskomukset alkaisivat jotenkin aiheittaa älyllistä tökkimistä tai ähkyä. Minusta ei kannata sitoutua mihinkään malliin tutkimusten ollessa vielä monessa asiassa täysin alkutekijöissään.

        "Hyvä, että olet mukana keskusteluissa, sillä se selvästi korreloi positiivisesti niiden keskimääräisen laadun kanssa. ..."

        Ehkä en välttämättä halua olla jatkuvasti bongattavissa :D.

        Joskus kirjoittelen täysin ilman nikkiä ja käytän sitä vain jos syntyy pidempää keskustelua jonkun kanssa. En myöskään oikein ymmärrä miksi pidät minun kirjoitteluani jotenkin laadukkaana vaikka ilmeisesti olet usein täysin vastakkaista mieltä asioista.

        Aikaisemmin vuosia sitten käytin vaihtuvia nikkejä joista "Mandrake"(n haamu) lienee yleisin ja se lienee lähinnä tarkoitettu haukkumanimeksi käsittelmieni melko esoteeristen ja valtavirrasta poikkeavien kirjoituksieni vuoksi ja siksi koska se nimi muistuttaa skepojen pääinhokin R. Sheldraken nimeä.

        Mandrake on vanha sarjakuvahahmo josta löytyy youtubesta hauskoja vanhoja tv-sarjojen edeltäjiä (ns. serials joita näytettiin leffateattereissa ennen television aikaa 1940-50 luvuilla)

        Esim. Mandrake the Magician-1939

        https://youtu.be/ZYYoAhvXLp4?list=PL6x36jJhLF3FCqrkEiCTGpieKZcVmhU6g

        Belisario

        Nimimerkki: Professori Realisti (XPR-0010)

        Belisario: "Minusta olet turhan sitoutunut jonkinlaiseen koneparadigmaan..."

        Todellisuus pakottaa minut ulkoistamaan kuvitteellisen satunnaisuuden systeemistäni...

        "eli ts. nonlokaali tietoisuus tuottaa aineellisen maailman järjestyksen jonkinlaisen lomittumisen tai analogisen epälineaarisen prosessin kautta (vrt. sympathetic magic renesanssiajan alkemistien teoksissa)"

        Tarkoitatko, että se tosiaan tuottaa tuon *järjestyksen*, vaiko vain sen *kuvauksen"? En usko ollenkaan tuollaiseen, vaan mm. antrooppisen periaatteen ns. heikkoon muotoon, eli siihen että havaitsemisen ja tietoisuuden sisältöjen ja niiden edellytysten välillä vallitsee ilmeinen korrelaatio.

        "Ei vaan se [potentiaali] on aivan yhtä konkreettinen ellei jopa todellisempi kuin se mitä kutsumme aineelliseksi todellisuudeksi."

        Tullaanko tässä siihen, että käytät termiä "potentiaali" jossakin sellaisessa merkityksessä, että tuota merkitystä ei voida kommunikoida täsmällisesti, vai mistä tässä on kyse? Eli, yleisesti tunnettu kommunikaatio-ongelmahan on se, että ns. mystisiä kokemuksia ei välttämättä kyetä kuvaamaan niin hyvin, että kommunikaatio onnistuisi. Eli, jos esim. kahdella henkilöllä on kummallakin samankaltaisia mystisiä kokemuksia, niin he voivat kylläkin ymmärtää toisiaan *viittaamalla* samankaltaisiin kokemuksiin, mutta jos tuollaista yhteistä kokemuspohjaa ei ole, niin kielellinen kommunikaatio ei onnistu.

        "Toinen vaihtoehto on sitten olla ontologisesti sitoutumaton tai käyttää useampia näennäisesti toisiinsa nähden ristiriitaisia ontologioita samanaikaisesti..."

        Kuten aiemmin olen todennutkin, minun rajanani on (vain) mahdottomuus, mutta siltä osin en tunne epämääräisyyttä. :D

        "Kannattaa siis välttää kaikkea dogmaattisuutta ja sellaisia kuvitelmia että kaikki olennaiset perusasiat tieteessä muka jo tunnetaan ja että jäljellä on enää vain tarkkuuden kasvattaminen."

        Kannatatko oikeasti tuota dogmia, vai oletko vain kannattavinasi sitä? :D

        "Kaikenkaikkiaan minusta näytttää siltä että todellisuus on paljon monimuotoisempi ja oudompi kuin superdeterminismi ja kvanttifysiikan deterministinen monimaailmatulkinta mahdollistaa eikä sellainen malli ole minusta edes kovin kiinnostava koska se johtaa minusta aika selkeään umpikujaan."

        Multiversumin tulkinta umpikujaksi on hyvin outo näkemys, koska multiversumihan toteuttaa *väistämättä* *kaikki* mahdollisuudet...

        "Minun näkemykseni voi tuntua sinusta täysin käsittämättömältä mutta minä kylllä varsin hyvin ymmärrän sinun näkemyksesi ja se muistuttaa minusta jonkinlaista uskonnollista ja hyvin dogmaattista fundamentalismia esim. tuon superdeterminismin ja deterministisen monimaailmatulkinnan suhteen"

        Fundamentalismia tuskin on se, että ei usko ilmeisen mahdottomiin asioihin.

        "Minusta ei kannata sitoutua mihinkään malliin tutkimusten ollessa vielä monessa asiassa täysin alkutekijöissään."

        Mutta olemmeko esim. *me* enää alkutekijöissämme, eli onko aikaa olla sitoutumatta? Eli, ehkä pääero "filosofioissamme" on se, että en usko, että optimaalinen toimintatapa on odottaa varman tiedon ilmaantumista sellaisten asioiden osalta, joiden osalta sitä ei välttämättä koskaan ilmaannukaan. Eli, tietenkin tuo tarkoittaa riskin ottamista, mutta jos esim. kokemusten perusteella muotoutunut ns. sisäinen vakaumus on se, että indeterminismi on mahdotonta, niin miksi loppuelämänsä pitäisi viettää pähkäilemällä tuota asiaa?

        "En myöskään oikein ymmärrä miksi pidät minun kirjoitteluani jotenkin laadukkaana vaikka ilmeisesti olet usein täysin vastakkaista mieltä asioista."

        Kyllähän kirjoitusta laadukkaana voi pitää, vaikka ei olisikaan samaa mieltä sen sisällöstä. Kirjoitat hyvällä suomenkielellä ja tuot esille mielenkiintoisia näkemyksiä, vaikka en tietenkään hurahdakaan niihin kaikkiin... Lisäksi "näkemisen ääni" kertoo minulle usein, että olet älykäs, joten kirjoituksissasi täytyy olla *jotain* järkeä. :D

        "Aikaisemmin vuosia sitten käytin vaihtuvia nikkejä joista "Mandrake"(n haamu) lienee yleisin..."

        Käsittääkseni olemme itseasiassa molemmat mystikoita. Sinä olet käsittääkseni ns. "epämääräinen mystikko" ja minä "deterministinen mystikko". Nuo ovat mystikoiden koulukuntia vähän samaan tapaan kuin kung fu -koulut Kiinassa. Vähän olen kuitenkin havainnut sellaista tendenssiä, että oletat, että olen reduktionisti, kaiketi lähinnä sillä perusteella, että pidän täsmällisyyttä mm. kommunikaation kannalta tavoiteltavana asiana, eli vähän samaan tapaan kuin, jos joku on aina bongannut vain mustia variksia, niin jos sitten vastaan tuleekin valkoinen varis, niin sitä vaan itsepintaisesti väittää joutseneksi tms., koska valkoinen varis ei ole ennestään tuttu asia. :D

        [Kirjoittamisiini tässä ketjussa voi nyt tulla enemmän tai vähemmän taukoa, sillä aion vähän välillä tutkiskella erilaisten kvanttimekaanisten tulkintojen ihmeellistä maailmaa...]


      • Anonyymi
        Anonyymi kirjoitti:

        Nimimerkki: Professori Realisti (XPR-0008)

        Belisario: "Nyt sitten [...] mukaan tulee melkein kaikki kuviteltavissa olevat ns. filosofiset tai metafyysiset erimielisyydet mitä erilaisten ontologioiden välillä keskusteluissa voi esiintyä."

        Kiitos "kannustuksesta". :D

        "Minä lähden siltä kannalta että esim. se mitä kutsumme mieleksi, tietoisuudeksi tai vielä tarkemmin ilmaistuna ainutlaatuiseksi ja yksilölliseksi persoonallisuudeksi on aina paljon enemmän kuin "osiensa summa""

        Ymmärtänen hyvin mitä noilla tarkoitat ja tuon tässä vain esille joitakin sekalaisia kommenttejani. Ensinnäkin, minusta *kaikki* on tavallaan "enemmän kuin osiensa summa", *siinä merkityksessä*, että noiden osien ja myös muiden osien välillä on vuorovaikutusta, mutta en siis usko, että *reaalinen* vuorovaikutus voisi olla epämääräistä... Toisaalta, esim. ihmisen tapauksessa, kyllähän muutoksia tapahtuu myös persoonallisuuden tasolla, *jos* todellisuus niihin pakottaa, eli ihmisen mielessä on keskimäärin joitakin melko pysyviä osia, mutta lopultahan kaikki kuitenkin muuttuu. Ilmeisesti ihmisen muistin jatkuva toiminta on edellytys sille, että kokemuksellisten sisältöjen välillä koetaan olevan jatkuvuutta.

        "joilla taas ole mitään yksilöllisyyttä (kuten ne molekyylit, atomit & alkeishiukkaset)."

        Ymmärtänen hyvin, mitä tarkoittanet sanoa, mutta sanoisin lisäksi, että *niilläkin* on tavallaan jonkinlainen identiteetti; siinä merkityksessä, että niillä on tietty vuorovaikutus ajan funktiona muiden alkeishiukkasten jne. kanssa ja toisaalta ihmistenkin yksilöllisyys on ajallisesti rajattua.

        "eli esim. minun pikkusormeni kynnen molekyyli tai atomi ei millään tavalla määrittele eikä determinoi minun toimintaani"

        En ole tuosta samaa mieltä, *juuri koska* todellisuus on vuorovaikutusverkosto, eli mielestäni kohtelet nyt tuota atomia "epäreilun epämystifioivasti"; vastoin yleistä filosofiaasi, sillä filosofiasi mukaisesti olisi johdonmukaista olla poissulkematta sitä "epämääräistä mahdollisuutta", että se määrittääkin kaiken toimintasi. Ymmärrän, että yrität vakavissasi selittää tuota asiaa ja kiitos siitä, mutta kyllä se toisaalta kuulostaa siis aika hassultakin, ottaen huomioon mystifioivan filosofiasi, eli otat tuossa kuitenkin tietyn todellisuuden osan tarkasteluun ja päätät, että sillä ei voi olla mystisiä ominaisuuksia tai edes *mitään* vaikutustakaan toimintaasi. :D

        "Voisi ehkä ottaa myös huomioon sellainen mahdollisuus että kaikkien olemassaolevien asioiden "todellisuus" tai "konkreettisuus" ei ole aina samantasoista vaan ikäänkuin hierarkkinen ontologia joissa sen ontologisen hierarkian "ylätasot" (ns. "yksilölliset" oliot) determinoivat ja organisoivat niitä alemman ontologisen tason ("epäyksilölliset" atomit, alkeishiukkaset & molekyylit) ainetta jolloin se alataso voi olla tietyssä mielessä deterministinen ja tarkoin määriteltävissä eli "aktuaalinen" kun taas se "ylempi" taso aistihavaintojen kannalta näyttäytyy aina jotenkin epämääräisenä ja potentiaalisena."

        Ymmärtänen hyvin, mitä tuolla tarkoitat, mutta itse en usko, että mikään "todellisuuden taso" tai osa olisi oikeasti muita "determinoivammassa roolissa", vaan tuo vaikutelma on vain seurausta tavasta mallintaa todellisuutta. On tietenkin aivan luonnollista, että kompleksisemmilla järjestelmillä voidaan mieltää olevan enemmän ominaisuuksia kuin vähemmän kompleksisilla. En kuitenkaan usko, että todellisuudesta voidaan poistaa mitään osia; muuttamatta todellisuutta, tai edes siihenkään, että niitä ylipäätäänkään voitaisiin poistaa...

        "Minua jostain syystä miellyttävät ja kiinnostavat kaikenlaiset avoimet ja vähän epämääräisemmät systeemit ja ideat joista voi kehittyä jotain uutta ja mielenkiintoista juuri nimenomaan sen takia että ne eivät ole etukäteen lukkoon lyötyjä vaan ainakin osittain ominaisuuksiltaan vielä tuntemattomia ja edellyttävät ehkä uusia analogioita."

        Tuntematon tosiaan kiehtoo ihmisiä ja hyvä niin, mutta jos toisaalta pyritään aina pitämään "kaikki kannat ns. avoimina", niin ilmeisesti hintana on se, että ei ehditä edetä niin pitkälle johonkin *tiettyyn* suuntaan; tutkimaan uusia tuntemattomia asioita, kuin mikä muuten olisi mahdollista. Kykysi ylläpitää epämääräistä kantaa on myös epätavallinen ja hyvin harvoilla käsittääkseni on tuollaiseen edellytyksiä ja/tai taipumustakaan.

        XPR-0008

        Nyt tarvitaan sitten vähän pidempiä selityksiä jotta asia tuli ehkä paremmin ymmärretyksi ja samalla siirrytään tais vaihteeksi aihepiireihin jotka joillekin voivat olla hyvinkin vaikeita hahmottaa.

        Ei se minun näennäinen epämääräisyyteni ole mikään itsetarkoitus vaan kyse on lähinnä siitä että siinä keskustelun alkup. viitekehyksessä on hankala esittää asioita sekä lyhyesti että täsmällisesti samanaikaisesti. Melkein jokaisesta asian aspektista pitäisi kirjoittaa vähintään monisatasivuinen kirja.

        Samalla täytyy myös aika radikaalisti rukata ja yhteensovittaa uudella tavalla nykytieteen toisiinsa nivoutuneiden valistuneiden arvausten verkostoa nykyisestä valtavirrasta poikkeavaksi eli käytännössä poisoppia melkein kokonaan se mitä pidetään nykyään tieteellisenä maailmankuvana koska kyse on suurimmaksi osaksi nimenomaan sellaisista asioista jotka eivät (vielä) oikein mahdu siihen maailmankuvaan jonka suurin osa on omaksunut tieteellisenä totuutena.

        "että noiden osien ja myös muiden osien välillä on vuorovaikutusta, mutta en siis usko, että *reaalinen* vuorovaikutus voisi olla epämääräistä... "

        Konkreettinen yksittäinen vuorovaikutus ei tietenkään ole sellaisenaan epämääräinen varsinkaan fysikaalisella tasolla mutta toisaalta sen vuorovaikutuksen määrä, laatu ja intensiteetti voi vaihdella ajan ja paikan mukaan eli jos puhutaan teoreettisella tasolla joidenkin vuorovaikutustapojen ominaisuuksista niin niissä voi olla sen verran variaatiota että niistä voi olla vaikeaa tai jopa mahdotonta vetää sellaisia yleisiä johtopäätöksiä jotka olisivat oikeasti niin tarkkoja että ne sallisivat sellaisten vuorovaikutusten ennustamisen kovin suurella tarkkuudella vaan kyse on todennäköisyyksistä ja tilastollisuudesta.

        Vaikka minusta kaikki luonnossa esiintyvät systeemit ovat avoimia systeemejä niin siitä ei pidä binäärisesti päätellä että on olemassa vain joko täysin avoimia tai täysin suljettuja systeemejä vaan että tietyt systeemit (ne yksilölliset) ovat avoimia vain joidenkin tiettyjen rajapintojen kautta ja muuten käytännössä lähes suljettuja (kuten esim. fysiikan ja kemian tason keho verrattuna nonlokaaliin tietoiseen mieleen jolloin se kehon lokaali biologisuus (hermosto,dna=vastaanotin & käyttöliittymä) toimii vuorovaikutuksen välittäjänä)

        Sitten vähän esoteerisemmalta kannalta niin oikeastaan ihmisellä on ns. kuudes aisti joka ei ole millään tavalla yliluonnollinen vaan hyvinkin luonnollinen kyky aistia ympäristön "tunnelmaa" (ambience) eli tietyissä paikoissa, tiettyinä aikoina, tietyt yksilöt tuottavat selkeästi omanlaisensa tunnelman aivan kuten eri aikakausina on huomattavissa erilainen ajanhenki (erilaiset suuntaukset, kulttuuri jne) ja varsinkin vanhoissa rakennuksissa on jotenkin aistittavissa niiden historia eli niiden ambience on täysin erilainen kuin muutaman kymmenen vuoden aikana rakennetussa arkkitehtuurissa jne.

        On paikkoja joista ikäänkuin huokuu rauhallisuus ja hyvä olo ja taas vastaavasti täysin toisenlaisen negatiivisen tunnelman tuottavia paikkoja ja tietysti myös suhteellisen neutraaleja tunnelmia tuottavia.

        Ehkä voisi jopa ajatella että jonkinlainen "ajan & paikan henki" tuottaa erilaiset tunnistettavissa olevat kulttuurit, kielet, geneettiset ja epigeneettiset variaatiot eliöistä.

        "Toisaalta, esim. ihmisen tapauksessa, kyllähän muutoksia tapahtuu myös persoonallisuuden tasolla, *jos* todellisuus niihin pakottaa, eli ihmisen mielessä on keskimäärin joitakin melko pysyviä osia, mutta lopultahan kaikki kuitenkin muuttuu. Ilmeisesti ihmisen muistin jatkuva toiminta on edellytys sille, että kokemuksellisten sisältöjen välillä koetaan olevan jatkuvuutta."

        Kyllä mutta ihmisen tapauksessa sillä persoonallisuudella on selkeästi erilaisia tasoja (esim. automaattinen, rutiininomainen ja ehdollistettu taso joka muistuttaa eniten konetta ja sitten tarkkaavaisuuden taso ja harvinaisemmat poikkeukselliset tietoisuuden tilat).

        Pakolla ja ehdollistamisella voidaan vaikuttaa siihen automaattiseen tasoon jolloin yksilön kehitys saattaa pysähtyä tai jumittua joksikin aikaa ja sitten ottaa huomioon C.G. Jungin yksilöllistymisprosessin jonka päämäärä ja tarkoitus on nimenomaan realisoida se oma ainutlaatuinen yksilöllisyytensä jonka muut voivat kokea sen ambience-aistin kautta.

        Esim. hypnoosin avulla voidaan myös ottaa se kehon käyttöliittymä ja automaattiset toiminnot jonkun toisen tietoisuuden hallintaan.

        Ei se pelkkä muisti selitä tuota yksilöllisyyttä koska muistinsa väliaikaisesti menettäneet ovat edelleen se sama yksilöllinen persoona jolla on se oma yksilöllinen "sormenjäljen" kaltainen tunnistettavissa oleva tunnelmansa.

        "niillä on tietty vuorovaikutus ajan funktiona muiden alkeishiukkasten jne. kanssa .."

        https://en.wikipedia.org/wiki/Identical_particles

        (jatkuu)


      • Anonyymi
        Anonyymi kirjoitti:

        XPR-0008

        Nyt tarvitaan sitten vähän pidempiä selityksiä jotta asia tuli ehkä paremmin ymmärretyksi ja samalla siirrytään tais vaihteeksi aihepiireihin jotka joillekin voivat olla hyvinkin vaikeita hahmottaa.

        Ei se minun näennäinen epämääräisyyteni ole mikään itsetarkoitus vaan kyse on lähinnä siitä että siinä keskustelun alkup. viitekehyksessä on hankala esittää asioita sekä lyhyesti että täsmällisesti samanaikaisesti. Melkein jokaisesta asian aspektista pitäisi kirjoittaa vähintään monisatasivuinen kirja.

        Samalla täytyy myös aika radikaalisti rukata ja yhteensovittaa uudella tavalla nykytieteen toisiinsa nivoutuneiden valistuneiden arvausten verkostoa nykyisestä valtavirrasta poikkeavaksi eli käytännössä poisoppia melkein kokonaan se mitä pidetään nykyään tieteellisenä maailmankuvana koska kyse on suurimmaksi osaksi nimenomaan sellaisista asioista jotka eivät (vielä) oikein mahdu siihen maailmankuvaan jonka suurin osa on omaksunut tieteellisenä totuutena.

        "että noiden osien ja myös muiden osien välillä on vuorovaikutusta, mutta en siis usko, että *reaalinen* vuorovaikutus voisi olla epämääräistä... "

        Konkreettinen yksittäinen vuorovaikutus ei tietenkään ole sellaisenaan epämääräinen varsinkaan fysikaalisella tasolla mutta toisaalta sen vuorovaikutuksen määrä, laatu ja intensiteetti voi vaihdella ajan ja paikan mukaan eli jos puhutaan teoreettisella tasolla joidenkin vuorovaikutustapojen ominaisuuksista niin niissä voi olla sen verran variaatiota että niistä voi olla vaikeaa tai jopa mahdotonta vetää sellaisia yleisiä johtopäätöksiä jotka olisivat oikeasti niin tarkkoja että ne sallisivat sellaisten vuorovaikutusten ennustamisen kovin suurella tarkkuudella vaan kyse on todennäköisyyksistä ja tilastollisuudesta.

        Vaikka minusta kaikki luonnossa esiintyvät systeemit ovat avoimia systeemejä niin siitä ei pidä binäärisesti päätellä että on olemassa vain joko täysin avoimia tai täysin suljettuja systeemejä vaan että tietyt systeemit (ne yksilölliset) ovat avoimia vain joidenkin tiettyjen rajapintojen kautta ja muuten käytännössä lähes suljettuja (kuten esim. fysiikan ja kemian tason keho verrattuna nonlokaaliin tietoiseen mieleen jolloin se kehon lokaali biologisuus (hermosto,dna=vastaanotin & käyttöliittymä) toimii vuorovaikutuksen välittäjänä)

        Sitten vähän esoteerisemmalta kannalta niin oikeastaan ihmisellä on ns. kuudes aisti joka ei ole millään tavalla yliluonnollinen vaan hyvinkin luonnollinen kyky aistia ympäristön "tunnelmaa" (ambience) eli tietyissä paikoissa, tiettyinä aikoina, tietyt yksilöt tuottavat selkeästi omanlaisensa tunnelman aivan kuten eri aikakausina on huomattavissa erilainen ajanhenki (erilaiset suuntaukset, kulttuuri jne) ja varsinkin vanhoissa rakennuksissa on jotenkin aistittavissa niiden historia eli niiden ambience on täysin erilainen kuin muutaman kymmenen vuoden aikana rakennetussa arkkitehtuurissa jne.

        On paikkoja joista ikäänkuin huokuu rauhallisuus ja hyvä olo ja taas vastaavasti täysin toisenlaisen negatiivisen tunnelman tuottavia paikkoja ja tietysti myös suhteellisen neutraaleja tunnelmia tuottavia.

        Ehkä voisi jopa ajatella että jonkinlainen "ajan & paikan henki" tuottaa erilaiset tunnistettavissa olevat kulttuurit, kielet, geneettiset ja epigeneettiset variaatiot eliöistä.

        "Toisaalta, esim. ihmisen tapauksessa, kyllähän muutoksia tapahtuu myös persoonallisuuden tasolla, *jos* todellisuus niihin pakottaa, eli ihmisen mielessä on keskimäärin joitakin melko pysyviä osia, mutta lopultahan kaikki kuitenkin muuttuu. Ilmeisesti ihmisen muistin jatkuva toiminta on edellytys sille, että kokemuksellisten sisältöjen välillä koetaan olevan jatkuvuutta."

        Kyllä mutta ihmisen tapauksessa sillä persoonallisuudella on selkeästi erilaisia tasoja (esim. automaattinen, rutiininomainen ja ehdollistettu taso joka muistuttaa eniten konetta ja sitten tarkkaavaisuuden taso ja harvinaisemmat poikkeukselliset tietoisuuden tilat).

        Pakolla ja ehdollistamisella voidaan vaikuttaa siihen automaattiseen tasoon jolloin yksilön kehitys saattaa pysähtyä tai jumittua joksikin aikaa ja sitten ottaa huomioon C.G. Jungin yksilöllistymisprosessin jonka päämäärä ja tarkoitus on nimenomaan realisoida se oma ainutlaatuinen yksilöllisyytensä jonka muut voivat kokea sen ambience-aistin kautta.

        Esim. hypnoosin avulla voidaan myös ottaa se kehon käyttöliittymä ja automaattiset toiminnot jonkun toisen tietoisuuden hallintaan.

        Ei se pelkkä muisti selitä tuota yksilöllisyyttä koska muistinsa väliaikaisesti menettäneet ovat edelleen se sama yksilöllinen persoona jolla on se oma yksilöllinen "sormenjäljen" kaltainen tunnistettavissa oleva tunnelmansa.

        "niillä on tietty vuorovaikutus ajan funktiona muiden alkeishiukkasten jne. kanssa .."

        https://en.wikipedia.org/wiki/Identical_particles

        (jatkuu)

        "ja toisaalta ihmistenkin yksilöllisyys on ajallisesti rajattua."

        Oletko varma?

        "ottaen huomioon mystifioivan filosofiasi"

        Se epämääräisyys tulee minusta lähinnä siitä että meillä ei ole sellaista yhteistä kieltä ja siihen liittyviä käsitteellisiä työkaluja joiden avulla tällaisten hyvinkin hankalien asioiden käsittely olisi vähän helpompaa ja selkeämpää.

        "tuo vaikutelma on vain seurausta tavasta mallintaa todellisuutta. "

        On olemassa vähemmän sopivia ja toimivia malleja ja taas sellaisia malleja tai analogioita joista saa enemmän irti kunhan ei aseta sille ajattelulle varsinkaan etukäteen liian suuria esteitä ja rajoitteita kuten tuo superdeterminismi. Joskus aikaisemmin kirjoitit että tuskin mikään saa sinua muuttamaan kantaasi eikä varmaan saakaan jos pidät nykyistä kantaasi jollain tavalla saavutettuna etuna esim. ammatillisessa mielessä.

        "jos toisaalta pyritään aina pitämään "kaikki kannat ns. avoimina", niin ilmeisesti hintana on se, että ei ehditä edetä niin pitkälle johonkin *tiettyyn* suuntaan; tutkimaan uusia tuntemattomia asioita, kuin mikä muuten olisi mahdollista. Kykysi ylläpitää epämääräistä kantaa on myös epätavallinen ja hyvin harvoilla käsittääkseni on tuollaiseen edellytyksiä ja/tai taipumustakaan."

        Ei minulla kaikki kannat ole avoimia vaan ehkä jonkinlaisessa superpositiossa eli ainakin 3-5 lupaavaa ehdokasta jotka nekin ovat minusta aika pitkälle yhteensovitettavissa kun otetaan huomioon erilaiset toisiaan täydentävät hahmotustavat. Jätän myös sellaisen mahdollisuuden auki että jotain uutta ja vähintään yhtä lupaavaa voi vielä ilmaantua.

        Vertauskuvallisesti nykytieteessä on taipumus kaivaa kaikilla resursseilla vain yhdestä paikasta kerrallaan ja vielä ns. katuvalon alla koska niissä muissa mahdollisissa kaivuupaikoissa valaistus on heikompi. Vähän niin kuin malmin etsinnässä niin se yksi ja sama kaivuupaikka tuottaa sitten lopulta vähenevässä määrin jotain arvokasta ja ne muut mahdolliset kaivuupaikat jäävät sitten puutteellisesti tai kokonaan tutkimatta kaivajien enemmistön itsepäisyyden ja perinteen takia.

        Tietysti tulee ottaa myös huomioon se että nykyinen valtavirtatiede palvelee lähinnä ns. oligarkkien etuja monessakin mielessä eli se tieteen objektiivisuus on minusta aika pitkälle illuusiota.

        (jatkuu myöhemmin kun aikaa, inspiraatiota ja energiaa taas löytyy lisää...:D)

        Belisario


      • Anonyymi
        Anonyymi kirjoitti:

        Nimimerkki: Professori Realisti (XPR-0009)

        "Minusta matemaattisten asioiden olemassaolo ei siis mitenkään välttämättä edellytä niitä vastaavien fysikaalisten rakenteiden olemassaoloa (jos ymmärsin oikein ilmaisusi merkityksen)".

        Matematiikkaan liittyvät näkökantamme eivät näyttäisi juurikaan eroavan toisistaan. Nykyinen kantani ei ole sama kuin Tegmark:in, eli minusta tuollaista *vastaavuutta* ei tarvita, mutta fysikaaliset *edellytykset* silti tarvitaan, eli olisi mieletöntä puhua sellaisista matemaattisista rakenteista, joiden kuvaukset eivät voisi realisoitua. Tegmark lienee analysoinut sitä, että mitä tuollainen realisoituminen *edellyttää* paljonkin, mutta en ole ehtinyt tutustua siihen enempää, vaikka aihe tietysti kiinnostaakin...

        "Yleisesti ottaen mitä täsmällisempi ja aksiomaattisempi se kuvauskieli on niin sitä suljetumpi systeemi se samalla myös on eli se ei pysty käsittelemään sellaisia asioita joita sen systeemin aksioomeissa ei ole määritelty todellisiksi ja olemassaoleviksi."

        Päteekö myös omasta mielestäsi tältä osin ns. "nollasummapeli", eli että ei ole mitään keinoa lisätä "tarkoitetulla tavalla kommunikoitavissa olevan informaation määrää", lisäämättä viestin pituutta? Eli, joko ollaan täsmällisiä ja jotain jää viittaamatta tai ollaan epämääräisiä ja mahdollistetaan ei-tarkoitetut viestin tulkinnat.

        "Analogioiden ja vertauskuvien avulla päättely taas on jo luonnostaan avointa eli mahdollistaa tutkittavan asian tunnetuista aspekteista siirtymisen niiden tuntemattomiin aspekteihin ja ehkä sellaisenkin ajattelun voisi jotenkin formalisoida eksaktimmaksi esim. Leibzinin ja Chomskyn ideoiden ja jonkinlaisen topologian pohjalta jolloin tarkka minkä tahansa tilanteen missä tahansa viitekehyksessä analysointi olisi mahdollista riippumatta siitä minkä tieteiden reviireihin ne asiat perinteisesti kuuluvat."

        Tuo kuulostaa varsin mielenkiintoiselta idealta.

        XPR-0009


        "Päteekö myös omasta mielestäsi tältä osin ns. "nollasummapeli", eli että ei ole mitään keinoa lisätä "tarkoitetulla tavalla kommunikoitavissa olevan informaation määrää", lisäämättä viestin pituutta? Eli, joko ollaan täsmällisiä ja jotain jää viittaamatta tai ollaan epämääräisiä ja mahdollistetaan ei-tarkoitetut viestin tulkinnat."

        Tottakai se informaation määrä voi lisääntyä ja lisääntyy mutta sitten toisaalta se informaatio suljetussa systeemissä on aina laadultaan tietynlaista kuten esim. shakkipelissä ei voi syntyä uudenlaisia siirtoja koska pelit säännöt ovat lopulliset ja valmiit.

        Belisario


      • Anonyymi
        Anonyymi kirjoitti:

        Nimimerkki: Professori Realisti (XPR-0010)

        Belisario: "Minusta olet turhan sitoutunut jonkinlaiseen koneparadigmaan..."

        Todellisuus pakottaa minut ulkoistamaan kuvitteellisen satunnaisuuden systeemistäni...

        "eli ts. nonlokaali tietoisuus tuottaa aineellisen maailman järjestyksen jonkinlaisen lomittumisen tai analogisen epälineaarisen prosessin kautta (vrt. sympathetic magic renesanssiajan alkemistien teoksissa)"

        Tarkoitatko, että se tosiaan tuottaa tuon *järjestyksen*, vaiko vain sen *kuvauksen"? En usko ollenkaan tuollaiseen, vaan mm. antrooppisen periaatteen ns. heikkoon muotoon, eli siihen että havaitsemisen ja tietoisuuden sisältöjen ja niiden edellytysten välillä vallitsee ilmeinen korrelaatio.

        "Ei vaan se [potentiaali] on aivan yhtä konkreettinen ellei jopa todellisempi kuin se mitä kutsumme aineelliseksi todellisuudeksi."

        Tullaanko tässä siihen, että käytät termiä "potentiaali" jossakin sellaisessa merkityksessä, että tuota merkitystä ei voida kommunikoida täsmällisesti, vai mistä tässä on kyse? Eli, yleisesti tunnettu kommunikaatio-ongelmahan on se, että ns. mystisiä kokemuksia ei välttämättä kyetä kuvaamaan niin hyvin, että kommunikaatio onnistuisi. Eli, jos esim. kahdella henkilöllä on kummallakin samankaltaisia mystisiä kokemuksia, niin he voivat kylläkin ymmärtää toisiaan *viittaamalla* samankaltaisiin kokemuksiin, mutta jos tuollaista yhteistä kokemuspohjaa ei ole, niin kielellinen kommunikaatio ei onnistu.

        "Toinen vaihtoehto on sitten olla ontologisesti sitoutumaton tai käyttää useampia näennäisesti toisiinsa nähden ristiriitaisia ontologioita samanaikaisesti..."

        Kuten aiemmin olen todennutkin, minun rajanani on (vain) mahdottomuus, mutta siltä osin en tunne epämääräisyyttä. :D

        "Kannattaa siis välttää kaikkea dogmaattisuutta ja sellaisia kuvitelmia että kaikki olennaiset perusasiat tieteessä muka jo tunnetaan ja että jäljellä on enää vain tarkkuuden kasvattaminen."

        Kannatatko oikeasti tuota dogmia, vai oletko vain kannattavinasi sitä? :D

        "Kaikenkaikkiaan minusta näytttää siltä että todellisuus on paljon monimuotoisempi ja oudompi kuin superdeterminismi ja kvanttifysiikan deterministinen monimaailmatulkinta mahdollistaa eikä sellainen malli ole minusta edes kovin kiinnostava koska se johtaa minusta aika selkeään umpikujaan."

        Multiversumin tulkinta umpikujaksi on hyvin outo näkemys, koska multiversumihan toteuttaa *väistämättä* *kaikki* mahdollisuudet...

        "Minun näkemykseni voi tuntua sinusta täysin käsittämättömältä mutta minä kylllä varsin hyvin ymmärrän sinun näkemyksesi ja se muistuttaa minusta jonkinlaista uskonnollista ja hyvin dogmaattista fundamentalismia esim. tuon superdeterminismin ja deterministisen monimaailmatulkinnan suhteen"

        Fundamentalismia tuskin on se, että ei usko ilmeisen mahdottomiin asioihin.

        "Minusta ei kannata sitoutua mihinkään malliin tutkimusten ollessa vielä monessa asiassa täysin alkutekijöissään."

        Mutta olemmeko esim. *me* enää alkutekijöissämme, eli onko aikaa olla sitoutumatta? Eli, ehkä pääero "filosofioissamme" on se, että en usko, että optimaalinen toimintatapa on odottaa varman tiedon ilmaantumista sellaisten asioiden osalta, joiden osalta sitä ei välttämättä koskaan ilmaannukaan. Eli, tietenkin tuo tarkoittaa riskin ottamista, mutta jos esim. kokemusten perusteella muotoutunut ns. sisäinen vakaumus on se, että indeterminismi on mahdotonta, niin miksi loppuelämänsä pitäisi viettää pähkäilemällä tuota asiaa?

        "En myöskään oikein ymmärrä miksi pidät minun kirjoitteluani jotenkin laadukkaana vaikka ilmeisesti olet usein täysin vastakkaista mieltä asioista."

        Kyllähän kirjoitusta laadukkaana voi pitää, vaikka ei olisikaan samaa mieltä sen sisällöstä. Kirjoitat hyvällä suomenkielellä ja tuot esille mielenkiintoisia näkemyksiä, vaikka en tietenkään hurahdakaan niihin kaikkiin... Lisäksi "näkemisen ääni" kertoo minulle usein, että olet älykäs, joten kirjoituksissasi täytyy olla *jotain* järkeä. :D

        "Aikaisemmin vuosia sitten käytin vaihtuvia nikkejä joista "Mandrake"(n haamu) lienee yleisin..."

        Käsittääkseni olemme itseasiassa molemmat mystikoita. Sinä olet käsittääkseni ns. "epämääräinen mystikko" ja minä "deterministinen mystikko". Nuo ovat mystikoiden koulukuntia vähän samaan tapaan kuin kung fu -koulut Kiinassa. Vähän olen kuitenkin havainnut sellaista tendenssiä, että oletat, että olen reduktionisti, kaiketi lähinnä sillä perusteella, että pidän täsmällisyyttä mm. kommunikaation kannalta tavoiteltavana asiana, eli vähän samaan tapaan kuin, jos joku on aina bongannut vain mustia variksia, niin jos sitten vastaan tuleekin valkoinen varis, niin sitä vaan itsepintaisesti väittää joutseneksi tms., koska valkoinen varis ei ole ennestään tuttu asia. :D

        [Kirjoittamisiini tässä ketjussa voi nyt tulla enemmän tai vähemmän taukoa, sillä aion vähän välillä tutkiskella erilaisten kvanttimekaanisten tulkintojen ihmeellistä maailmaa...]

        XPR-0010

        "Tarkoitatko, että se tosiaan tuottaa tuon *järjestyksen*,... "

        Se tuottaa sekä järjestyksen että kuvauksen ja niiden välillä on takaisinkytkentä eli se millaisena koemme todellisuuden moduloi myös niitä tapoja joilla muokkaamme sitä todellisuutta.

        "Tullaanko tässä siihen, että käytät termiä "potentiaali"...."

        Ei oikeastaan vaan viittaan joihinkin aikaisempiin vieteihini missä kirjoitin ns. tyhjiöstä jonka sisäinen rakenne (ns. nollasummavektorit joissa niitä toisiaan tasapainottavia ja nollaavia sähkömagneettisia vektoreita voi olla enemmän kuin 2 ilman ylärajaa ja joka määrittelee mihin paikkoihin auringot, galaksit ja planeetat rakentuvat) on tietyssä mielessä informaatiota joka vaikuttaa kaikkialla ilman viiveitä (viiveet taas liittyvät sähkömagneettisiin herz-aaltoihin kun taas ns. tyhjiön rakenne muodostuu jonkinlaisista seisovista aalloista (standing waves) ja periaatteessa kaikki informaatio siitä mitä menneisyydessä on tapahtunut tai olisi voinut tapahtua tallentuu sinne nollasummavektoreiden muodostamaan rakenteeseen (akashic record) ja sama pätee myös tulevaisuuden mahdollisiin tapahtumiin.

        Niiden nollasummavektoreiden rakenne muodostuu toisiinsa viiveettä resonoivista harmoonisista oskillaattoreista jossa sen tyhjiön sisäisen "stressin" eli rakenteen fysikaalisesti olemassaolevat aktuaaliset seuraukset (auringot, planeetat) jne. keskinäisillä vuorovaikuksillaan (gravitaatio=nollasummevektoreiden rakenne) määrittelevät sen eksplisiittisen todellisuuden tapahtumien todennäköisyydet eri paikoissa eri aikoina (kuten astrologiassa joka on jäänne muinaisen supersivilisaation tieteestä joka perustui lähinnä harmoonisille analogisille resonansseille)

        Ja kuten delayed choice quantum eraser kokeessa niin sitä menneisyyttä voi muuttaa poistamalla sitä tapahtumaa koskeva informaatio eli kyseessä on siis non-lokaali systeemi jossa ei ole aikaa eikä paikkaa eli se on tavallaan jonkinlaista esitodellisuutta ja periaatteessa tietoisuus non-lokaalina systeeminä liittyy kiinteästi tuollaiseen tyhjiön käsitteeseen ja voi periaatteessa olla eräs tuon tyhjiön aspekteista.

        Se tyhjiö/dynaaminen eetteri on siis sama asia kuin se Bohmin implisiittinen järjestys ja se kokemamme aineellinen maailma taas on se eksplisiittinen järjestys ja niiden välillä on siis jatkuva takaisinkytkentä. Tämä on tietysti hyvin vahvasti spekulatiivista mutta periaatteessa voisi myös selittää monet sellaiset oudot ja yliluonnollisilta tuntuvat asiat mitä kutsumme paranormaaleiksi ilmiöiksi.

        Tämä ei tietenkään ole tieteellisin kokein ainakaan helposti verifioitavissa eikä falsifioitavissa mutta toisaalta ei sellaista laajaa tieteellistä teoriaa olekaan jonka kaikki komponentit olisivat falsifioitavissa ja/tai verifioitavissa ja kyseessä on muutenkin vain hahmotelma jonka olen koonnut ja hahmottanut lähinnä vain itselleni useammista eri lähteistä ja erityisesti J.P. Farrellin kirjoista.

        Kyse on siis lähinnä jonkinlaisesta Tesla-fysiikasta joka poikkeaa aika radikaalisti nykyisestä julkisesta kulutusfysiikasta ja johon liittyy myös kylmäfuusio ja homeopatia (joita molempia on tutkinut myös Nobel fyysikko Brian Josephson) ja Sheldraken muotokentät jotka minusta ovat selitettävissä nimenomaan tuolla tyhjiöidealla sekä hyvin suuri osa sellaisista esoteerisista opeista jotka liittyvät okkultismiin, magiaan, alkemiaan yms. Alkemistien Prima Materia ja Maailman sielu (Anima mundi) vastaa hyvin tällaista tyhjiön ideaa joka on vain modernisoitu versio samasta asiasta.

        https://en.wikipedia.org/wiki/Prima_materia

        https://en.wikipedia.org/wiki/Anima_mundi


        Olen tuollaista mallia pohtinut ja tutkinut suht tiiviisti vasta aika vähän aikaa mutta minusta se vaikuttaa varsin lupaavalta vaikkakin tietysti hyvinkin hullulta mutta minusta aidon todellisuuden selityksen pitääkin olla aika hullu ja ongelmaksi voi lähinnä muodostua se että se ei edelleenkään ole tarpeeksi hullu selittääkseen kaiken mahdollisen.

        Belisario


      • Anonyymi
        Anonyymi kirjoitti:

        XPR-0010

        "Tarkoitatko, että se tosiaan tuottaa tuon *järjestyksen*,... "

        Se tuottaa sekä järjestyksen että kuvauksen ja niiden välillä on takaisinkytkentä eli se millaisena koemme todellisuuden moduloi myös niitä tapoja joilla muokkaamme sitä todellisuutta.

        "Tullaanko tässä siihen, että käytät termiä "potentiaali"...."

        Ei oikeastaan vaan viittaan joihinkin aikaisempiin vieteihini missä kirjoitin ns. tyhjiöstä jonka sisäinen rakenne (ns. nollasummavektorit joissa niitä toisiaan tasapainottavia ja nollaavia sähkömagneettisia vektoreita voi olla enemmän kuin 2 ilman ylärajaa ja joka määrittelee mihin paikkoihin auringot, galaksit ja planeetat rakentuvat) on tietyssä mielessä informaatiota joka vaikuttaa kaikkialla ilman viiveitä (viiveet taas liittyvät sähkömagneettisiin herz-aaltoihin kun taas ns. tyhjiön rakenne muodostuu jonkinlaisista seisovista aalloista (standing waves) ja periaatteessa kaikki informaatio siitä mitä menneisyydessä on tapahtunut tai olisi voinut tapahtua tallentuu sinne nollasummavektoreiden muodostamaan rakenteeseen (akashic record) ja sama pätee myös tulevaisuuden mahdollisiin tapahtumiin.

        Niiden nollasummavektoreiden rakenne muodostuu toisiinsa viiveettä resonoivista harmoonisista oskillaattoreista jossa sen tyhjiön sisäisen "stressin" eli rakenteen fysikaalisesti olemassaolevat aktuaaliset seuraukset (auringot, planeetat) jne. keskinäisillä vuorovaikuksillaan (gravitaatio=nollasummevektoreiden rakenne) määrittelevät sen eksplisiittisen todellisuuden tapahtumien todennäköisyydet eri paikoissa eri aikoina (kuten astrologiassa joka on jäänne muinaisen supersivilisaation tieteestä joka perustui lähinnä harmoonisille analogisille resonansseille)

        Ja kuten delayed choice quantum eraser kokeessa niin sitä menneisyyttä voi muuttaa poistamalla sitä tapahtumaa koskeva informaatio eli kyseessä on siis non-lokaali systeemi jossa ei ole aikaa eikä paikkaa eli se on tavallaan jonkinlaista esitodellisuutta ja periaatteessa tietoisuus non-lokaalina systeeminä liittyy kiinteästi tuollaiseen tyhjiön käsitteeseen ja voi periaatteessa olla eräs tuon tyhjiön aspekteista.

        Se tyhjiö/dynaaminen eetteri on siis sama asia kuin se Bohmin implisiittinen järjestys ja se kokemamme aineellinen maailma taas on se eksplisiittinen järjestys ja niiden välillä on siis jatkuva takaisinkytkentä. Tämä on tietysti hyvin vahvasti spekulatiivista mutta periaatteessa voisi myös selittää monet sellaiset oudot ja yliluonnollisilta tuntuvat asiat mitä kutsumme paranormaaleiksi ilmiöiksi.

        Tämä ei tietenkään ole tieteellisin kokein ainakaan helposti verifioitavissa eikä falsifioitavissa mutta toisaalta ei sellaista laajaa tieteellistä teoriaa olekaan jonka kaikki komponentit olisivat falsifioitavissa ja/tai verifioitavissa ja kyseessä on muutenkin vain hahmotelma jonka olen koonnut ja hahmottanut lähinnä vain itselleni useammista eri lähteistä ja erityisesti J.P. Farrellin kirjoista.

        Kyse on siis lähinnä jonkinlaisesta Tesla-fysiikasta joka poikkeaa aika radikaalisti nykyisestä julkisesta kulutusfysiikasta ja johon liittyy myös kylmäfuusio ja homeopatia (joita molempia on tutkinut myös Nobel fyysikko Brian Josephson) ja Sheldraken muotokentät jotka minusta ovat selitettävissä nimenomaan tuolla tyhjiöidealla sekä hyvin suuri osa sellaisista esoteerisista opeista jotka liittyvät okkultismiin, magiaan, alkemiaan yms. Alkemistien Prima Materia ja Maailman sielu (Anima mundi) vastaa hyvin tällaista tyhjiön ideaa joka on vain modernisoitu versio samasta asiasta.

        https://en.wikipedia.org/wiki/Prima_materia

        https://en.wikipedia.org/wiki/Anima_mundi


        Olen tuollaista mallia pohtinut ja tutkinut suht tiiviisti vasta aika vähän aikaa mutta minusta se vaikuttaa varsin lupaavalta vaikkakin tietysti hyvinkin hullulta mutta minusta aidon todellisuuden selityksen pitääkin olla aika hullu ja ongelmaksi voi lähinnä muodostua se että se ei edelleenkään ole tarpeeksi hullu selittääkseen kaiken mahdollisen.

        Belisario

        Lyhyemmin ilmaistuna:

        yhtenäiskenttä=implisiittinen järjestys=dynaaminen eetteri=nonlokaali=informaatio="henki"

        hiukkaset,atomit,molekyylit =eksplisiittinen järjestys=yhtenäiskentän eksitaatioita=lokaali=aine

        eliöt(biologiset, epäorgaanisetkuten esim. "plasmaeliöt")koostuvat sekä implisiittisestä että eksplisiittisistä komponteista joista se ekplsiittinen on lokaalia ja mittavissa ja se implisiittinen komponentti nonlokaalia eikä ainakaan helposti mitattavissa.

        Belisario


      • Anonyymi
        Anonyymi kirjoitti:

        Lyhyemmin ilmaistuna:

        yhtenäiskenttä=implisiittinen järjestys=dynaaminen eetteri=nonlokaali=informaatio="henki"

        hiukkaset,atomit,molekyylit =eksplisiittinen järjestys=yhtenäiskentän eksitaatioita=lokaali=aine

        eliöt(biologiset, epäorgaanisetkuten esim. "plasmaeliöt")koostuvat sekä implisiittisestä että eksplisiittisistä komponteista joista se ekplsiittinen on lokaalia ja mittavissa ja se implisiittinen komponentti nonlokaalia eikä ainakaan helposti mitattavissa.

        Belisario

        Vähän tarkempi kuvaus:

        Maxwellin alkup. yhtälöt sekä E.T. Whittakerin oivallukset yhdistettynä myöhempiin David Bohmin ja Thomas Beardenin ideoihin ovat hyvä ehdokas sellaiseksi kaiken teoriaksi joka kykenee yhdistämään sekä sähkömagnetismin että gravitaation samaan malliin ja jopa siten että se gravitaatio on periaatteessa manipuloitavissa paikallisesti laboratorio-olosuhteissa eli aika-avaruuden paikallinen manipulointi on mahdollista.

        Whittakerin wikiartikkelin linkeistä löytyy linkki pdf tiedostoon jonka lopussa Whittaker selkeästi ilmaisee että gravitaation nopeus on rajallinen mutta voi olla äärimmäisen paljon suurempi kuin valonnopeus (mikä voisi selittää ns. lomittumisilmiön deterministisesti eli kyseessä olisi piilomuuttujamalli ja siis jonkinlainen superdeterminismi joka ei kuitenkaan sulje pois ns. vapaata valintaa koska se vapaa valinta on ikäänkuin jo sisäänrakennettu siihen dynaamiseen eetteriin vaikka se ekpslisiittinen todellisuus onkin deterministinen:D)

        1903: "On the partial differential equations of mathematical physics."

        Whittaker on myös kirjoittanut laajan teoksen eetterin ja sähkämagnetismin historiasta ja ko. teos löytyy Archive.org sivustolta:

        "A history of the theories of aether and electricity : from the age of Descartes to the close of the nineteenth century" (1910)

        https://archive.org/details/historyoftheorie00whitrich

        https://en.wikipedia.org/wiki/E._T._Whittaker

        https://www.enterprisemissions.com/hyper2.html

        Tällä ylläoolevan linkin sivustolla on aika hyvä kooste tästä ideasta joka liittyy myös ns. dynaamiseen eetteriin jolla sattuu myös olemaan joitakin hyvin pitkälle samankaltaisia ominaisuusaspekteja kuin nykyisen kosmologian pimeällä aineella ja pimeällä energialla.

        Myös ns. Aharanov-Bohm effekti viittaa aika selkeästi jonkinlaisen eetterin olemassaoloon.

        https://en.wikipedia.org/wiki/Aharonov–Bohm_effect

        Aiheeseen kiinteästi liittyy myös Gabriel Kron:

        https://quantumantigravity.wordpress.com/gabriel-kron/

        Nykyisessä muodossaan sekä suhteellisuusteoriat että kvanttifysiikka johtavat teoreettiseen umpikujaan.

        Maailmankaikkeus on pohjimmiltaan itseorganisoituva avoin systeemi (suhteessa eetteriin tai kvanttityhjiöön) jossa tapahtuu jatkuvaa spontaania itseorganisoitumista eli kaikkeus ei ole mikään kone joka alkuräjähdyksen jälkeen vääjäämättä lähestyy jonkinlaista lämpökuolemaa termodynamiikan 2. pääsäännön mukaan (joka koskee vain suljettuja systeemejä) vaan organismin kaltainen kokonaisuus joka pystyy ylläpitämään ja lisäämään järjestystään ns. nollapiste-energian avulla periaatteessa loputtomiin vaikkakin jonkinlaisia leposyklejä voi esiintyä (vrt. Hindukosmologian Brahman yö)

        https://en.wikipedia.org/wiki/Hindu_cosmology

        Joseph P. Farrellin fysikaalis-teologis-esoteerinen topologinen vertauskuva liittyy myös tähän dynaamiseen eetteriin ja sillä on Farrellin mielestä hyvin laajakantoiset seuraukset varsinkin maailmantalouteen joka toistaiseksi on mallinnettu suljettuna systeeminä.
        Netistä löytyi aiheesta tällainen linkki (itse olen perehtynyt Farrellin hyvin laajaan ja monipuoliseen kirjalliseen tuotantoon jossa hän soveltaa omaperäistä analogisen ajattelun malliaan joka mahdollistaa luontevan poikkitieteellisen siirtymisen viitekehyksestä toiseen.

        https://erenow.net/common/financial-vipers-of-venice-alchemical-money-magical-physics-banking-middle-ages-renaissance/2.php

        Tässä lyhyesti koottuna kannattamaani minusta hyvin lupaavan kaiken teoriaan liittyviä asioita n. parin päivän melko intensiivisen googlauksen lopputuloksena. :D

        Tutkimukset jatkuvat....

        Belisario


      • Anonyymi
        Anonyymi kirjoitti:

        Vähän tarkempi kuvaus:

        Maxwellin alkup. yhtälöt sekä E.T. Whittakerin oivallukset yhdistettynä myöhempiin David Bohmin ja Thomas Beardenin ideoihin ovat hyvä ehdokas sellaiseksi kaiken teoriaksi joka kykenee yhdistämään sekä sähkömagnetismin että gravitaation samaan malliin ja jopa siten että se gravitaatio on periaatteessa manipuloitavissa paikallisesti laboratorio-olosuhteissa eli aika-avaruuden paikallinen manipulointi on mahdollista.

        Whittakerin wikiartikkelin linkeistä löytyy linkki pdf tiedostoon jonka lopussa Whittaker selkeästi ilmaisee että gravitaation nopeus on rajallinen mutta voi olla äärimmäisen paljon suurempi kuin valonnopeus (mikä voisi selittää ns. lomittumisilmiön deterministisesti eli kyseessä olisi piilomuuttujamalli ja siis jonkinlainen superdeterminismi joka ei kuitenkaan sulje pois ns. vapaata valintaa koska se vapaa valinta on ikäänkuin jo sisäänrakennettu siihen dynaamiseen eetteriin vaikka se ekpslisiittinen todellisuus onkin deterministinen:D)

        1903: "On the partial differential equations of mathematical physics."

        Whittaker on myös kirjoittanut laajan teoksen eetterin ja sähkämagnetismin historiasta ja ko. teos löytyy Archive.org sivustolta:

        "A history of the theories of aether and electricity : from the age of Descartes to the close of the nineteenth century" (1910)

        https://archive.org/details/historyoftheorie00whitrich

        https://en.wikipedia.org/wiki/E._T._Whittaker

        https://www.enterprisemissions.com/hyper2.html

        Tällä ylläoolevan linkin sivustolla on aika hyvä kooste tästä ideasta joka liittyy myös ns. dynaamiseen eetteriin jolla sattuu myös olemaan joitakin hyvin pitkälle samankaltaisia ominaisuusaspekteja kuin nykyisen kosmologian pimeällä aineella ja pimeällä energialla.

        Myös ns. Aharanov-Bohm effekti viittaa aika selkeästi jonkinlaisen eetterin olemassaoloon.

        https://en.wikipedia.org/wiki/Aharonov–Bohm_effect

        Aiheeseen kiinteästi liittyy myös Gabriel Kron:

        https://quantumantigravity.wordpress.com/gabriel-kron/

        Nykyisessä muodossaan sekä suhteellisuusteoriat että kvanttifysiikka johtavat teoreettiseen umpikujaan.

        Maailmankaikkeus on pohjimmiltaan itseorganisoituva avoin systeemi (suhteessa eetteriin tai kvanttityhjiöön) jossa tapahtuu jatkuvaa spontaania itseorganisoitumista eli kaikkeus ei ole mikään kone joka alkuräjähdyksen jälkeen vääjäämättä lähestyy jonkinlaista lämpökuolemaa termodynamiikan 2. pääsäännön mukaan (joka koskee vain suljettuja systeemejä) vaan organismin kaltainen kokonaisuus joka pystyy ylläpitämään ja lisäämään järjestystään ns. nollapiste-energian avulla periaatteessa loputtomiin vaikkakin jonkinlaisia leposyklejä voi esiintyä (vrt. Hindukosmologian Brahman yö)

        https://en.wikipedia.org/wiki/Hindu_cosmology

        Joseph P. Farrellin fysikaalis-teologis-esoteerinen topologinen vertauskuva liittyy myös tähän dynaamiseen eetteriin ja sillä on Farrellin mielestä hyvin laajakantoiset seuraukset varsinkin maailmantalouteen joka toistaiseksi on mallinnettu suljettuna systeeminä.
        Netistä löytyi aiheesta tällainen linkki (itse olen perehtynyt Farrellin hyvin laajaan ja monipuoliseen kirjalliseen tuotantoon jossa hän soveltaa omaperäistä analogisen ajattelun malliaan joka mahdollistaa luontevan poikkitieteellisen siirtymisen viitekehyksestä toiseen.

        https://erenow.net/common/financial-vipers-of-venice-alchemical-money-magical-physics-banking-middle-ages-renaissance/2.php

        Tässä lyhyesti koottuna kannattamaani minusta hyvin lupaavan kaiken teoriaan liittyviä asioita n. parin päivän melko intensiivisen googlauksen lopputuloksena. :D

        Tutkimukset jatkuvat....

        Belisario

        Palaan nyt takaisin otsikon varsinaiseen aiheeseen eli "MItä on olemassaolo?"
        Kaikki eksplisiittinen on olemassaolevaa ja kaikki implisiittinen ei ole olemassaolevaa mutta silti todellista eli todellisuus on laajempi käsite kuin olemassaolo.

        Engl. kielinen sana "nothing" kuvaa tätä aika hyvin eli " no thing"=ei esine,ei konkreettinen asia ja sen vastakohta "something" eli "some thing"=jokin konkreeettinen asia

        Konkreettinen= mitattavissa ja havaittavissa oleva (ilman tulkintoja ja metakognitiota) =deterministinen vaikka tieteen kannalta tilastollinen useimmissa tapauksissa

        Tulkinnat ja metakognitiot (teoriat yms) taas kuuluvat ei-olemassaolevaan maailmaan eli voivat vaikuttaa epätasapainossa olevan itseorganisoituvan systeemin (implisiittisen järjestyksen aktualisoija) käyttäytymiseen eli ovat todellisia mutta ei olemassaolevia edellä olevan määritelmän mukaan.

        Ihan samalla tavalla fysiikkatieteessä tunnetaan potentiaalinen energia ja kentät (magneetti-, gravitaatio, spekulatiivinen Higgs kenttä joka on oikeastaan vain uusi versio eetteristä) jotka eivät ole olemassaolevia mutta silti todellisia ja se olemassaolo liittyy taas siihen että potentiaalienergia muuttuu aktuaaliseksi energiaksi tai että kentästä "eksistoituu" hiukkasia.

        Tästä taas seuraa että esim. mieli ja tietoisuus ovat todellisia mutta eivät olemassaolevia samoin kuin matematiikka, logiikka, tieteen mallit & teoriat ja kaikki muutkin abstraktit asiat.

        Jossain mielessä eräs varhaiskeskiajan ortodoksinen kirkkoisä oli ehkä oikeilla jäljillä kun sanoi että jos Jumala on olemassa niin maailma ei voi olla olemassa ja jos maailma on olemassa niin Jumala ei voi olla olemassa:D (Lähde: Joseph P. Farrell)

        Itseorganisoituvat epätasapainossa olevat systeemit eli erilaiset organismit ovat rajapinta olemassaolevan fysikaalisen maailman ja olemattoman mutta silti todellisen implisiittisen todellisuuden välillä eli siinä määrin kuin se olio on vapautunut sen olemassaolevan maailman ehdollistumisesta niin sillä oliolla voi olla suhteellisen vapaa tahto eli ts. oliot voivat olla enemmän tai vähemmän ehdollistuneita implisiittisen todellisuuden ehdollistumisia.

        Belisario


      • Anonyymi
        Anonyymi kirjoitti:

        Palaan nyt takaisin otsikon varsinaiseen aiheeseen eli "MItä on olemassaolo?"
        Kaikki eksplisiittinen on olemassaolevaa ja kaikki implisiittinen ei ole olemassaolevaa mutta silti todellista eli todellisuus on laajempi käsite kuin olemassaolo.

        Engl. kielinen sana "nothing" kuvaa tätä aika hyvin eli " no thing"=ei esine,ei konkreettinen asia ja sen vastakohta "something" eli "some thing"=jokin konkreeettinen asia

        Konkreettinen= mitattavissa ja havaittavissa oleva (ilman tulkintoja ja metakognitiota) =deterministinen vaikka tieteen kannalta tilastollinen useimmissa tapauksissa

        Tulkinnat ja metakognitiot (teoriat yms) taas kuuluvat ei-olemassaolevaan maailmaan eli voivat vaikuttaa epätasapainossa olevan itseorganisoituvan systeemin (implisiittisen järjestyksen aktualisoija) käyttäytymiseen eli ovat todellisia mutta ei olemassaolevia edellä olevan määritelmän mukaan.

        Ihan samalla tavalla fysiikkatieteessä tunnetaan potentiaalinen energia ja kentät (magneetti-, gravitaatio, spekulatiivinen Higgs kenttä joka on oikeastaan vain uusi versio eetteristä) jotka eivät ole olemassaolevia mutta silti todellisia ja se olemassaolo liittyy taas siihen että potentiaalienergia muuttuu aktuaaliseksi energiaksi tai että kentästä "eksistoituu" hiukkasia.

        Tästä taas seuraa että esim. mieli ja tietoisuus ovat todellisia mutta eivät olemassaolevia samoin kuin matematiikka, logiikka, tieteen mallit & teoriat ja kaikki muutkin abstraktit asiat.

        Jossain mielessä eräs varhaiskeskiajan ortodoksinen kirkkoisä oli ehkä oikeilla jäljillä kun sanoi että jos Jumala on olemassa niin maailma ei voi olla olemassa ja jos maailma on olemassa niin Jumala ei voi olla olemassa:D (Lähde: Joseph P. Farrell)

        Itseorganisoituvat epätasapainossa olevat systeemit eli erilaiset organismit ovat rajapinta olemassaolevan fysikaalisen maailman ja olemattoman mutta silti todellisen implisiittisen todellisuuden välillä eli siinä määrin kuin se olio on vapautunut sen olemassaolevan maailman ehdollistumisesta niin sillä oliolla voi olla suhteellisen vapaa tahto eli ts. oliot voivat olla enemmän tai vähemmän ehdollistuneita implisiittisen todellisuuden ehdollistumisia.

        Belisario

        Uusi dokumenttielokuva David Bohmista:

        Infinite Potential: The Life and Ideas of David Bohm (kesto 1 tunti 11min)

        https://www.youtube.com/watch?v=XDpurdHKpb8

        Elokuvassa käsitellään David Bohmin elämää ja ideoita jotka koskettavat erityisesti olemassaoloa ja ns. nonlokaalia kvanttipotentiaalia joka määrittelee nonlokaalien piilomuuttujien (ns. viivettömät pilottiaallot) kvanttifysikaalisten hiukkasten ja sitä kautta myös makrofysikaalisten ilmiöiden käyttäytymisen.

        Bohmin piilomuuttujamallin perusteella voidaan ennustaa tarkasti myös elektronien käyttäytyminen kaksoisrakokeessa.

        Katsoin tuon juuri äsken ja voin suositella kaikille filosofiasta, fysiikasta ja todellisuuden kokonaisymmärtämisestä kiinnostuneille.
        .....

        Omia johtopäätöksiäni katsottuani ko. dokkarin:

        Silloin kun ihminen on ns. "aaltotilassa" eli mahdollisimman samaistumaton omaan kehoonsa, ajatuksiinsa, uskomuksiinsa ja tunteisiinsa niin hänellä on vapaan tahdon mahdollisuus kun taas kaikenlainen samaistuminen ja ehdollistuminen eli "hiukkastila" rajoittaa valinnan vapautta.

        Vaikka Bohmin malli onkin deterministinen niin se ei ole silti perinteinen deterministinen ja mekaaninen konemalli vaan holistinen ja orgaaninen malli jossa ihmisen tietoisuus implisiittisenä (aaltomoodi) järjestyksenä pyrkii mahdollisimman deterministisesti vaikuttamaan siihen aineelliseen eli ekplisiittiseen todellisuuteen (hiukkasmoodi) olematta itse kuitenkaan ainakaan täysin sen aineellisen todellisuuden ehdollistama.

        Itse viihdyn kaikkein parhaiten ns. "aaltotilassa" joka on helpointa saavuttaa yksin ja ilman häiritseviä vuorovaikutuksia ja sitten taas vain tarvittaessa siinä "hiukkasmoodissa" kun vuorovaikutan muiden kanssa. Aaltotilassa koen olevani nonlokaali ja vastaavasti hiukkastilassa selkeästi lokalisoituneena johonkin aikaan ja paikkaan eikä tässä ole kyse pelkästään vertauskuvasta vaan hyvinkin konkreettisesta asiasta.

        Aaltotilaa voidaan myös kutsua jonkinlaiseksi meditaatioksi koska siinä hiljenee sisäinen kielellinen yksinpuhelu tarkkaavaisuuden säilyessä samanaikaisesti. Tällaisessa meditatiivisessä tilassa olen usein herätessä suht. hyvin nukutun yön jälkeen tai ulkona varsinkin luonnossa kävellessä.

        Vastakkaisia varsin negatiivisia vaikutelmia syntyi aikoinaan koulussa ja työelämässä jossa koin usein olevani lähinnä jonkinlainen robotti jolloin kaikki tahdonvapauden ja valinnanmahdollisuuden kokemukset lähinnä loistivat poissaolollaan suurimman osan ajasta.

        Belisario


      • Anonyymi kirjoitti:

        Uusi dokumenttielokuva David Bohmista:

        Infinite Potential: The Life and Ideas of David Bohm (kesto 1 tunti 11min)

        https://www.youtube.com/watch?v=XDpurdHKpb8

        Elokuvassa käsitellään David Bohmin elämää ja ideoita jotka koskettavat erityisesti olemassaoloa ja ns. nonlokaalia kvanttipotentiaalia joka määrittelee nonlokaalien piilomuuttujien (ns. viivettömät pilottiaallot) kvanttifysikaalisten hiukkasten ja sitä kautta myös makrofysikaalisten ilmiöiden käyttäytymisen.

        Bohmin piilomuuttujamallin perusteella voidaan ennustaa tarkasti myös elektronien käyttäytyminen kaksoisrakokeessa.

        Katsoin tuon juuri äsken ja voin suositella kaikille filosofiasta, fysiikasta ja todellisuuden kokonaisymmärtämisestä kiinnostuneille.
        .....

        Omia johtopäätöksiäni katsottuani ko. dokkarin:

        Silloin kun ihminen on ns. "aaltotilassa" eli mahdollisimman samaistumaton omaan kehoonsa, ajatuksiinsa, uskomuksiinsa ja tunteisiinsa niin hänellä on vapaan tahdon mahdollisuus kun taas kaikenlainen samaistuminen ja ehdollistuminen eli "hiukkastila" rajoittaa valinnan vapautta.

        Vaikka Bohmin malli onkin deterministinen niin se ei ole silti perinteinen deterministinen ja mekaaninen konemalli vaan holistinen ja orgaaninen malli jossa ihmisen tietoisuus implisiittisenä (aaltomoodi) järjestyksenä pyrkii mahdollisimman deterministisesti vaikuttamaan siihen aineelliseen eli ekplisiittiseen todellisuuteen (hiukkasmoodi) olematta itse kuitenkaan ainakaan täysin sen aineellisen todellisuuden ehdollistama.

        Itse viihdyn kaikkein parhaiten ns. "aaltotilassa" joka on helpointa saavuttaa yksin ja ilman häiritseviä vuorovaikutuksia ja sitten taas vain tarvittaessa siinä "hiukkasmoodissa" kun vuorovaikutan muiden kanssa. Aaltotilassa koen olevani nonlokaali ja vastaavasti hiukkastilassa selkeästi lokalisoituneena johonkin aikaan ja paikkaan eikä tässä ole kyse pelkästään vertauskuvasta vaan hyvinkin konkreettisesta asiasta.

        Aaltotilaa voidaan myös kutsua jonkinlaiseksi meditaatioksi koska siinä hiljenee sisäinen kielellinen yksinpuhelu tarkkaavaisuuden säilyessä samanaikaisesti. Tällaisessa meditatiivisessä tilassa olen usein herätessä suht. hyvin nukutun yön jälkeen tai ulkona varsinkin luonnossa kävellessä.

        Vastakkaisia varsin negatiivisia vaikutelmia syntyi aikoinaan koulussa ja työelämässä jossa koin usein olevani lähinnä jonkinlainen robotti jolloin kaikki tahdonvapauden ja valinnanmahdollisuuden kokemukset lähinnä loistivat poissaolollaan suurimman osan ajasta.

        Belisario

        De Broglie–Bohm teoria on vain yksi mahdollinen tulkinta. Aalto-hiukkas dualismi on mielenkiintoinen, mikä osoittaa hyvin sen, että koko mikromaailma on täysin erilainen kuin me yleensä kuvittelemme.

        Aalto luonne on merkittävä vain mikro-maailmassa. Mitä isompi kohde sitä merkityksettömämpi siitä tulee. Muistaakseni suurimmat kappaleet, joilla aalto-luonne on saatu havaittua ovat yksittäisiä molekyylejä.

        Ei ole mitään perustetta kuvitella, että meditatiivisilla tiloilla olisi jotain tekemistä kvanttimekaanisten ilmiöiden kanssa. Erilaisia meditatiivisia tiloja ollaan tutkittu ja ne ovat havaittavissa aivojen neuroverkon aktiviteetissa.


      • Anonyymi
        Järkisyitä kirjoitti:

        De Broglie–Bohm teoria on vain yksi mahdollinen tulkinta. Aalto-hiukkas dualismi on mielenkiintoinen, mikä osoittaa hyvin sen, että koko mikromaailma on täysin erilainen kuin me yleensä kuvittelemme.

        Aalto luonne on merkittävä vain mikro-maailmassa. Mitä isompi kohde sitä merkityksettömämpi siitä tulee. Muistaakseni suurimmat kappaleet, joilla aalto-luonne on saatu havaittua ovat yksittäisiä molekyylejä.

        Ei ole mitään perustetta kuvitella, että meditatiivisilla tiloilla olisi jotain tekemistä kvanttimekaanisten ilmiöiden kanssa. Erilaisia meditatiivisia tiloja ollaan tutkittu ja ne ovat havaittavissa aivojen neuroverkon aktiviteetissa.

        "De Broglie–Bohm teoria on vain yksi mahdollinen tulkinta. Aalto-hiukkas dualismi on mielenkiintoinen, mikä osoittaa hyvin sen, että koko mikromaailma on täysin erilainen kuin me yleensä kuvittelemme. "

        Tässä kannattaa kyllä erottaa Bohmin varhaisvaihe eli Bohm-Broglie malli(50-luku) ja sitten Bohmin myöhäisvaihe eli "Wholeness And Implicate order" (80-luku) selkeästi toisitaan vaikka niissä onkinedelleen jonkin verran yhtäläisyyksiä. Toistaiseksi ei ole olemassa empiiristä keinoa erottaa kvanttifysikkan tulkintoja toisistaan ja niitä yleisemmin tunnettuja tulkintoja on jo toistakymmentä.

        Jos ns. mikromaailma on täysin erilainen kuin kuvittelemme niin myös sen makromaailman pitää olla vastaavalla tavalla hyvin erilainen kuin olemme perinteellisesti fysiikassa ja arkiymmärryksessä hahmottaneet.

        Katso siitä linkittämästäni Bohmin dokkarista kohdasta 51 min alusta eteenpäin niin siinä kerrotaan tietokonesimulaatiosta joka Bohmin mallin perusteella kykenee ennustamaan tarkasti mihin kohtiin kaksoisrakokokeen interferenssikuvio (aalto) muodostuu.

        Kyse on tietenkin tietokonesimulaatiosta joka ei korvaa mittauksia mutta tällainen deterministisempi malli periaatteessa voi mahdollistaa kvanttimaailman eli todellisuuden perustan hyvinkin pitkälle menevän manipulaation mitä ilmeisesti on jo harjoitettu eri maiden salaisissa tai esoteerisissa projekteissa jo ennen 2. maailmansotaa.
        Ks. esim. Hans Coler apparaatti jonka kerrotaan toimineen nollapiste-energialla ja jota kuulemma suunniteltiin natsien oman ufoprojektin voimanlähteeksi.

        https://www.researchgate.net/publication/251709903_Experiments_with_Coler_magnetic_current_apparatus

        "Aalto luonne on merkittävä vain mikro-maailmassa. Mitä isompi kohde sitä merkityksettömämpi siitä tulee. Muistaakseni suurimmat kappaleet, joilla aalto-luonne on saatu havaittua ovat yksittäisiä molekyylejä."

        Tuskin pitää paikkaansa. Koska makroskooppisilla kappaleilla on äärimmäisen lyhyt aallonpituus niin aalto-ominaisuuksia on niissä hyvin vaikea havaita. Ks. allaoleva wiki 4. rivi alusta.

        "For macroscopic particles, because of their extremely short wavelengths, wave properties usually cannot be detected.[3]"

        https://en.wikipedia.org/wiki/Wave–particle_duality

        "Ei ole mitään perustetta kuvitella, että meditatiivisilla tiloilla olisi jotain tekemistä kvanttimekaanisten ilmiöiden kanssa. Erilaisia meditatiivisia tiloja ollaan tutkittu ja ne ovat havaittavissa aivojen neuroverkon aktiviteetissa."

        Miksi ei olisi? Periaatteessa kaikki liittyy kaikkeen vaikka akateemisessa maailmassa todellisuuden tutkiminen onkin jaettu hyvin pieniin lokeroihin. Tottakai meditatiiviset tilat ovat todettavissa esim. aivokuvantamisen avulla mutta siitä on taas melkoinen uskonhyppy oletukseen että ne kausaalisesti johtuisivat hermoverkon rakenteesta varsinkin kun ylipäätänsä tietoisuutta ei ole kyetty selittämään aivojen tuotteena (ns. tietoisuuden vaikea ongelma). Kaikki mitä mittausvälineillä kyetään suoraan tutkimaaan on aina sitä eksplisiittistä järjestystä eli 3. persoonan ulkopuolinen näkökulma todellisuuteen.

        Minä tietenkin tarkoitin lähinnä jonkinlaista analogiaa tai vertauskuvaa mutta en sinänsä myöskään sulje pois oletusarvoisesti sellaista mahdollisuutta että kyse olisi nimenomaan täysin samasta asiasta kuin kvanttifysiikan ilmiöissä eli jos Bohmin malli pitää paikkansa niin sen eksplisiittisen (aineen) ja implisiittisen ("eetterin" tai 0-piste-energian tai "hengen" tai informaation) rajapinta sijaitsee nimenomaan kvanttimaailmassa eli tavallaan se implisiittinen järjestys simuloi sitä eksplisiittistä järjestystä eli se aineellinen eksplisiittinen maailma on sen eetterin tuottama simulaatio.

        Aina on hyvä kyseenalaistaa tai kysyä jos ei ymmärrä. Usein ihmettelen miksi ei kukaan vaivaudu kysymään vaikka aika usein kirjoitan tekstiä minkä ehdottomasti pitäisi aiheuttaa kyseenalaistamisen tai kysymisen tarvetta. Joskus sitten provoilen ihan tahallani vaikka itse pidän niitä provoilujakin ainakin jossain määrin perusteltavissa olevina vaikkakin samalla myös usein melko spekulatiivisina.


        Kiitos!

        Belisario


      • Anonyymi
        Anonyymi kirjoitti:

        "De Broglie–Bohm teoria on vain yksi mahdollinen tulkinta. Aalto-hiukkas dualismi on mielenkiintoinen, mikä osoittaa hyvin sen, että koko mikromaailma on täysin erilainen kuin me yleensä kuvittelemme. "

        Tässä kannattaa kyllä erottaa Bohmin varhaisvaihe eli Bohm-Broglie malli(50-luku) ja sitten Bohmin myöhäisvaihe eli "Wholeness And Implicate order" (80-luku) selkeästi toisitaan vaikka niissä onkinedelleen jonkin verran yhtäläisyyksiä. Toistaiseksi ei ole olemassa empiiristä keinoa erottaa kvanttifysikkan tulkintoja toisistaan ja niitä yleisemmin tunnettuja tulkintoja on jo toistakymmentä.

        Jos ns. mikromaailma on täysin erilainen kuin kuvittelemme niin myös sen makromaailman pitää olla vastaavalla tavalla hyvin erilainen kuin olemme perinteellisesti fysiikassa ja arkiymmärryksessä hahmottaneet.

        Katso siitä linkittämästäni Bohmin dokkarista kohdasta 51 min alusta eteenpäin niin siinä kerrotaan tietokonesimulaatiosta joka Bohmin mallin perusteella kykenee ennustamaan tarkasti mihin kohtiin kaksoisrakokokeen interferenssikuvio (aalto) muodostuu.

        Kyse on tietenkin tietokonesimulaatiosta joka ei korvaa mittauksia mutta tällainen deterministisempi malli periaatteessa voi mahdollistaa kvanttimaailman eli todellisuuden perustan hyvinkin pitkälle menevän manipulaation mitä ilmeisesti on jo harjoitettu eri maiden salaisissa tai esoteerisissa projekteissa jo ennen 2. maailmansotaa.
        Ks. esim. Hans Coler apparaatti jonka kerrotaan toimineen nollapiste-energialla ja jota kuulemma suunniteltiin natsien oman ufoprojektin voimanlähteeksi.

        https://www.researchgate.net/publication/251709903_Experiments_with_Coler_magnetic_current_apparatus

        "Aalto luonne on merkittävä vain mikro-maailmassa. Mitä isompi kohde sitä merkityksettömämpi siitä tulee. Muistaakseni suurimmat kappaleet, joilla aalto-luonne on saatu havaittua ovat yksittäisiä molekyylejä."

        Tuskin pitää paikkaansa. Koska makroskooppisilla kappaleilla on äärimmäisen lyhyt aallonpituus niin aalto-ominaisuuksia on niissä hyvin vaikea havaita. Ks. allaoleva wiki 4. rivi alusta.

        "For macroscopic particles, because of their extremely short wavelengths, wave properties usually cannot be detected.[3]"

        https://en.wikipedia.org/wiki/Wave–particle_duality

        "Ei ole mitään perustetta kuvitella, että meditatiivisilla tiloilla olisi jotain tekemistä kvanttimekaanisten ilmiöiden kanssa. Erilaisia meditatiivisia tiloja ollaan tutkittu ja ne ovat havaittavissa aivojen neuroverkon aktiviteetissa."

        Miksi ei olisi? Periaatteessa kaikki liittyy kaikkeen vaikka akateemisessa maailmassa todellisuuden tutkiminen onkin jaettu hyvin pieniin lokeroihin. Tottakai meditatiiviset tilat ovat todettavissa esim. aivokuvantamisen avulla mutta siitä on taas melkoinen uskonhyppy oletukseen että ne kausaalisesti johtuisivat hermoverkon rakenteesta varsinkin kun ylipäätänsä tietoisuutta ei ole kyetty selittämään aivojen tuotteena (ns. tietoisuuden vaikea ongelma). Kaikki mitä mittausvälineillä kyetään suoraan tutkimaaan on aina sitä eksplisiittistä järjestystä eli 3. persoonan ulkopuolinen näkökulma todellisuuteen.

        Minä tietenkin tarkoitin lähinnä jonkinlaista analogiaa tai vertauskuvaa mutta en sinänsä myöskään sulje pois oletusarvoisesti sellaista mahdollisuutta että kyse olisi nimenomaan täysin samasta asiasta kuin kvanttifysiikan ilmiöissä eli jos Bohmin malli pitää paikkansa niin sen eksplisiittisen (aineen) ja implisiittisen ("eetterin" tai 0-piste-energian tai "hengen" tai informaation) rajapinta sijaitsee nimenomaan kvanttimaailmassa eli tavallaan se implisiittinen järjestys simuloi sitä eksplisiittistä järjestystä eli se aineellinen eksplisiittinen maailma on sen eetterin tuottama simulaatio.

        Aina on hyvä kyseenalaistaa tai kysyä jos ei ymmärrä. Usein ihmettelen miksi ei kukaan vaivaudu kysymään vaikka aika usein kirjoitan tekstiä minkä ehdottomasti pitäisi aiheuttaa kyseenalaistamisen tai kysymisen tarvetta. Joskus sitten provoilen ihan tahallani vaikka itse pidän niitä provoilujakin ainakin jossain määrin perusteltavissa olevina vaikkakin samalla myös usein melko spekulatiivisina.


        Kiitos!

        Belisario

        Luonnontieteet kykenevät käsittelemään ainoastaan eksplisiittisiä asioita ja niihin kuuluu kaikki määrät jotka ovat aistivaikutelmien (=ekplisiittisen "maailman" hahmotus) aspektien (etäisyys, taajuus., voima, intensiteetti yms. mitattavissa tai oikeammin keskenään vertailtavissa olevia asioita ) suhteita toisiinsa ja ne ovat metrisiä ja/tai geometrisia mutta koska ne ovat vain sen eksplisiittisen virtuaalitodellisuuden hahmotuksia ja hahmotustapoja niin luonnontieteet ylipäätänsä ei kykene ainakaan nykyisessä muodossaan käsittelemään sitä perimmäistä implisiittistä todellisuutta jossa ei ole määriä eikä metristä geometriaa koska se on nonlokaali (=ei etäisyyksiä eikä määrien välisiä suhteita) ja se implisiittinen todellisuus ilmenee inhimillisessä kokemuksessa (joka on paljon laajempi kuin aistihavainnot) laatuina, resonansseina, synkronisiteettina, ideoina, tunteina, inspiraationa, etiikkana, moraalina, viisautena.

        Koska kvanttifysiikka käsittelee ilmiöitä jotka vaikuttavat aivan eksplisiittisen ja implisiittisen todellisuuden rajapinnassa niin sen ilmiöt voivat olla sekä non-lokaaleja että hyvin outoja (superpositiot, tunneloitumiset, hiukkas-aaltodualismi,lomittuminen) ja koska kaikki metriikka ja geometria liittyvät ainoastaan eksplisiittisen todellisuuden hahmotukseen ja aistikokemuksien aspektien välisiin vertailuihin niin kvanttifysiikan ilmiöitä on mahdotonta yhdistää eksplisiittisen geometrisen ja metrisen fysiikan (suhteellisuusteoriat ja Newtonin fysiikka) teorioihin ja malleihin jolloin suurin osa nykyisestä kvanttifysiikastakin on pielessä koska se yrittää mallintaa todellisuutta kvantin eli kuvitellun pienimmän mahdollisen etäisyyden ja vuorovaikutuksen pohjalta (Planckin vakio) eli yrittää määritellä sellaisia ominaisuuksia kuvitelluille alkeishiukkasille joita nonlokaalissa maailmassa ei voi esiintyä esim. senttimetrien murto-osien ja kuviteltujen "mittabosonien" ja sähkövarausten ja massan kautta jotka on päätelty tai kuviteltu kuplakammioiden kuvioiden tai hiukkaskiihdyttimien tilastollisten ilmiöiden (eräänlaisia Rossachin musteläiskiä).

        Alkeishiukkasia ovat pikseleitä eksplisiitisen virtuaalitodellisuuden "kuvaruudulla" on ja aallot ja kentät ovat vertauskuvia jotka viittaavat nonlokaaliin implisiittiseen todellisuuteen.

        Voiko tiede ylipäätänsä kertoa mitään sellaisesta todellisuudesta joka ei ole mitattavissa eikä laskettavissa kvantitatiivisesti eikä siis myöskään geometrisesti kuten suhteellisuusteorioissa?

        Voi ja kun metriikka ja geometria tiputetaan pois niin jäljelle jää vielä topologian käytön mahdollisuus. Luonnontieteet siis tutkivat lähinnä virtuaalitodellisuutta.


        Nyt tuli näköjään aika hegeliläistä hyvin koukeroista tekstiä jota on vaikea kirjoittaa ja varmaan vielä vaikeampaa ymmärtää koska yritin tietoisesti ja kieli keskellä suuta varovaisesti hahmotella jotenkin edes auttavasti ja eksplisiittisesti sellaista minkä voi kunnolla ymmärtää ainoastaan intuition ja vertauskuvien avulla koska olen huomannut että ns. luonnontieteellis-matemaattisesti orientoituneet yksilöt eivät ymmärrä vertauskuvia eikä analogioita vaan kaikki pitää vääntää hyvin yksiselitteisesti, kirjaimellisesti ja mahdollisimman eksaktisti hyvin paksulla rautalangalla:D

        Eikä siitä sittenkään varmaan kukaan mitään ymmärrä enkä varmaan itsekään ymmärtäisi jos aikaisempi minäni muutama kymmenen vuotta sitten lukisi tätä tekstiä. [ :D & :( ]

        Belisario


      • Anonyymi
        Anonyymi kirjoitti:

        Uusi dokumenttielokuva David Bohmista:

        Infinite Potential: The Life and Ideas of David Bohm (kesto 1 tunti 11min)

        https://www.youtube.com/watch?v=XDpurdHKpb8

        Elokuvassa käsitellään David Bohmin elämää ja ideoita jotka koskettavat erityisesti olemassaoloa ja ns. nonlokaalia kvanttipotentiaalia joka määrittelee nonlokaalien piilomuuttujien (ns. viivettömät pilottiaallot) kvanttifysikaalisten hiukkasten ja sitä kautta myös makrofysikaalisten ilmiöiden käyttäytymisen.

        Bohmin piilomuuttujamallin perusteella voidaan ennustaa tarkasti myös elektronien käyttäytyminen kaksoisrakokeessa.

        Katsoin tuon juuri äsken ja voin suositella kaikille filosofiasta, fysiikasta ja todellisuuden kokonaisymmärtämisestä kiinnostuneille.
        .....

        Omia johtopäätöksiäni katsottuani ko. dokkarin:

        Silloin kun ihminen on ns. "aaltotilassa" eli mahdollisimman samaistumaton omaan kehoonsa, ajatuksiinsa, uskomuksiinsa ja tunteisiinsa niin hänellä on vapaan tahdon mahdollisuus kun taas kaikenlainen samaistuminen ja ehdollistuminen eli "hiukkastila" rajoittaa valinnan vapautta.

        Vaikka Bohmin malli onkin deterministinen niin se ei ole silti perinteinen deterministinen ja mekaaninen konemalli vaan holistinen ja orgaaninen malli jossa ihmisen tietoisuus implisiittisenä (aaltomoodi) järjestyksenä pyrkii mahdollisimman deterministisesti vaikuttamaan siihen aineelliseen eli ekplisiittiseen todellisuuteen (hiukkasmoodi) olematta itse kuitenkaan ainakaan täysin sen aineellisen todellisuuden ehdollistama.

        Itse viihdyn kaikkein parhaiten ns. "aaltotilassa" joka on helpointa saavuttaa yksin ja ilman häiritseviä vuorovaikutuksia ja sitten taas vain tarvittaessa siinä "hiukkasmoodissa" kun vuorovaikutan muiden kanssa. Aaltotilassa koen olevani nonlokaali ja vastaavasti hiukkastilassa selkeästi lokalisoituneena johonkin aikaan ja paikkaan eikä tässä ole kyse pelkästään vertauskuvasta vaan hyvinkin konkreettisesta asiasta.

        Aaltotilaa voidaan myös kutsua jonkinlaiseksi meditaatioksi koska siinä hiljenee sisäinen kielellinen yksinpuhelu tarkkaavaisuuden säilyessä samanaikaisesti. Tällaisessa meditatiivisessä tilassa olen usein herätessä suht. hyvin nukutun yön jälkeen tai ulkona varsinkin luonnossa kävellessä.

        Vastakkaisia varsin negatiivisia vaikutelmia syntyi aikoinaan koulussa ja työelämässä jossa koin usein olevani lähinnä jonkinlainen robotti jolloin kaikki tahdonvapauden ja valinnanmahdollisuuden kokemukset lähinnä loistivat poissaolollaan suurimman osan ajasta.

        Belisario

        Nimimerkki: Professori Realisti (XPR-0044)

        Belisario: " Bohmin piilomuuttujamallin perusteella voidaan ennustaa tarkasti myös elektronien käyttäytyminen kaksoisrakokeessa. [...]"

        Olen (vasta) nyt takaisin tässä ketjussa [tosin todennäköisesti "ehtimisteknisistä" syistä nytkin vain hyvin väliaikaisesti...]. Yritän ehtiä katsoa tuon viittaamasi Bohm-dokumentin, koska aihe kuulostaa kiinnostavalta. Tässä vaiheessa toisin lähinnä esille seuraavia huomioitani de Broglie-Bohm -teoriaan (BBT) liittyen.

        BBT:n kannatus asiantuntijoiden parissa on oheisen kartoitukseni mukaan vain n. 0.5% (11. sija) ja sen perusoletustenkin kannatus on selvästi vähäisin 15:n merkittävimmän tulkinnan joukossa (n. 22%). Itseasiassa sen kaikkien yhdeksän perusoletuksen kannatus jää vähemmistöön, mikä on selvästi poikkeuksellista (se ei myöskään ota selvästi kantaa aaltofunktion ns. romahduksen reaalisuuteen).
        https://keskustelu.suomi24.fi/t/16472246/kvanttimekaniikan-tulkinnoista

        Eniten sen suosiota vähentää se, että se olettaa piilomuuttujia (n. 96% vastanneista asiantuntijoista piti välillisesti tuota kantaa virheellisenä) ja se, että se olettaa suorittamattomien mittausten tulosten yksikäsitteisyyden (CFD) (n. 85% pitää tuota virheellisenä). Asiantuntijoiden vähiten todennäköisesti vääränä pitämä BBT:n oletus on se, että järjestelmillä on yksikäsitteinen historia (UNH), mutta enemmistö (n. 70%) pitää sitäkin vääränä.

        Tosin, esim. omasta mielestäni todennäköisimmin juuri tuo UNH-oletus on todennäköisimmin väärä, koska tutkittaessa oletuksiltaan erilaisten merkittävien tulkintojen kantoja; noiden yhdeksän aspektin/oletuksen osalta, selvimmin konsensuksena (n. 10% keskiarvosta) erottuu se, että tuo oletus on väärä; jota seikkaa on valaistu tuossa aloittamassani ketjussa. UNH vaikuttaa melko vahvasti lähinnä jonkinlaiselta aaltofunktion ns. romahduksen reaalisuuden "ad hoc" -tyyppiseltä apuoletukselta. BBT tosin ei oleta yksikäsitteistä aaltohistoriaa; kuten esim. Kööpenhaminan tulkinta, vaan vain yksikäsitteiden hiukkashistorian.

        Kaikesta tuosta huolimatta BBT menestyy itseasiassa hyvin; arvioitaessa sitä nimenomaan tuon "sen oletuksista erilaisten tulkintojen osalta vallitsevan konsensuksen perusteella", eli se on toisaalta hyvin erilainen kuin suosituimmat tulkinnat, mutta toisaalta omaa suhteellisen paljon yhteisiä piirteitä monien vähemmän suosittujen tulkintojen kanssa. Tuolla tavalla arvioituna se sijoittuu kolmanneksi; tosin hyvin selvästi MMI:n ja MWI:n jälkeen, joista se eroaa mm. siinä suhteessa, että se ei oleta lokaalia realismia. BBT on ehkä yksi oudoimmista kvanttimekaniikan tulkinnoista, mitä todennäköisesti pidät plussana todellisuuden selittämisen kannalta. :D

        Kannattamani MWI:n oletusten osalta tuo lokaali realismi on ehkä vähiten varmasti oikeana pitämäni, mutta mahdollisesti kaikki havainnot ovat silti selitettävissä MWI:n kautta, vaikka tuo oletus olisikin virheellinen? Eräs tuon analyysini tulos oli se, että vaikuttaa vahvasti siltä, että tulkintojen suosituimmuus selittyy hyvin ns. "sopuli-ilmiön" kautta ja toisaalta kun esim. Sinut tuntien tietää, että vastustat ns. "main-stream -tiedettä" kiivasti; koska se on "tylsää ja epäilyttävää", niin mieleen tulee tietenkin se, että voisiko tuo BBT:n kannatuksesi puolestaan selittyä parhaiten ns. "käänteisellä sopuli-ilmiöllä". :D


      • Anonyymi
        Anonyymi kirjoitti:

        Nimimerkki: Professori Realisti (XPR-0044)

        Belisario: " Bohmin piilomuuttujamallin perusteella voidaan ennustaa tarkasti myös elektronien käyttäytyminen kaksoisrakokeessa. [...]"

        Olen (vasta) nyt takaisin tässä ketjussa [tosin todennäköisesti "ehtimisteknisistä" syistä nytkin vain hyvin väliaikaisesti...]. Yritän ehtiä katsoa tuon viittaamasi Bohm-dokumentin, koska aihe kuulostaa kiinnostavalta. Tässä vaiheessa toisin lähinnä esille seuraavia huomioitani de Broglie-Bohm -teoriaan (BBT) liittyen.

        BBT:n kannatus asiantuntijoiden parissa on oheisen kartoitukseni mukaan vain n. 0.5% (11. sija) ja sen perusoletustenkin kannatus on selvästi vähäisin 15:n merkittävimmän tulkinnan joukossa (n. 22%). Itseasiassa sen kaikkien yhdeksän perusoletuksen kannatus jää vähemmistöön, mikä on selvästi poikkeuksellista (se ei myöskään ota selvästi kantaa aaltofunktion ns. romahduksen reaalisuuteen).
        https://keskustelu.suomi24.fi/t/16472246/kvanttimekaniikan-tulkinnoista

        Eniten sen suosiota vähentää se, että se olettaa piilomuuttujia (n. 96% vastanneista asiantuntijoista piti välillisesti tuota kantaa virheellisenä) ja se, että se olettaa suorittamattomien mittausten tulosten yksikäsitteisyyden (CFD) (n. 85% pitää tuota virheellisenä). Asiantuntijoiden vähiten todennäköisesti vääränä pitämä BBT:n oletus on se, että järjestelmillä on yksikäsitteinen historia (UNH), mutta enemmistö (n. 70%) pitää sitäkin vääränä.

        Tosin, esim. omasta mielestäni todennäköisimmin juuri tuo UNH-oletus on todennäköisimmin väärä, koska tutkittaessa oletuksiltaan erilaisten merkittävien tulkintojen kantoja; noiden yhdeksän aspektin/oletuksen osalta, selvimmin konsensuksena (n. 10% keskiarvosta) erottuu se, että tuo oletus on väärä; jota seikkaa on valaistu tuossa aloittamassani ketjussa. UNH vaikuttaa melko vahvasti lähinnä jonkinlaiselta aaltofunktion ns. romahduksen reaalisuuden "ad hoc" -tyyppiseltä apuoletukselta. BBT tosin ei oleta yksikäsitteistä aaltohistoriaa; kuten esim. Kööpenhaminan tulkinta, vaan vain yksikäsitteiden hiukkashistorian.

        Kaikesta tuosta huolimatta BBT menestyy itseasiassa hyvin; arvioitaessa sitä nimenomaan tuon "sen oletuksista erilaisten tulkintojen osalta vallitsevan konsensuksen perusteella", eli se on toisaalta hyvin erilainen kuin suosituimmat tulkinnat, mutta toisaalta omaa suhteellisen paljon yhteisiä piirteitä monien vähemmän suosittujen tulkintojen kanssa. Tuolla tavalla arvioituna se sijoittuu kolmanneksi; tosin hyvin selvästi MMI:n ja MWI:n jälkeen, joista se eroaa mm. siinä suhteessa, että se ei oleta lokaalia realismia. BBT on ehkä yksi oudoimmista kvanttimekaniikan tulkinnoista, mitä todennäköisesti pidät plussana todellisuuden selittämisen kannalta. :D

        Kannattamani MWI:n oletusten osalta tuo lokaali realismi on ehkä vähiten varmasti oikeana pitämäni, mutta mahdollisesti kaikki havainnot ovat silti selitettävissä MWI:n kautta, vaikka tuo oletus olisikin virheellinen? Eräs tuon analyysini tulos oli se, että vaikuttaa vahvasti siltä, että tulkintojen suosituimmuus selittyy hyvin ns. "sopuli-ilmiön" kautta ja toisaalta kun esim. Sinut tuntien tietää, että vastustat ns. "main-stream -tiedettä" kiivasti; koska se on "tylsää ja epäilyttävää", niin mieleen tulee tietenkin se, että voisiko tuo BBT:n kannatuksesi puolestaan selittyä parhaiten ns. "käänteisellä sopuli-ilmiöllä". :D

        Nimimerkki: Professori Realisti (XPR-0045)

        XPR-0044: "Asiantuntijoiden vähiten todennäköisesti vääränä pitämä BBT:n oletus on se, että järjestelmillä on yksikäsitteinen historia (UNH), mutta enemmistö (n. 70%) pitää sitäkin vääränä."

        Tuo enemmistö meni tuossa tietenkin väärinpäin; eli, tuo on kylläkin asiantuntijoiden vähiten todennäköisesti vääränä pitämä BBT:n oletus, mutta enemmistö (n. 70%) pitää sitä *oikeana*, eli se on *selvästi* noiden asiantuntijoiden sen vähiten vääränä pitämä oletus...


      • Anonyymi kirjoitti:

        Nimimerkki: Professori Realisti (XPR-0044)

        Belisario: " Bohmin piilomuuttujamallin perusteella voidaan ennustaa tarkasti myös elektronien käyttäytyminen kaksoisrakokeessa. [...]"

        Olen (vasta) nyt takaisin tässä ketjussa [tosin todennäköisesti "ehtimisteknisistä" syistä nytkin vain hyvin väliaikaisesti...]. Yritän ehtiä katsoa tuon viittaamasi Bohm-dokumentin, koska aihe kuulostaa kiinnostavalta. Tässä vaiheessa toisin lähinnä esille seuraavia huomioitani de Broglie-Bohm -teoriaan (BBT) liittyen.

        BBT:n kannatus asiantuntijoiden parissa on oheisen kartoitukseni mukaan vain n. 0.5% (11. sija) ja sen perusoletustenkin kannatus on selvästi vähäisin 15:n merkittävimmän tulkinnan joukossa (n. 22%). Itseasiassa sen kaikkien yhdeksän perusoletuksen kannatus jää vähemmistöön, mikä on selvästi poikkeuksellista (se ei myöskään ota selvästi kantaa aaltofunktion ns. romahduksen reaalisuuteen).
        https://keskustelu.suomi24.fi/t/16472246/kvanttimekaniikan-tulkinnoista

        Eniten sen suosiota vähentää se, että se olettaa piilomuuttujia (n. 96% vastanneista asiantuntijoista piti välillisesti tuota kantaa virheellisenä) ja se, että se olettaa suorittamattomien mittausten tulosten yksikäsitteisyyden (CFD) (n. 85% pitää tuota virheellisenä). Asiantuntijoiden vähiten todennäköisesti vääränä pitämä BBT:n oletus on se, että järjestelmillä on yksikäsitteinen historia (UNH), mutta enemmistö (n. 70%) pitää sitäkin vääränä.

        Tosin, esim. omasta mielestäni todennäköisimmin juuri tuo UNH-oletus on todennäköisimmin väärä, koska tutkittaessa oletuksiltaan erilaisten merkittävien tulkintojen kantoja; noiden yhdeksän aspektin/oletuksen osalta, selvimmin konsensuksena (n. 10% keskiarvosta) erottuu se, että tuo oletus on väärä; jota seikkaa on valaistu tuossa aloittamassani ketjussa. UNH vaikuttaa melko vahvasti lähinnä jonkinlaiselta aaltofunktion ns. romahduksen reaalisuuden "ad hoc" -tyyppiseltä apuoletukselta. BBT tosin ei oleta yksikäsitteistä aaltohistoriaa; kuten esim. Kööpenhaminan tulkinta, vaan vain yksikäsitteiden hiukkashistorian.

        Kaikesta tuosta huolimatta BBT menestyy itseasiassa hyvin; arvioitaessa sitä nimenomaan tuon "sen oletuksista erilaisten tulkintojen osalta vallitsevan konsensuksen perusteella", eli se on toisaalta hyvin erilainen kuin suosituimmat tulkinnat, mutta toisaalta omaa suhteellisen paljon yhteisiä piirteitä monien vähemmän suosittujen tulkintojen kanssa. Tuolla tavalla arvioituna se sijoittuu kolmanneksi; tosin hyvin selvästi MMI:n ja MWI:n jälkeen, joista se eroaa mm. siinä suhteessa, että se ei oleta lokaalia realismia. BBT on ehkä yksi oudoimmista kvanttimekaniikan tulkinnoista, mitä todennäköisesti pidät plussana todellisuuden selittämisen kannalta. :D

        Kannattamani MWI:n oletusten osalta tuo lokaali realismi on ehkä vähiten varmasti oikeana pitämäni, mutta mahdollisesti kaikki havainnot ovat silti selitettävissä MWI:n kautta, vaikka tuo oletus olisikin virheellinen? Eräs tuon analyysini tulos oli se, että vaikuttaa vahvasti siltä, että tulkintojen suosituimmuus selittyy hyvin ns. "sopuli-ilmiön" kautta ja toisaalta kun esim. Sinut tuntien tietää, että vastustat ns. "main-stream -tiedettä" kiivasti; koska se on "tylsää ja epäilyttävää", niin mieleen tulee tietenkin se, että voisiko tuo BBT:n kannatuksesi puolestaan selittyä parhaiten ns. "käänteisellä sopuli-ilmiöllä". :D

        Minäkin koen BBT:n eräänlaisena ad-hoc selityksenä.

        En ole varma mitä tulkintaa itse kannattaisin. Pidän mahdollisena, että täysin materialistinen hiukkas/aalto-ajattelu voi olla perusteiltaan väärä.

        Jos olet perehtynyt aiheeseen syvemmin, niin pistä ihmeessä ketju, missä esität kvanttimekaanisen mysteerin ja lyhyen yhteenvedon mahdollisista eri tulkinnoista.


      • Anonyymi kirjoitti:

        Luonnontieteet kykenevät käsittelemään ainoastaan eksplisiittisiä asioita ja niihin kuuluu kaikki määrät jotka ovat aistivaikutelmien (=ekplisiittisen "maailman" hahmotus) aspektien (etäisyys, taajuus., voima, intensiteetti yms. mitattavissa tai oikeammin keskenään vertailtavissa olevia asioita ) suhteita toisiinsa ja ne ovat metrisiä ja/tai geometrisia mutta koska ne ovat vain sen eksplisiittisen virtuaalitodellisuuden hahmotuksia ja hahmotustapoja niin luonnontieteet ylipäätänsä ei kykene ainakaan nykyisessä muodossaan käsittelemään sitä perimmäistä implisiittistä todellisuutta jossa ei ole määriä eikä metristä geometriaa koska se on nonlokaali (=ei etäisyyksiä eikä määrien välisiä suhteita) ja se implisiittinen todellisuus ilmenee inhimillisessä kokemuksessa (joka on paljon laajempi kuin aistihavainnot) laatuina, resonansseina, synkronisiteettina, ideoina, tunteina, inspiraationa, etiikkana, moraalina, viisautena.

        Koska kvanttifysiikka käsittelee ilmiöitä jotka vaikuttavat aivan eksplisiittisen ja implisiittisen todellisuuden rajapinnassa niin sen ilmiöt voivat olla sekä non-lokaaleja että hyvin outoja (superpositiot, tunneloitumiset, hiukkas-aaltodualismi,lomittuminen) ja koska kaikki metriikka ja geometria liittyvät ainoastaan eksplisiittisen todellisuuden hahmotukseen ja aistikokemuksien aspektien välisiin vertailuihin niin kvanttifysiikan ilmiöitä on mahdotonta yhdistää eksplisiittisen geometrisen ja metrisen fysiikan (suhteellisuusteoriat ja Newtonin fysiikka) teorioihin ja malleihin jolloin suurin osa nykyisestä kvanttifysiikastakin on pielessä koska se yrittää mallintaa todellisuutta kvantin eli kuvitellun pienimmän mahdollisen etäisyyden ja vuorovaikutuksen pohjalta (Planckin vakio) eli yrittää määritellä sellaisia ominaisuuksia kuvitelluille alkeishiukkasille joita nonlokaalissa maailmassa ei voi esiintyä esim. senttimetrien murto-osien ja kuviteltujen "mittabosonien" ja sähkövarausten ja massan kautta jotka on päätelty tai kuviteltu kuplakammioiden kuvioiden tai hiukkaskiihdyttimien tilastollisten ilmiöiden (eräänlaisia Rossachin musteläiskiä).

        Alkeishiukkasia ovat pikseleitä eksplisiitisen virtuaalitodellisuuden "kuvaruudulla" on ja aallot ja kentät ovat vertauskuvia jotka viittaavat nonlokaaliin implisiittiseen todellisuuteen.

        Voiko tiede ylipäätänsä kertoa mitään sellaisesta todellisuudesta joka ei ole mitattavissa eikä laskettavissa kvantitatiivisesti eikä siis myöskään geometrisesti kuten suhteellisuusteorioissa?

        Voi ja kun metriikka ja geometria tiputetaan pois niin jäljelle jää vielä topologian käytön mahdollisuus. Luonnontieteet siis tutkivat lähinnä virtuaalitodellisuutta.


        Nyt tuli näköjään aika hegeliläistä hyvin koukeroista tekstiä jota on vaikea kirjoittaa ja varmaan vielä vaikeampaa ymmärtää koska yritin tietoisesti ja kieli keskellä suuta varovaisesti hahmotella jotenkin edes auttavasti ja eksplisiittisesti sellaista minkä voi kunnolla ymmärtää ainoastaan intuition ja vertauskuvien avulla koska olen huomannut että ns. luonnontieteellis-matemaattisesti orientoituneet yksilöt eivät ymmärrä vertauskuvia eikä analogioita vaan kaikki pitää vääntää hyvin yksiselitteisesti, kirjaimellisesti ja mahdollisimman eksaktisti hyvin paksulla rautalangalla:D

        Eikä siitä sittenkään varmaan kukaan mitään ymmärrä enkä varmaan itsekään ymmärtäisi jos aikaisempi minäni muutama kymmenen vuotta sitten lukisi tätä tekstiä. [ :D & :( ]

        Belisario

        Luulen ymmärtäneeni.

        Kuulostaa, että sinulla on jonkinlainen intuitiivinen ”syvempi” kuva fysikaalisen maailman alla olevasta luonteesta, mitä et oikein osaa selittää analyyttisesti. Näet kvanttimekaniikassa jotain mystisiä piirteitä ja oletat intuitiivisesti, että tämä on jonkinlainen silta alla olevan hengellisen aidon todellisuuden ja fysikaalisen eli henkisestä todellisuudesta nousevan virtuaalisen todellisuuden välillä.

        Minä ajattelen toisinpäin: Henkinen todellisuus nousee materialistisen pohjalta. Toki todellisuus ei koostu hiukkasista vaan kvanttimekaniikan ilmiöt osoittavat, että kaiken alla oleva pohjimmainen todellisuus on sumeampi.

        Minun on vaikea ymmärtää, minkä järkevän linkin näet kvanttimekaniikan ja kokemuksellisen henkisen todellisuutemme välillä. Kenties osaat selventää tätä?

        Samoin en ymmärrä, miten selität itsellesi erilaiset neurologian havainnot siitä, miten kokemusmaailmamme on liitoksissa ”virtuaalisessa” fysikaalisessa maailmassa olevaan neuroverkkoon.


      • Anonyymi
        Järkisyitä kirjoitti:

        Luulen ymmärtäneeni.

        Kuulostaa, että sinulla on jonkinlainen intuitiivinen ”syvempi” kuva fysikaalisen maailman alla olevasta luonteesta, mitä et oikein osaa selittää analyyttisesti. Näet kvanttimekaniikassa jotain mystisiä piirteitä ja oletat intuitiivisesti, että tämä on jonkinlainen silta alla olevan hengellisen aidon todellisuuden ja fysikaalisen eli henkisestä todellisuudesta nousevan virtuaalisen todellisuuden välillä.

        Minä ajattelen toisinpäin: Henkinen todellisuus nousee materialistisen pohjalta. Toki todellisuus ei koostu hiukkasista vaan kvanttimekaniikan ilmiöt osoittavat, että kaiken alla oleva pohjimmainen todellisuus on sumeampi.

        Minun on vaikea ymmärtää, minkä järkevän linkin näet kvanttimekaniikan ja kokemuksellisen henkisen todellisuutemme välillä. Kenties osaat selventää tätä?

        Samoin en ymmärrä, miten selität itsellesi erilaiset neurologian havainnot siitä, miten kokemusmaailmamme on liitoksissa ”virtuaalisessa” fysikaalisessa maailmassa olevaan neuroverkkoon.

        "Luulen ymmärtäneeni."

        Epäilen vahvasti sinun vain luulevan:D

        "Kuulostaa, että sinulla on jonkinlainen intuitiivinen ”syvempi” kuva fysikaalisen maailman alla olevasta luonteesta, mitä et oikein osaa selittää analyyttisesti. Näet kvanttimekaniikassa jotain mystisiä piirteitä ja oletat intuitiivisesti, että tämä on jonkinlainen silta alla olevan hengellisen aidon todellisuuden ja fysikaalisen eli henkisestä todellisuudesta nousevan virtuaalisen todellisuuden välillä."

        Lukaisin äsken kokonaan Paul LaVioletten kuvauksen hänen omasta eetterimallistaan "subquantum kinetics" joka vastaa aika pitkälle sitä mitä olen tässä ketjussa yrittänyt esittää jonkinlaisena "kaiken teoriana".

        https://www.researchgate.net/publication/264785921_The_Cosmic_Ether_Introduction_to_Subquantum_Kinetics

        Tietysti minulla on intutiivinen käsitys joka ohjaa ajatteluani ja jota on melko vaikea esittää sillä tavalla analyyttisesti että voisi eliminoida samalla kaikki mahdolliset väärinymmärrykset.

        "Minä ajattelen toisinpäin: Henkinen todellisuus nousee materialistisen pohjalta. Toki todellisuus ei koostu hiukkasista vaan kvanttimekaniikan ilmiöt osoittavat, että kaiken alla oleva pohjimmainen todellisuus on sumeampi."

        Voi sitä niinkin ajatella eli ensin "laskeudutaan aineeseen" ja sitten aineellisten kokemusten kautta "noustaan takaisin henkeen" kokemusten suhteen rikkaampana.

        "Minun on vaikea ymmärtää, minkä järkevän linkin näet kvanttimekaniikan ja kokemuksellisen henkisen todellisuutemme välillä. Kenties osaat selventää tätä?"

        Nykyisessä neurotieteessä ei yleensä edes yritetä hahmottaa aivojen ilmiöitä kvanttifysiikan pohjalta mutta on jo olemassa kvanttibiologiaa ja esim. Hameroffin malli mikrotubuuleista joka pyrkii ottamaan tämänkin huomioon. Aivojen kvantti-ilmiöitä on tietysti hankala tutkia hiukkasfysiikan tutkimusvälineillä kuten hiukkaskiihdyttimillä tuhoamatta samalla sitä tutkittavaa kohdetta. Kokemuksellisuus on aina subjektiivista eli ei ole havaittavissa sellaisenaan millään laitteistolla muuten kuin ulkoisten näkyvien seuraustensa osalta.

        "Samoin en ymmärrä, miten selität itsellesi erilaiset neurologian havainnot siitä, miten kokemusmaailmamme on liitoksissa ”virtuaalisessa” fysikaalisessa maailmassa olevaan neuroverkkoon."

        Suurin piirtein samalla tavalla kuin esim. Bernardo Kastrup eli että se neuroverkko (aivot & hermosto) on se miltä se kokemusmaailma näyttää kolmannen ulkopuolisen persoonan näkökulmasta.

        Nyt olen istunut jo niin kauan yhteenmenoon koneen ääressä että lopetan tältä päivältä.

        Belisario


      • Anonyymi kirjoitti:

        "Luulen ymmärtäneeni."

        Epäilen vahvasti sinun vain luulevan:D

        "Kuulostaa, että sinulla on jonkinlainen intuitiivinen ”syvempi” kuva fysikaalisen maailman alla olevasta luonteesta, mitä et oikein osaa selittää analyyttisesti. Näet kvanttimekaniikassa jotain mystisiä piirteitä ja oletat intuitiivisesti, että tämä on jonkinlainen silta alla olevan hengellisen aidon todellisuuden ja fysikaalisen eli henkisestä todellisuudesta nousevan virtuaalisen todellisuuden välillä."

        Lukaisin äsken kokonaan Paul LaVioletten kuvauksen hänen omasta eetterimallistaan "subquantum kinetics" joka vastaa aika pitkälle sitä mitä olen tässä ketjussa yrittänyt esittää jonkinlaisena "kaiken teoriana".

        https://www.researchgate.net/publication/264785921_The_Cosmic_Ether_Introduction_to_Subquantum_Kinetics

        Tietysti minulla on intutiivinen käsitys joka ohjaa ajatteluani ja jota on melko vaikea esittää sillä tavalla analyyttisesti että voisi eliminoida samalla kaikki mahdolliset väärinymmärrykset.

        "Minä ajattelen toisinpäin: Henkinen todellisuus nousee materialistisen pohjalta. Toki todellisuus ei koostu hiukkasista vaan kvanttimekaniikan ilmiöt osoittavat, että kaiken alla oleva pohjimmainen todellisuus on sumeampi."

        Voi sitä niinkin ajatella eli ensin "laskeudutaan aineeseen" ja sitten aineellisten kokemusten kautta "noustaan takaisin henkeen" kokemusten suhteen rikkaampana.

        "Minun on vaikea ymmärtää, minkä järkevän linkin näet kvanttimekaniikan ja kokemuksellisen henkisen todellisuutemme välillä. Kenties osaat selventää tätä?"

        Nykyisessä neurotieteessä ei yleensä edes yritetä hahmottaa aivojen ilmiöitä kvanttifysiikan pohjalta mutta on jo olemassa kvanttibiologiaa ja esim. Hameroffin malli mikrotubuuleista joka pyrkii ottamaan tämänkin huomioon. Aivojen kvantti-ilmiöitä on tietysti hankala tutkia hiukkasfysiikan tutkimusvälineillä kuten hiukkaskiihdyttimillä tuhoamatta samalla sitä tutkittavaa kohdetta. Kokemuksellisuus on aina subjektiivista eli ei ole havaittavissa sellaisenaan millään laitteistolla muuten kuin ulkoisten näkyvien seuraustensa osalta.

        "Samoin en ymmärrä, miten selität itsellesi erilaiset neurologian havainnot siitä, miten kokemusmaailmamme on liitoksissa ”virtuaalisessa” fysikaalisessa maailmassa olevaan neuroverkkoon."

        Suurin piirtein samalla tavalla kuin esim. Bernardo Kastrup eli että se neuroverkko (aivot & hermosto) on se miltä se kokemusmaailma näyttää kolmannen ulkopuolisen persoonan näkökulmasta.

        Nyt olen istunut jo niin kauan yhteenmenoon koneen ääressä että lopetan tältä päivältä.

        Belisario

        Kiitos linkistä - koitan lukaista, kun aika sallii. Hyvää lauantain jatkoa!


      • Anonyymi kirjoitti:

        "Luulen ymmärtäneeni."

        Epäilen vahvasti sinun vain luulevan:D

        "Kuulostaa, että sinulla on jonkinlainen intuitiivinen ”syvempi” kuva fysikaalisen maailman alla olevasta luonteesta, mitä et oikein osaa selittää analyyttisesti. Näet kvanttimekaniikassa jotain mystisiä piirteitä ja oletat intuitiivisesti, että tämä on jonkinlainen silta alla olevan hengellisen aidon todellisuuden ja fysikaalisen eli henkisestä todellisuudesta nousevan virtuaalisen todellisuuden välillä."

        Lukaisin äsken kokonaan Paul LaVioletten kuvauksen hänen omasta eetterimallistaan "subquantum kinetics" joka vastaa aika pitkälle sitä mitä olen tässä ketjussa yrittänyt esittää jonkinlaisena "kaiken teoriana".

        https://www.researchgate.net/publication/264785921_The_Cosmic_Ether_Introduction_to_Subquantum_Kinetics

        Tietysti minulla on intutiivinen käsitys joka ohjaa ajatteluani ja jota on melko vaikea esittää sillä tavalla analyyttisesti että voisi eliminoida samalla kaikki mahdolliset väärinymmärrykset.

        "Minä ajattelen toisinpäin: Henkinen todellisuus nousee materialistisen pohjalta. Toki todellisuus ei koostu hiukkasista vaan kvanttimekaniikan ilmiöt osoittavat, että kaiken alla oleva pohjimmainen todellisuus on sumeampi."

        Voi sitä niinkin ajatella eli ensin "laskeudutaan aineeseen" ja sitten aineellisten kokemusten kautta "noustaan takaisin henkeen" kokemusten suhteen rikkaampana.

        "Minun on vaikea ymmärtää, minkä järkevän linkin näet kvanttimekaniikan ja kokemuksellisen henkisen todellisuutemme välillä. Kenties osaat selventää tätä?"

        Nykyisessä neurotieteessä ei yleensä edes yritetä hahmottaa aivojen ilmiöitä kvanttifysiikan pohjalta mutta on jo olemassa kvanttibiologiaa ja esim. Hameroffin malli mikrotubuuleista joka pyrkii ottamaan tämänkin huomioon. Aivojen kvantti-ilmiöitä on tietysti hankala tutkia hiukkasfysiikan tutkimusvälineillä kuten hiukkaskiihdyttimillä tuhoamatta samalla sitä tutkittavaa kohdetta. Kokemuksellisuus on aina subjektiivista eli ei ole havaittavissa sellaisenaan millään laitteistolla muuten kuin ulkoisten näkyvien seuraustensa osalta.

        "Samoin en ymmärrä, miten selität itsellesi erilaiset neurologian havainnot siitä, miten kokemusmaailmamme on liitoksissa ”virtuaalisessa” fysikaalisessa maailmassa olevaan neuroverkkoon."

        Suurin piirtein samalla tavalla kuin esim. Bernardo Kastrup eli että se neuroverkko (aivot & hermosto) on se miltä se kokemusmaailma näyttää kolmannen ulkopuolisen persoonan näkökulmasta.

        Nyt olen istunut jo niin kauan yhteenmenoon koneen ääressä että lopetan tältä päivältä.

        Belisario

        Nyt olen vielä enemmän pihalla siitä, mitä ajattelet. Et oikeasti voi millään sulattaa kaikkia LaVioletten paperissaan esittämiä ajatuksia...

        Ei tuota LaVioletten paperia voi ottaa kovin vakavasti. Herra referoi enimmäkseen vain omiin aikaisempiin papereihinsa. Heti toisella sivulla hän kertoo kumonneensa suhteellisuusteorian ja mitanneen laboratoriossa viisinkertaisen valonnopeuden. Hän katsoo tämän ja erään toisen kokeen kumoavan suhteellisuusteorian perusteellisesti ja esittää tilalle omaa uudelleen subkvantti-eetteri hypoteesiaan. Hän katsoo Big Bangin virheelliseksi, selittää punasiirtymän väsyneellä valolla ja esittää, että tähtiä ja planeettoja syntyy eetteristä jatkuvasti lisää tyhjästä.

        Pienellä googlettamisella selvisi, että LaViolette on myös kirjoittanut kirjan, missä hän vertaa kuinka hyvin luomismyytit sopivat uusimpaan tieteelliseen tietoon. Hän on myös perehtynyt UFO:ssa käytettyihin tekniikoihin kuten antigravity-moottoriin.


      • Anonyymi
        Järkisyitä kirjoitti:

        Nyt olen vielä enemmän pihalla siitä, mitä ajattelet. Et oikeasti voi millään sulattaa kaikkia LaVioletten paperissaan esittämiä ajatuksia...

        Ei tuota LaVioletten paperia voi ottaa kovin vakavasti. Herra referoi enimmäkseen vain omiin aikaisempiin papereihinsa. Heti toisella sivulla hän kertoo kumonneensa suhteellisuusteorian ja mitanneen laboratoriossa viisinkertaisen valonnopeuden. Hän katsoo tämän ja erään toisen kokeen kumoavan suhteellisuusteorian perusteellisesti ja esittää tilalle omaa uudelleen subkvantti-eetteri hypoteesiaan. Hän katsoo Big Bangin virheelliseksi, selittää punasiirtymän väsyneellä valolla ja esittää, että tähtiä ja planeettoja syntyy eetteristä jatkuvasti lisää tyhjästä.

        Pienellä googlettamisella selvisi, että LaViolette on myös kirjoittanut kirjan, missä hän vertaa kuinka hyvin luomismyytit sopivat uusimpaan tieteelliseen tietoon. Hän on myös perehtynyt UFO:ssa käytettyihin tekniikoihin kuten antigravity-moottoriin.

        "Nyt olen vielä enemmän pihalla siitä, mitä ajattelet. Et oikeasti voi millään sulattaa kaikkia LaVioletten paperissaan esittämiä ajatuksia..."

        Niin sen LaVioletten perusteet teorialleen ovat aivan erilaiset kuin valtavirtafysiikassa joten ne vaativat aika paljon sulattelua.

        LaVioletten mallin ominaisuuksia:

        1) Mallin perustana on yleinen systeemiteoria eli sen luonteeltaan holistinen eikä reduktionistinen malli kuten perinteisen fysiikan mallit

        2) Alkeishiukkaset ovat Turing aalto rakenteita (pattern) jotka itseorganisoituvat dynaamisessa eetterissä ja muostavat perustan kaikille fysikaalisille muodoille maailmankaikkeudessa (vrt. Sheldraken muotokentät)

        https://en.wikipedia.org/wiki/Turing_pattern

        Tämä eetteri on hyperdimensioonalinen kuvaukseltaan eli edellyttää enemmän vapausasteita kuin esim. suhteellisuusteorin 3d 1t malli. Eetteri on myös jatkuvasti aktiivinen reaktio-diffuusio järjestelmä.

        https://en.wikipedia.org/wiki/Reaction–diffusion_system

        Alkeishiukkaset ovat avoimia systeejä eli edellyttävät jatkuvaa energian vaihtoa eetterin kanssa aivan samoin kuin orgaaniset eliöt aineenvaihdunnassaan. Eetteri on epälineaarinen, reaktiivinen ja vuorovaikutteinen suhteessa alkeishiukkasiin jatkuvasti.

        "Heti toisella sivulla hän kertoo kumonneensa suhteellisuusteorian ja mitanneen laboratoriossa viisinkertaisen valonnopeuden."

        Engl. fyysikko E.T. Whittaker esitti jo ennen suppean suhteellisuusteorian julkistusta että eetterin sisäiset vuorovaikutukset ja gravitatio voivat liikkua äärimmäisen paljon valonnopeutta nopeammin (ks. aikaisemmat linkkini tässä ketjussa) eli ovat ns. skalaari tai pitkittäisaaltoja (longitudinal waves) erotuksena herz-aalloista joihin pätee valonnopeuden vakio.

        Eetteriä sinänsä ei ole kumottu ja sen osoitti Sagnac 1900 luvun alussa jossa hän teki korjatun Michelson-Morley kokeen jossa otettiin huomioon maapallon pyöriminen akselinsa ympäri.

        "Hän katsoo Big Bangin virheelliseksi"

        Perusteluna on että niin valtavaa kvanttifluktuaatiota ei voi tapahtua kuin BB-malli edellyttää mutta sen sijaan hiukkasia syntyy jatkuvasti eetterin sisäisten nollasummavektoreiden tuottamasta stressistä josta kirjoitin aikaisemmin tässä ketjussa.
        https://www.researchgate.net/publication/281440634_Study_of_Light_Interaction_with_Gravity_Impulses_and_Measurements_of_the_Speed_of_Gravity_Impulses

        "selittää punasiirtymän väsyneellä valolla"

        Niin koska ne hiukkaset vuorovaikuttavat jatkuvasti eetterin stressipisteiden (auringot, planeetat) joista ne saavat energiaa joka taas kuluu niiden liikkuessa tyhjässä avaruudessa eli ts. elektronit ovat samalla tavalla avoimia systeemeitä kuin organismit. Väsyneen valon hypoteesi on fiksumpi idea kuin se että fotoni liikkuisi jatkuvasti samalla nopeudella. Einstein itsekin mietti sitä että valonnopeus voisi vaihdella gravitaation mukaan eli olla nopeampi tyhjässä avaruudessa ja gravitaatiokeskittymien lähellä vastaavasti hitaammin jolloin ei tarvitsisi postuloida avaruuden kaareutumista mikä onkin aika outo oletus.

        ks aloitus ketjussa jossa kerron Unzickerin tutkimuksista aiheesta

        https://keskustelu.suomi24.fi/t/16461316/suhteellisuusteorioiden-kritiikki-(a--unzicker)

        Einsteinin 1928 yhteinäiskenttäteoriassa oli mukana mahdollisuus manipuloida aika-avaruutta eli siis myös gravitaatiota paikallisesti.

        Eetterin itseorganisoituvuus liittyy Nobel palkitin Ilya Prigoginen tutkimuksiin itseorganisoituvista epätasapainossa olevista systeemeistä.

        https://en.wikipedia.org/wiki/Ilya_Prigogine

        LaViolette vertaa eetterimalliaan brusselaattoriin eli autokatalyyttisten reaktioiden teor. malliin

        https://en.wikipedia.org/wiki/Brusselator

        Kun selviää kulttuurishokista niin tuo vaikuttaa hyvinkin järkevältä mallilta . Itse olen suurin piirtein koko ikäni tutkinut hyvinkin outoja asioita joten suhtiksien, valonnopeuden ja BB.mallin kumoaminen ovat minulle pikkujuttuja vaikka tietysti noiden mallien kannattajille varmaan vähän isompi juttu:D

        "Pienellä googlettamisella selvisi, että LaViolette on myös kirjoittanut kirjan, missä hän vertaa kuinka hyvin luomismyytit sopivat uusimpaan tieteelliseen tietoon."

        Joo mulla on se kirja ja se on varsin mielenkiintoinen. Hindumyytit varsinkin ovat hyvin yhteensopivia modernin tieteen kanssa.

        "Hän on myös perehtynyt UFO:ssa käytettyihin tekniikoihin kuten antigravity-moottoriin."

        Niin minäkin. Antigravitaatioista oli paljon kirjallisuutta julkisuudessa maailmansotien välissä mutta ne katosivat sitten jossain vaiheessa. Gravitaatio syntyy jonkinlaisesta spin-polarisaatiosta ja eetterin nollapistevektoreiden stressistä kuten T. Bearden ja T.E. Whittaker esittävät ehkä tähän tyyliin:

        Searches for the role of spin and polarization in gravity

        https://iopscience.iop.org/article/10.1088/0034-4885/73/5/056901

        itselläni on ainakin 3 LaViolette kirjaa jotka luin aika monta vuotta sitten. Pitäs lukea uudestaan kun nyt vois ymmärtää vähän enemmän kuin silloin.

        Belisario


      • Anonyymi kirjoitti:

        "Nyt olen vielä enemmän pihalla siitä, mitä ajattelet. Et oikeasti voi millään sulattaa kaikkia LaVioletten paperissaan esittämiä ajatuksia..."

        Niin sen LaVioletten perusteet teorialleen ovat aivan erilaiset kuin valtavirtafysiikassa joten ne vaativat aika paljon sulattelua.

        LaVioletten mallin ominaisuuksia:

        1) Mallin perustana on yleinen systeemiteoria eli sen luonteeltaan holistinen eikä reduktionistinen malli kuten perinteisen fysiikan mallit

        2) Alkeishiukkaset ovat Turing aalto rakenteita (pattern) jotka itseorganisoituvat dynaamisessa eetterissä ja muostavat perustan kaikille fysikaalisille muodoille maailmankaikkeudessa (vrt. Sheldraken muotokentät)

        https://en.wikipedia.org/wiki/Turing_pattern

        Tämä eetteri on hyperdimensioonalinen kuvaukseltaan eli edellyttää enemmän vapausasteita kuin esim. suhteellisuusteorin 3d 1t malli. Eetteri on myös jatkuvasti aktiivinen reaktio-diffuusio järjestelmä.

        https://en.wikipedia.org/wiki/Reaction–diffusion_system

        Alkeishiukkaset ovat avoimia systeejä eli edellyttävät jatkuvaa energian vaihtoa eetterin kanssa aivan samoin kuin orgaaniset eliöt aineenvaihdunnassaan. Eetteri on epälineaarinen, reaktiivinen ja vuorovaikutteinen suhteessa alkeishiukkasiin jatkuvasti.

        "Heti toisella sivulla hän kertoo kumonneensa suhteellisuusteorian ja mitanneen laboratoriossa viisinkertaisen valonnopeuden."

        Engl. fyysikko E.T. Whittaker esitti jo ennen suppean suhteellisuusteorian julkistusta että eetterin sisäiset vuorovaikutukset ja gravitatio voivat liikkua äärimmäisen paljon valonnopeutta nopeammin (ks. aikaisemmat linkkini tässä ketjussa) eli ovat ns. skalaari tai pitkittäisaaltoja (longitudinal waves) erotuksena herz-aalloista joihin pätee valonnopeuden vakio.

        Eetteriä sinänsä ei ole kumottu ja sen osoitti Sagnac 1900 luvun alussa jossa hän teki korjatun Michelson-Morley kokeen jossa otettiin huomioon maapallon pyöriminen akselinsa ympäri.

        "Hän katsoo Big Bangin virheelliseksi"

        Perusteluna on että niin valtavaa kvanttifluktuaatiota ei voi tapahtua kuin BB-malli edellyttää mutta sen sijaan hiukkasia syntyy jatkuvasti eetterin sisäisten nollasummavektoreiden tuottamasta stressistä josta kirjoitin aikaisemmin tässä ketjussa.
        https://www.researchgate.net/publication/281440634_Study_of_Light_Interaction_with_Gravity_Impulses_and_Measurements_of_the_Speed_of_Gravity_Impulses

        "selittää punasiirtymän väsyneellä valolla"

        Niin koska ne hiukkaset vuorovaikuttavat jatkuvasti eetterin stressipisteiden (auringot, planeetat) joista ne saavat energiaa joka taas kuluu niiden liikkuessa tyhjässä avaruudessa eli ts. elektronit ovat samalla tavalla avoimia systeemeitä kuin organismit. Väsyneen valon hypoteesi on fiksumpi idea kuin se että fotoni liikkuisi jatkuvasti samalla nopeudella. Einstein itsekin mietti sitä että valonnopeus voisi vaihdella gravitaation mukaan eli olla nopeampi tyhjässä avaruudessa ja gravitaatiokeskittymien lähellä vastaavasti hitaammin jolloin ei tarvitsisi postuloida avaruuden kaareutumista mikä onkin aika outo oletus.

        ks aloitus ketjussa jossa kerron Unzickerin tutkimuksista aiheesta

        https://keskustelu.suomi24.fi/t/16461316/suhteellisuusteorioiden-kritiikki-(a--unzicker)

        Einsteinin 1928 yhteinäiskenttäteoriassa oli mukana mahdollisuus manipuloida aika-avaruutta eli siis myös gravitaatiota paikallisesti.

        Eetterin itseorganisoituvuus liittyy Nobel palkitin Ilya Prigoginen tutkimuksiin itseorganisoituvista epätasapainossa olevista systeemeistä.

        https://en.wikipedia.org/wiki/Ilya_Prigogine

        LaViolette vertaa eetterimalliaan brusselaattoriin eli autokatalyyttisten reaktioiden teor. malliin

        https://en.wikipedia.org/wiki/Brusselator

        Kun selviää kulttuurishokista niin tuo vaikuttaa hyvinkin järkevältä mallilta . Itse olen suurin piirtein koko ikäni tutkinut hyvinkin outoja asioita joten suhtiksien, valonnopeuden ja BB.mallin kumoaminen ovat minulle pikkujuttuja vaikka tietysti noiden mallien kannattajille varmaan vähän isompi juttu:D

        "Pienellä googlettamisella selvisi, että LaViolette on myös kirjoittanut kirjan, missä hän vertaa kuinka hyvin luomismyytit sopivat uusimpaan tieteelliseen tietoon."

        Joo mulla on se kirja ja se on varsin mielenkiintoinen. Hindumyytit varsinkin ovat hyvin yhteensopivia modernin tieteen kanssa.

        "Hän on myös perehtynyt UFO:ssa käytettyihin tekniikoihin kuten antigravity-moottoriin."

        Niin minäkin. Antigravitaatioista oli paljon kirjallisuutta julkisuudessa maailmansotien välissä mutta ne katosivat sitten jossain vaiheessa. Gravitaatio syntyy jonkinlaisesta spin-polarisaatiosta ja eetterin nollapistevektoreiden stressistä kuten T. Bearden ja T.E. Whittaker esittävät ehkä tähän tyyliin:

        Searches for the role of spin and polarization in gravity

        https://iopscience.iop.org/article/10.1088/0034-4885/73/5/056901

        itselläni on ainakin 3 LaViolette kirjaa jotka luin aika monta vuotta sitten. Pitäs lukea uudestaan kun nyt vois ymmärtää vähän enemmän kuin silloin.

        Belisario

        Ihanko oikeasti kuvittelet, että yksi tyyppi olisi kumonnut suhteellisuusteorian, keksinyt antigravitaatiomoottorin periaatteen sekä vielä kumonnut Big Bang:in tuoden paremman selityksen asioille? Jätkällehän kuuluisi Nobelin palkinto toisensa jälkeen.

        Mutta kun lukee tuon hemmon paperia, niin koko paperi on tyhjän päälle rakennettu lähinnä verbaalisti höpötetty hypoteesi, jonka tueksi ei esitetä oikein mitään mittauksia. Hän ei esitä mitään kunnon mallinnuksia sille, miten esimerkiksi aikadilaation suhteellisuusteorian mittaustulokset selitetään eetteri-hypoteesin avulla. Samoin jää epäselväksi, mitä hän on edes kyennyt mallintamaan hypoteesinsa kanssa. Hän ottaa esiin vain joitakin satunnaisia fysiikanilmiöitä, mutta herää kysymys sopiiko malli oikein mihinkään muuhun kuin muutamaan yksittäisen ilmiön selittäjäksi.

        ”Perusteluna on että niin valtavaa kvanttifluktuaatiota ei voi tapahtua kuin BB-malli edellyttää”

        Tuosta kommentista selviää, että LaViolette ei ole asiansa tasalla. Jokainen BB-malliin perehtynyt tietää, ettei kvanttifluktuaatiota pidetä yleisesti BB:n aiheuttajana. Kyseinen ajatus on esitetty, mutta se on varsin spekulatiivinen - eikä ole mitenkään hyväksytty. Argumentti siis ampuu täysin ohi maalistaan ja ei voi kuin ihmetellä, miten LaViolette ei tiennyt tätä.

        ”Eetteriä sinänsä ei ole kumottu ja sen osoitti Sagnac 1900 luvun alussa”

        Olet ajastasi yli sata vuotta jäljessä. Eetteri teoriat kumoutuivat, kun valon nopeus mitattiin kerta toisensa jälkeen vakioksi. Erilaisia eetteri-teorioita on esitetty, mutta ne täyttävät Wikipediankin mukaan kaikki pseudotieteen tunnusmerkit.


      • Anonyymi
        Järkisyitä kirjoitti:

        Ihanko oikeasti kuvittelet, että yksi tyyppi olisi kumonnut suhteellisuusteorian, keksinyt antigravitaatiomoottorin periaatteen sekä vielä kumonnut Big Bang:in tuoden paremman selityksen asioille? Jätkällehän kuuluisi Nobelin palkinto toisensa jälkeen.

        Mutta kun lukee tuon hemmon paperia, niin koko paperi on tyhjän päälle rakennettu lähinnä verbaalisti höpötetty hypoteesi, jonka tueksi ei esitetä oikein mitään mittauksia. Hän ei esitä mitään kunnon mallinnuksia sille, miten esimerkiksi aikadilaation suhteellisuusteorian mittaustulokset selitetään eetteri-hypoteesin avulla. Samoin jää epäselväksi, mitä hän on edes kyennyt mallintamaan hypoteesinsa kanssa. Hän ottaa esiin vain joitakin satunnaisia fysiikanilmiöitä, mutta herää kysymys sopiiko malli oikein mihinkään muuhun kuin muutamaan yksittäisen ilmiön selittäjäksi.

        ”Perusteluna on että niin valtavaa kvanttifluktuaatiota ei voi tapahtua kuin BB-malli edellyttää”

        Tuosta kommentista selviää, että LaViolette ei ole asiansa tasalla. Jokainen BB-malliin perehtynyt tietää, ettei kvanttifluktuaatiota pidetä yleisesti BB:n aiheuttajana. Kyseinen ajatus on esitetty, mutta se on varsin spekulatiivinen - eikä ole mitenkään hyväksytty. Argumentti siis ampuu täysin ohi maalistaan ja ei voi kuin ihmetellä, miten LaViolette ei tiennyt tätä.

        ”Eetteriä sinänsä ei ole kumottu ja sen osoitti Sagnac 1900 luvun alussa”

        Olet ajastasi yli sata vuotta jäljessä. Eetteri teoriat kumoutuivat, kun valon nopeus mitattiin kerta toisensa jälkeen vakioksi. Erilaisia eetteri-teorioita on esitetty, mutta ne täyttävät Wikipediankin mukaan kaikki pseudotieteen tunnusmerkit.

        "Ihanko oikeasti kuvittelet, että yksi tyyppi ..."

        En.

        Suhteellisuusteorioita ei olisi pitänyt koskaan hyväksyä. LaViolette ei ole eikä ollut ainoa suhtiksien kriitikko. Niitä oli useita (mm. Tesla joka oikeasti keksi jotain hyödyllistä eikä pelkästään harrastanut nojatuolissaan ajatuskokeita kuten Einstein). LaViolette ei keksinyt antigravitaatiota vaan osa sen perusteista oli jo löydettävissä maailmansotien välisestä tieteellisestä kirjallisuudesta ja myöhemmin esim. Thomas Townsend Brownin tutkimuksista.


        https://en.wikipedia.org/wiki/Thomas_Townsend_Brown

        Suurin osa tämän alan tutkimuksista on ollut koko ajan salaista tai kadonnut julkisuudesta 1950-luvun jälkeen ja osa on julkaistu varsinkin 1990-luvun jälkeen (Neuvostoliiton kaatuminen ja Saksojen yhdistyminen). LaViolette kirjoitti aiheesta kirjan jonka ajattelin kohta lukea uudestaan kunhan olen ensin lukenut pari muuta paksuhkoa teosta samaa aihetta sivuten.

        Nobeleita myönnetään lähinnä sellaisille oligarkkeja miellyttäville tutkijoille joilla usein on loistava tulevaisuus kaukana takanapäin... :D

        Tyypillisenä skepona et vaivaudu edes lukemaan annettuja linkkejä etkä varsinkaan vaivaudu edes yrittämään ymmmärtää niitä koska uskot jo tietäväsi asian oikein laidan.

        "Mutta kun lukee tuon hemmon paperia, ....
        "
        Koska eetterin implisiittinen "rakenne" koostuu nollasummavektoreista niin niitä on vaikea mitata muuten kuin sähkömagneettisena potentiaalina (ks. Aharonov–Bohm effect)

        "Hän ei esitä mitään kunnon mallinnuksia sille, miten esimerkiksi aikadilaation suhteellisuusteorian mittaustulokset selitetään eetteri-hypoteesin avulla."

        Jonkinlainen aikadilaatio syntyy kun eetteriä manipuloidaan eli vääristetään aika-avaruutta. Eetterissä ei ole aikaa eikä paikkaa vaan se on käytännössä nonlokaali ja eetterin sisäiset vuorovaikutukset (ns. lomittuminen) voi olla vaikka miljoona kertaa C tai lähes välittömät. Gravitaation avulla voidaan sekä hidastaa että nopeuttaa luonnon normaaleja syklejä.

        LaViolette ei ole ainoa joka on kirjoittanut dynaamisesta eetteristä (myös Tesla, Tom Bearden,Eric Dollard jne. ) ja hän on lähinnä vain postuloinut siihen jonkinlaisen eetterihiukkasen (alkeishiukkasten ja hypoteettisten kvarkkien perustan) joka vaikuttaa suurinpiirtein Planckin mittakaavassa ja tuottaa avaruudelle solumaisen (lattice) rakenteen jolloin eetterivuorovaikutukset ovat luonteeltaan elektro-akustisia eli jonkinlaisia paineaaltoja jolloin vuorovaikutukset koko maailmankaikkeuden alueella ovat miltei välittömiä)

        " Samoin jää epäselväksi, ....."

        Et ilmeisestikään ymmärtänyt siitä lukemastasi juuri mitään enkä minäkään aikoinaan n. 10 v sitten kun aloin tiiviimmin tutkimaan tämäntyyppisiä asioita. Juuri tuon takia pyrin välttämään kaikkea uskomista mikä saattaisi vaikeuttaa uudenlaisen tiedon ymmärtämistä.

        "Tuosta kommentista selviää, että LaViolette ei ole asiansa tasalla. ..."

        Kvanttifluktuaatiota on varsin yleisesti esitetty alkuräjähdyksen alullepanijana joten kritiikkisi ampuu taas ohi. Tyypillisen skeptikon ei tarvitse koskaan yrittää ymmärtää mitään vaan pelkästään kyseenalaistaa vaivautumatta perehtymään asiaan pintaa syvemmälle.

        Ateisti-skepot ovat minusta varsin säälittävää väkeä jotka pyrkivät ratsastamaan tieteen arvovallalla kaikkea sellaista vastaan mitä eivät halua ymmärtää tai joka tuntuu heistä vastenmieliseltä. Tällaisten tyyppien kanssa on melko turha alkaa väitellä paria viestiä pidempään koska heidän intressinsä on lähinnä leikkiä jonkinlaista ajatuspoliisia jolloin he samalla kuvittelevat olevansa jotenkin tieteen asialla.

        "Olet ajastasi yli sata vuotta jäljessä. ...."

        Tottakai se on vakio maanpinnalla tai Maapallon läheisyydessä mitattuna mutta jos vaihdetaan avaruuden kaareutumisen postulaatti valonnopeuden vaihteluun paikan mukaan niin on aivan mahdollista että ns. syvässä avaruudessa saadaan aivan eri mittaustulos kuin esim aivan auringon lähellä. Et näköjään taaskaan vaivautunut lukemaan ja ymmärtämään lukemaasi.

        Wikipedialla samoin kuin RationalWikiä-sivustoilla on selkeä agenda poistaa tai mustamaalata kaikki valtavirrasta poikkeavat näkemykset esim. kuittaamalla ns. pseudotieteeksi jossa taustakuvitelmana on että vain virallinen ns. tiedeyhteisön konseksuksen hyväksymä komiteatiede on sitä "oikeaa tiedettä"

        Kirjoitit juuri sellaisen vastauksen kuin oletinkin sinun kirjoittavan mikä on sekä aika huvittavaa että myös surullista & säälittävää.

        Belisario


      • Anonyymi kirjoitti:

        "Ihanko oikeasti kuvittelet, että yksi tyyppi ..."

        En.

        Suhteellisuusteorioita ei olisi pitänyt koskaan hyväksyä. LaViolette ei ole eikä ollut ainoa suhtiksien kriitikko. Niitä oli useita (mm. Tesla joka oikeasti keksi jotain hyödyllistä eikä pelkästään harrastanut nojatuolissaan ajatuskokeita kuten Einstein). LaViolette ei keksinyt antigravitaatiota vaan osa sen perusteista oli jo löydettävissä maailmansotien välisestä tieteellisestä kirjallisuudesta ja myöhemmin esim. Thomas Townsend Brownin tutkimuksista.


        https://en.wikipedia.org/wiki/Thomas_Townsend_Brown

        Suurin osa tämän alan tutkimuksista on ollut koko ajan salaista tai kadonnut julkisuudesta 1950-luvun jälkeen ja osa on julkaistu varsinkin 1990-luvun jälkeen (Neuvostoliiton kaatuminen ja Saksojen yhdistyminen). LaViolette kirjoitti aiheesta kirjan jonka ajattelin kohta lukea uudestaan kunhan olen ensin lukenut pari muuta paksuhkoa teosta samaa aihetta sivuten.

        Nobeleita myönnetään lähinnä sellaisille oligarkkeja miellyttäville tutkijoille joilla usein on loistava tulevaisuus kaukana takanapäin... :D

        Tyypillisenä skepona et vaivaudu edes lukemaan annettuja linkkejä etkä varsinkaan vaivaudu edes yrittämään ymmmärtää niitä koska uskot jo tietäväsi asian oikein laidan.

        "Mutta kun lukee tuon hemmon paperia, ....
        "
        Koska eetterin implisiittinen "rakenne" koostuu nollasummavektoreista niin niitä on vaikea mitata muuten kuin sähkömagneettisena potentiaalina (ks. Aharonov–Bohm effect)

        "Hän ei esitä mitään kunnon mallinnuksia sille, miten esimerkiksi aikadilaation suhteellisuusteorian mittaustulokset selitetään eetteri-hypoteesin avulla."

        Jonkinlainen aikadilaatio syntyy kun eetteriä manipuloidaan eli vääristetään aika-avaruutta. Eetterissä ei ole aikaa eikä paikkaa vaan se on käytännössä nonlokaali ja eetterin sisäiset vuorovaikutukset (ns. lomittuminen) voi olla vaikka miljoona kertaa C tai lähes välittömät. Gravitaation avulla voidaan sekä hidastaa että nopeuttaa luonnon normaaleja syklejä.

        LaViolette ei ole ainoa joka on kirjoittanut dynaamisesta eetteristä (myös Tesla, Tom Bearden,Eric Dollard jne. ) ja hän on lähinnä vain postuloinut siihen jonkinlaisen eetterihiukkasen (alkeishiukkasten ja hypoteettisten kvarkkien perustan) joka vaikuttaa suurinpiirtein Planckin mittakaavassa ja tuottaa avaruudelle solumaisen (lattice) rakenteen jolloin eetterivuorovaikutukset ovat luonteeltaan elektro-akustisia eli jonkinlaisia paineaaltoja jolloin vuorovaikutukset koko maailmankaikkeuden alueella ovat miltei välittömiä)

        " Samoin jää epäselväksi, ....."

        Et ilmeisestikään ymmärtänyt siitä lukemastasi juuri mitään enkä minäkään aikoinaan n. 10 v sitten kun aloin tiiviimmin tutkimaan tämäntyyppisiä asioita. Juuri tuon takia pyrin välttämään kaikkea uskomista mikä saattaisi vaikeuttaa uudenlaisen tiedon ymmärtämistä.

        "Tuosta kommentista selviää, että LaViolette ei ole asiansa tasalla. ..."

        Kvanttifluktuaatiota on varsin yleisesti esitetty alkuräjähdyksen alullepanijana joten kritiikkisi ampuu taas ohi. Tyypillisen skeptikon ei tarvitse koskaan yrittää ymmärtää mitään vaan pelkästään kyseenalaistaa vaivautumatta perehtymään asiaan pintaa syvemmälle.

        Ateisti-skepot ovat minusta varsin säälittävää väkeä jotka pyrkivät ratsastamaan tieteen arvovallalla kaikkea sellaista vastaan mitä eivät halua ymmärtää tai joka tuntuu heistä vastenmieliseltä. Tällaisten tyyppien kanssa on melko turha alkaa väitellä paria viestiä pidempään koska heidän intressinsä on lähinnä leikkiä jonkinlaista ajatuspoliisia jolloin he samalla kuvittelevat olevansa jotenkin tieteen asialla.

        "Olet ajastasi yli sata vuotta jäljessä. ...."

        Tottakai se on vakio maanpinnalla tai Maapallon läheisyydessä mitattuna mutta jos vaihdetaan avaruuden kaareutumisen postulaatti valonnopeuden vaihteluun paikan mukaan niin on aivan mahdollista että ns. syvässä avaruudessa saadaan aivan eri mittaustulos kuin esim aivan auringon lähellä. Et näköjään taaskaan vaivautunut lukemaan ja ymmärtämään lukemaasi.

        Wikipedialla samoin kuin RationalWikiä-sivustoilla on selkeä agenda poistaa tai mustamaalata kaikki valtavirrasta poikkeavat näkemykset esim. kuittaamalla ns. pseudotieteeksi jossa taustakuvitelmana on että vain virallinen ns. tiedeyhteisön konseksuksen hyväksymä komiteatiede on sitä "oikeaa tiedettä"

        Kirjoitit juuri sellaisen vastauksen kuin oletinkin sinun kirjoittavan mikä on sekä aika huvittavaa että myös surullista & säälittävää.

        Belisario

        ”Suhteellisuusteorioita ei olisi pitänyt koskaan hyväksyä.”

        Erityinen suhteellisuusteoria on todistettu kerta toisensa jälkeen paikkansa pitäväksi. Se perustuu yhteen ainoaan havaintoon: valonnopeus tyhjiössä on sama kaikille tarkkailijoille. Aina silloin tällöin löytyy joku tutkija, joka pääsee otsikoihin väittäen mitanneen yli valonnopeudella tapahtuneen ilmiön, mutta nämä on kerta toisensa jälkeen osoittautuneet mittavirheiksi, kun niitä tutkitaan lähemmin. Toki voit löytää kvanttifysiikan puolelta ilmiöitä, jotka mahdollistavat non-lokaalisuuden tai hiukkasen tunnelointi-ilmiöitä, jotka jopa rikkovat suhteellisuusteoriaa, mutta nämä ilmiöt eivät mahdollista yli valonnopeudella tapahtuvaa tiedonsiirtoa makromaailmassa.

        ”Jonkinlainen aikadilaatio syntyy kun eetteriä manipuloidaan eli vääristetään aika-avaruutta.”

        Voit verbaalisti väittää vaikka mitä, mutta huomannet, ettei LaVioleteltä löydy mitään teoreettista pohjaa johtaa mallia aikadilaatiolle lähtien hänen eetteri-teoriastaan. On täysin eri asia esittää jokin sumea verbaalinen selitys aikadilaatiolle. Täysin eri asia on johtaa perusteista lähtien teoreettinen matemaattinen malli joka täsmää mittauksien kanssa jatkuvana funktiona eikä vain yhdessä erityistilanteessa.

        ”Suurin osa tämän alan tutkimuksista on ollut koko ajan salaista tai kadonnut ”

        Joopa-joo. Nämä salaliitto-teoriat ovat jo aika väsähtäneitä. Eikö ole outoa, että aina kun joku tukee jotain tosi outoa tieteen-vastavirran vastaista teoriaa, niin sitten aletaan tuoda esiin salaliittoteorioita ja syyttämään toista puolta ad-hominem argumentaatiolla.

        Laita linkki matemaattisten aikadilaatio-kaavojen johtoon LaVioleten eetteri-mallista lähtien. Jos tälläisiä ei edes löydy, niin LaVioleten eetteri-hypoteesi on vielä varsin raakile.

        ”LaViolette ei ole ainoa joka on kirjoittanut dynaamisesta eetteristä (myös Tesla, Tom Bearden,Eric Dollard jne. )”

        Niin. Eetteri-hypoteeseja esitettiin paljonkin ennen suhteellisuusteorian vakiintumista... Jokainen tieteellinen hypoteesi lepää kuitenkin todisteidensa varassa.


      • Anonyymi
        Anonyymi kirjoitti:

        "Ihanko oikeasti kuvittelet, että yksi tyyppi ..."

        En.

        Suhteellisuusteorioita ei olisi pitänyt koskaan hyväksyä. LaViolette ei ole eikä ollut ainoa suhtiksien kriitikko. Niitä oli useita (mm. Tesla joka oikeasti keksi jotain hyödyllistä eikä pelkästään harrastanut nojatuolissaan ajatuskokeita kuten Einstein). LaViolette ei keksinyt antigravitaatiota vaan osa sen perusteista oli jo löydettävissä maailmansotien välisestä tieteellisestä kirjallisuudesta ja myöhemmin esim. Thomas Townsend Brownin tutkimuksista.


        https://en.wikipedia.org/wiki/Thomas_Townsend_Brown

        Suurin osa tämän alan tutkimuksista on ollut koko ajan salaista tai kadonnut julkisuudesta 1950-luvun jälkeen ja osa on julkaistu varsinkin 1990-luvun jälkeen (Neuvostoliiton kaatuminen ja Saksojen yhdistyminen). LaViolette kirjoitti aiheesta kirjan jonka ajattelin kohta lukea uudestaan kunhan olen ensin lukenut pari muuta paksuhkoa teosta samaa aihetta sivuten.

        Nobeleita myönnetään lähinnä sellaisille oligarkkeja miellyttäville tutkijoille joilla usein on loistava tulevaisuus kaukana takanapäin... :D

        Tyypillisenä skepona et vaivaudu edes lukemaan annettuja linkkejä etkä varsinkaan vaivaudu edes yrittämään ymmmärtää niitä koska uskot jo tietäväsi asian oikein laidan.

        "Mutta kun lukee tuon hemmon paperia, ....
        "
        Koska eetterin implisiittinen "rakenne" koostuu nollasummavektoreista niin niitä on vaikea mitata muuten kuin sähkömagneettisena potentiaalina (ks. Aharonov–Bohm effect)

        "Hän ei esitä mitään kunnon mallinnuksia sille, miten esimerkiksi aikadilaation suhteellisuusteorian mittaustulokset selitetään eetteri-hypoteesin avulla."

        Jonkinlainen aikadilaatio syntyy kun eetteriä manipuloidaan eli vääristetään aika-avaruutta. Eetterissä ei ole aikaa eikä paikkaa vaan se on käytännössä nonlokaali ja eetterin sisäiset vuorovaikutukset (ns. lomittuminen) voi olla vaikka miljoona kertaa C tai lähes välittömät. Gravitaation avulla voidaan sekä hidastaa että nopeuttaa luonnon normaaleja syklejä.

        LaViolette ei ole ainoa joka on kirjoittanut dynaamisesta eetteristä (myös Tesla, Tom Bearden,Eric Dollard jne. ) ja hän on lähinnä vain postuloinut siihen jonkinlaisen eetterihiukkasen (alkeishiukkasten ja hypoteettisten kvarkkien perustan) joka vaikuttaa suurinpiirtein Planckin mittakaavassa ja tuottaa avaruudelle solumaisen (lattice) rakenteen jolloin eetterivuorovaikutukset ovat luonteeltaan elektro-akustisia eli jonkinlaisia paineaaltoja jolloin vuorovaikutukset koko maailmankaikkeuden alueella ovat miltei välittömiä)

        " Samoin jää epäselväksi, ....."

        Et ilmeisestikään ymmärtänyt siitä lukemastasi juuri mitään enkä minäkään aikoinaan n. 10 v sitten kun aloin tiiviimmin tutkimaan tämäntyyppisiä asioita. Juuri tuon takia pyrin välttämään kaikkea uskomista mikä saattaisi vaikeuttaa uudenlaisen tiedon ymmärtämistä.

        "Tuosta kommentista selviää, että LaViolette ei ole asiansa tasalla. ..."

        Kvanttifluktuaatiota on varsin yleisesti esitetty alkuräjähdyksen alullepanijana joten kritiikkisi ampuu taas ohi. Tyypillisen skeptikon ei tarvitse koskaan yrittää ymmärtää mitään vaan pelkästään kyseenalaistaa vaivautumatta perehtymään asiaan pintaa syvemmälle.

        Ateisti-skepot ovat minusta varsin säälittävää väkeä jotka pyrkivät ratsastamaan tieteen arvovallalla kaikkea sellaista vastaan mitä eivät halua ymmärtää tai joka tuntuu heistä vastenmieliseltä. Tällaisten tyyppien kanssa on melko turha alkaa väitellä paria viestiä pidempään koska heidän intressinsä on lähinnä leikkiä jonkinlaista ajatuspoliisia jolloin he samalla kuvittelevat olevansa jotenkin tieteen asialla.

        "Olet ajastasi yli sata vuotta jäljessä. ...."

        Tottakai se on vakio maanpinnalla tai Maapallon läheisyydessä mitattuna mutta jos vaihdetaan avaruuden kaareutumisen postulaatti valonnopeuden vaihteluun paikan mukaan niin on aivan mahdollista että ns. syvässä avaruudessa saadaan aivan eri mittaustulos kuin esim aivan auringon lähellä. Et näköjään taaskaan vaivautunut lukemaan ja ymmärtämään lukemaasi.

        Wikipedialla samoin kuin RationalWikiä-sivustoilla on selkeä agenda poistaa tai mustamaalata kaikki valtavirrasta poikkeavat näkemykset esim. kuittaamalla ns. pseudotieteeksi jossa taustakuvitelmana on että vain virallinen ns. tiedeyhteisön konseksuksen hyväksymä komiteatiede on sitä "oikeaa tiedettä"

        Kirjoitit juuri sellaisen vastauksen kuin oletinkin sinun kirjoittavan mikä on sekä aika huvittavaa että myös surullista & säälittävää.

        Belisario

        Ikävää Einsteinin mustamaalausta. Hän ei ollut vain nojatuoliajattelija, vaan aito keksijä, joka osasi rakennella jo nuorena kaikenlaisia kojeita. Hän paransi myöhemmin mm. jääkaapin toimintaa.


      • Anonyymi
        Järkisyitä kirjoitti:

        ”Suhteellisuusteorioita ei olisi pitänyt koskaan hyväksyä.”

        Erityinen suhteellisuusteoria on todistettu kerta toisensa jälkeen paikkansa pitäväksi. Se perustuu yhteen ainoaan havaintoon: valonnopeus tyhjiössä on sama kaikille tarkkailijoille. Aina silloin tällöin löytyy joku tutkija, joka pääsee otsikoihin väittäen mitanneen yli valonnopeudella tapahtuneen ilmiön, mutta nämä on kerta toisensa jälkeen osoittautuneet mittavirheiksi, kun niitä tutkitaan lähemmin. Toki voit löytää kvanttifysiikan puolelta ilmiöitä, jotka mahdollistavat non-lokaalisuuden tai hiukkasen tunnelointi-ilmiöitä, jotka jopa rikkovat suhteellisuusteoriaa, mutta nämä ilmiöt eivät mahdollista yli valonnopeudella tapahtuvaa tiedonsiirtoa makromaailmassa.

        ”Jonkinlainen aikadilaatio syntyy kun eetteriä manipuloidaan eli vääristetään aika-avaruutta.”

        Voit verbaalisti väittää vaikka mitä, mutta huomannet, ettei LaVioleteltä löydy mitään teoreettista pohjaa johtaa mallia aikadilaatiolle lähtien hänen eetteri-teoriastaan. On täysin eri asia esittää jokin sumea verbaalinen selitys aikadilaatiolle. Täysin eri asia on johtaa perusteista lähtien teoreettinen matemaattinen malli joka täsmää mittauksien kanssa jatkuvana funktiona eikä vain yhdessä erityistilanteessa.

        ”Suurin osa tämän alan tutkimuksista on ollut koko ajan salaista tai kadonnut ”

        Joopa-joo. Nämä salaliitto-teoriat ovat jo aika väsähtäneitä. Eikö ole outoa, että aina kun joku tukee jotain tosi outoa tieteen-vastavirran vastaista teoriaa, niin sitten aletaan tuoda esiin salaliittoteorioita ja syyttämään toista puolta ad-hominem argumentaatiolla.

        Laita linkki matemaattisten aikadilaatio-kaavojen johtoon LaVioleten eetteri-mallista lähtien. Jos tälläisiä ei edes löydy, niin LaVioleten eetteri-hypoteesi on vielä varsin raakile.

        ”LaViolette ei ole ainoa joka on kirjoittanut dynaamisesta eetteristä (myös Tesla, Tom Bearden,Eric Dollard jne. )”

        Niin. Eetteri-hypoteeseja esitettiin paljonkin ennen suhteellisuusteorian vakiintumista... Jokainen tieteellinen hypoteesi lepää kuitenkin todisteidensa varassa.

        ”Suhteellisuusteorioita ei olisi pitänyt koskaan hyväksyä.”

        Einstein kehitti er. suhteellisuuteorian Michelson-Morley kokeen perusteella joka oli väärin suunniteltu ja mittasi väärää asiaa. Sagnac suoritti korjatun kokeen ja löysi eetterivaikutuksen 1913 jota nykyään kutsutaan Sagnac-efektiksi koska er.suht. teoria oli jo yleisesti siihen aikaan jo hyväksytty vaikka se perustuikin väärään oletukseen että eetteriä ei ole olemassa. Dynaaminen eetteri on kuitenkin aivan eri asia kuin 1800-luvun fysiikan staattinen eetteri.

        https://en.wikipedia.org/wiki/Sagnac_effect

        "Erityinen suhteellisuusteoria on todistettu kerta toisensa jälkeen paikkansa pitäväksi. Se perustuu yhteen ainoaan havaintoon: valonnopeus tyhjiössä on sama kaikille tarkkailijoille. "

        Tuo ei ole edes mikään havainto vaan teoreettinen postulaatti joka on virheellinen.

        "Aina silloin tällöin löytyy joku tutkija, ..."

        Niin koska he yrittävät mitata ns. herz-aaltoja. Skalaariaallot ovat aivan eri asia ja ne toimivat epälineaarisesti eli ovat hyvin samantapaista resonanssia kuin esim. lasin rikkominen äänen avulla. Skalaariaallot löydettiin vahingossa 2. maailmansodan aikana tutkittaessa tutkien tuottamia anomalioita Saksassa. Ne toimivat esim. interferenssiperiaatteella eli vaativat 2 vastakkaista aaltoa (ks. phase conjugate mirror)

        "Toki voit löytää kvanttifysiikan puolelta ilmiöitä, jotka mahdollistavat non-lokaalisuuden tai hiukkasen tunnelointi-ilmiöitä, jotka jopa rikkovat suhteellisuusteoriaa, mutta nämä ilmiöt eivät mahdollista yli valonnopeudella tapahtuvaa tiedonsiirtoa makromaailmassa."

        Niinhän sitä väitetään mutta se ei pidä paikkaansa. Sekä suhtis & kvanttifysiikka ovat molemmat pielessä (tarkoituksella):D

        "Voit verbaalisti väittää vaikka mitä, mutta huomannet, ettei LaVioleteltä löydy mitään teoreettista pohjaa johtaa mallia aikadilaatiolle lähtien hänen eetteri-teoriastaan."

        Kyse ei ole samanlaisesta aikadilaatiosta kuin yleisessä suht. teoriassa joka ei mahdollista paikallista gravitaation muokkausta. Kyse on kaikenkaikkiaan hyvin erilaisesta ja monimutkaisesta asiasta kuin ns. valtavirtafysiikan teoriat.

        Olen viime aikoina löytänyt n. sata julkaisua jotka liittyvät dynaamiseen eetteriin ja tänään löysin varsinaisen aarreaitan paikasta jossa se oli "näkyvillä piilossa". LaVioletten malli on vain yksi monista joista varhaisimmat liiittyvät Nikolai Teslaan, E.T. Whittakeriin, 90-luvulla julkaistuihin sodanaikaisiin salaisiin asiakirjoihin ja tutkimuksiin. Noita ei voi pelkältä mutupohjalta tyrmätä ja minullakin on tutkimukset vielä hyvin pahasti kesken vaikka periaatteessa tuo dyn. eetterin ja moninkertaisen valonnopeuden mahdollisuus on ollut tiedossa jo ainakin 10 vuotta mutta en ole siihen aikaisemmin syventynyt riittävällä intensiteetillä.

        Relativistisen fysiikan yksi ongelma on että se on aivan oma viitekehyksensä koska Einstein ei lähtenyt havainnoista vaan ajatuskokeista tutkimaan asiaa ja valitsi aksioomaksi eetterin olemassaolon puuttumisen. Kun Eddington & co. tekivät Einsteinista ensimmäisen tieteen superpoptähden niin harva on kehdannut esittää julkista kritiikkiä mallia kohtaan koska se tulkittaisiin poliittisesti epäkorrektiksi (poikkeuksena mm. Herbert Ives ja Tesla) . Ongelma mittauksissa on myös se että ne mittaukset tulkitaan suhteellisuusteorioiden viitekehyksessä joka on jo alunperin virheellinen.

        "Joopa-joo. Nämä salaliitto-teoriat ovat jo aika väsähtäneitä. ..."

        Heh. Suhtikset ja kvanttifysiikka ovat melko harmittomia teorioita verrattuna Teslan alunperin kehittämään malliin jossa on potentiaalia räjäyttää kokonaisia planeettoja suhteellisen nopeasti ja helposti jolloin vetypommit vaikuttavat tavalliselta ilotulitteelta.

        Käypä esim. vilkaisemassa tätä 33 dokumentin kokoelmaa tällä sivustolla:

        https://archive.org/details/beardenferdelance2ndednscalaremweapons8602www.cheniere.org

        Ei se ole ad-hominemia jos syytän sinua siitä että et ole edes vaivautunut perehtymään aikaisemmin linkittämääni aineistoon. Kritiikkisi roikkuu edelleen ilmassa koska sinulla ei ole pienintäkään hajua siitä mitä olet kyseenalaistamassa. Itse en ole mikään suht.teorioiden asiantuntija mutta tiedän niistä taatusti paljon enemmän kuin sinä dyn. eetterin mallista.

        "Laita linkki matemaattisten aikadilaatio-kaavojen johtoon ..."

        Edelleen kyse ei ole samasta asiasta kuin yleisestä suhteellisuusteoriassa ja tuo aikadilaatio aspekti on dyn. eetterimallissa hyvin erillainen muutenkin.

        "Niin. Eetteri-hypoteeseja esitettiin paljonkin ennen suhteellisuusteorian vakiintumista... Jokainen tieteellinen hypoteesi lepää kuitenkin todisteidensa varassa."

        Dyn. eetterin malli ei ole koskaan ollut mikään täysin julkinen koska se on paitsi äärimmäisen vaarallinen sotilaallisesti sovellettu myös samalla romuttaisi sekä vallitsevan talousjärjestelmän että energiateollisuuden (nollapiste energia)

        Belisario


      • Anonyymi kirjoitti:

        ”Suhteellisuusteorioita ei olisi pitänyt koskaan hyväksyä.”

        Einstein kehitti er. suhteellisuuteorian Michelson-Morley kokeen perusteella joka oli väärin suunniteltu ja mittasi väärää asiaa. Sagnac suoritti korjatun kokeen ja löysi eetterivaikutuksen 1913 jota nykyään kutsutaan Sagnac-efektiksi koska er.suht. teoria oli jo yleisesti siihen aikaan jo hyväksytty vaikka se perustuikin väärään oletukseen että eetteriä ei ole olemassa. Dynaaminen eetteri on kuitenkin aivan eri asia kuin 1800-luvun fysiikan staattinen eetteri.

        https://en.wikipedia.org/wiki/Sagnac_effect

        "Erityinen suhteellisuusteoria on todistettu kerta toisensa jälkeen paikkansa pitäväksi. Se perustuu yhteen ainoaan havaintoon: valonnopeus tyhjiössä on sama kaikille tarkkailijoille. "

        Tuo ei ole edes mikään havainto vaan teoreettinen postulaatti joka on virheellinen.

        "Aina silloin tällöin löytyy joku tutkija, ..."

        Niin koska he yrittävät mitata ns. herz-aaltoja. Skalaariaallot ovat aivan eri asia ja ne toimivat epälineaarisesti eli ovat hyvin samantapaista resonanssia kuin esim. lasin rikkominen äänen avulla. Skalaariaallot löydettiin vahingossa 2. maailmansodan aikana tutkittaessa tutkien tuottamia anomalioita Saksassa. Ne toimivat esim. interferenssiperiaatteella eli vaativat 2 vastakkaista aaltoa (ks. phase conjugate mirror)

        "Toki voit löytää kvanttifysiikan puolelta ilmiöitä, jotka mahdollistavat non-lokaalisuuden tai hiukkasen tunnelointi-ilmiöitä, jotka jopa rikkovat suhteellisuusteoriaa, mutta nämä ilmiöt eivät mahdollista yli valonnopeudella tapahtuvaa tiedonsiirtoa makromaailmassa."

        Niinhän sitä väitetään mutta se ei pidä paikkaansa. Sekä suhtis & kvanttifysiikka ovat molemmat pielessä (tarkoituksella):D

        "Voit verbaalisti väittää vaikka mitä, mutta huomannet, ettei LaVioleteltä löydy mitään teoreettista pohjaa johtaa mallia aikadilaatiolle lähtien hänen eetteri-teoriastaan."

        Kyse ei ole samanlaisesta aikadilaatiosta kuin yleisessä suht. teoriassa joka ei mahdollista paikallista gravitaation muokkausta. Kyse on kaikenkaikkiaan hyvin erilaisesta ja monimutkaisesta asiasta kuin ns. valtavirtafysiikan teoriat.

        Olen viime aikoina löytänyt n. sata julkaisua jotka liittyvät dynaamiseen eetteriin ja tänään löysin varsinaisen aarreaitan paikasta jossa se oli "näkyvillä piilossa". LaVioletten malli on vain yksi monista joista varhaisimmat liiittyvät Nikolai Teslaan, E.T. Whittakeriin, 90-luvulla julkaistuihin sodanaikaisiin salaisiin asiakirjoihin ja tutkimuksiin. Noita ei voi pelkältä mutupohjalta tyrmätä ja minullakin on tutkimukset vielä hyvin pahasti kesken vaikka periaatteessa tuo dyn. eetterin ja moninkertaisen valonnopeuden mahdollisuus on ollut tiedossa jo ainakin 10 vuotta mutta en ole siihen aikaisemmin syventynyt riittävällä intensiteetillä.

        Relativistisen fysiikan yksi ongelma on että se on aivan oma viitekehyksensä koska Einstein ei lähtenyt havainnoista vaan ajatuskokeista tutkimaan asiaa ja valitsi aksioomaksi eetterin olemassaolon puuttumisen. Kun Eddington & co. tekivät Einsteinista ensimmäisen tieteen superpoptähden niin harva on kehdannut esittää julkista kritiikkiä mallia kohtaan koska se tulkittaisiin poliittisesti epäkorrektiksi (poikkeuksena mm. Herbert Ives ja Tesla) . Ongelma mittauksissa on myös se että ne mittaukset tulkitaan suhteellisuusteorioiden viitekehyksessä joka on jo alunperin virheellinen.

        "Joopa-joo. Nämä salaliitto-teoriat ovat jo aika väsähtäneitä. ..."

        Heh. Suhtikset ja kvanttifysiikka ovat melko harmittomia teorioita verrattuna Teslan alunperin kehittämään malliin jossa on potentiaalia räjäyttää kokonaisia planeettoja suhteellisen nopeasti ja helposti jolloin vetypommit vaikuttavat tavalliselta ilotulitteelta.

        Käypä esim. vilkaisemassa tätä 33 dokumentin kokoelmaa tällä sivustolla:

        https://archive.org/details/beardenferdelance2ndednscalaremweapons8602www.cheniere.org

        Ei se ole ad-hominemia jos syytän sinua siitä että et ole edes vaivautunut perehtymään aikaisemmin linkittämääni aineistoon. Kritiikkisi roikkuu edelleen ilmassa koska sinulla ei ole pienintäkään hajua siitä mitä olet kyseenalaistamassa. Itse en ole mikään suht.teorioiden asiantuntija mutta tiedän niistä taatusti paljon enemmän kuin sinä dyn. eetterin mallista.

        "Laita linkki matemaattisten aikadilaatio-kaavojen johtoon ..."

        Edelleen kyse ei ole samasta asiasta kuin yleisestä suhteellisuusteoriassa ja tuo aikadilaatio aspekti on dyn. eetterimallissa hyvin erillainen muutenkin.

        "Niin. Eetteri-hypoteeseja esitettiin paljonkin ennen suhteellisuusteorian vakiintumista... Jokainen tieteellinen hypoteesi lepää kuitenkin todisteidensa varassa."

        Dyn. eetterin malli ei ole koskaan ollut mikään täysin julkinen koska se on paitsi äärimmäisen vaarallinen sotilaallisesti sovellettu myös samalla romuttaisi sekä vallitsevan talousjärjestelmän että energiateollisuuden (nollapiste energia)

        Belisario

        ”Sagnac suoritti korjatun kokeen ja löysi eetterivaikutuksen 1913 jota nykyään kutsutaan Sagnac-efektiksi koska er.suht. teoria oli jo yleisesti siihen aikaan jo hyväksytty vaikka se perustuikin väärään oletukseen että eetteriä ei ole olemassa.”

        Kannattaisi lukea Wikipedian artikkeli kokonaan. Max von Laue osoitti jo vuonna 1911, että Sagnacin koe on suhteellisuusteorian mukainen. Sen sijaan se oli staattisen eetteri-hypoteesin kanssa ristiriitainen, koska pyörivä liike vaikutti eri suuntiin kulkeviin valoihin eri tavalla.

        Fysiikan historia on mielenkiintoinen, mutten ymmärrä edelleenkään, miksi haluat palata sata vuotta vanhoihin keskusteluihin, jolloin näyttö suhteellisuusteorian puolesta oli vähäisempää ja monet fyysikot vielä pitivät eetteriä varteenotettavana kilpailevana hypoteesina.

        Löydät varmasti paljon eetteri-hypoteeseja etenkin, jos tutkit lähes sata vuotta vanhaa matskua. Toki löytyy myös muutama vastavirtaan kulkija, joka tekee vielä omaa pseudotiedettään. Maallikkona kannattaa lukea tiedelehtiä eikä kuvitella, että pystyt koulukurssien pohjalta ymmärtämään julkaisuja.

        Minä olen lukenut korkeakoulun kattavimmat perusfysiikat, siis samat, jotka teoreettiset fyysikot lukevat, mutta koska en ole fyysikko, niin julkaisujen yksityiskohdat jäävät usein hämäräksi. Pystyn kuitenkin arvioimaan, miltä teoreettiselta pohjalta tutkija ponnistaa ja tuo eetteri-hypoteesi ei oikein perustu muuhun kuin jonkun yksittäisen tutkijan ideoimaan malliin, jonka hän on rukannut sopimaan joihinkin yksittäisiin ilmiöihin ja esittää näitä sitten hypoteesinsa tueksi. Oleellista on johtaa mallin pohjalta kokonaisuutta kuvaava matemaattisen tarkka malli eikä vain heilutella käsiä ja sanoa, että ”tää on niin monimutkainen, ettei sitä saa malliksi väännettyä”. Tuollaiset verbaaliset ”mallit” ovat tyypillisiä pseudotieteelle, missä terve skeptisyys omaa ideaa kohtaa puuttuu ja takerrutaan niihin harvoihin havaintoihin, jotka tukevat omaa ajatusta.

        ”Tuo ei ole edes mikään havainto vaan teoreettinen postulaatti joka on virheellinen.”

        Valon nopeus on mitattu useilla eri menetelmillä. Toki voit esittää kilpailevan hypoteesin, miksi kaikki menetelmät osoittavat valon nopeuden samaksi ja tarjota sen sijaan vaihtoehtoisen selityksen.

        https://en.wikipedia.org/wiki/Speed_of_light#Measurement

        https://www.newscientist.com/article/mg23331184-900-atomic-clocks-make-best-measurement-yet-of-relativity-of-time/

        ”Ongelma mittauksissa on myös se että ne mittaukset tulkitaan suhteellisuusteorioiden viitekehyksessä joka on jo alunperin virheellinen.”

        Se, että kaikki varmennetut mittaukset sopivat yhteen suhteellisuusteorian kanssa, ei ole ongelma. Kilpailevia hypoteeseja voi esittää, mutten ole vielä nähnyt toista teoriaa, joka selittää niin monta eri asiaa ja vieläpä tarjoaa matemaattisen tarkat kaavat, joita vastaan mittauksia voidaan verrata.

        Jos luet edellä olevat linkit, niin huomaat, että suhteellisuusteoriaa yritetään vieläkin todistaa vääriksi. Jos havaittaisiin, että suhteellisuusteorian aika-malli poikkeaisi edes hieman havaitusta, niin tämä olisi mullistava-johtolanka yrittäessä kehittää entistäkin tarkempia teorioita.

        ”Itse en ole mikään suht.teorioiden asiantuntija mutta tiedän niistä taatusti paljon enemmän kuin sinä dyn. eetterin mallista. ”

        Lienet oikeassa. Silti ymmärtänet, että jos kyseinen teoria olisi varteenotettava vaihtoehto suhteellisuusteorialle, niin siitä kirjoitettaisiin tiedelehdissä. Dynaamista eetteriä esittävillä on siis vielä edessä iso työtaakka ennen kuin heidän hypoteesinsa olisi kelvollinen.

        Esimerkiksi fyysikko Mark Barton Glasgowin yliopistosta kommentoi 2018 eetteri teorioita seuraavasti: ”Tietääkseni kukaan ei ole luonut riittävästi työstettyä eetteri teoriaa siinä mielessä, että se käynnistäisi keskustelun erityisestä suhteellisuusteoriasta.”


      • Anonyymi
        Järkisyitä kirjoitti:

        ”Sagnac suoritti korjatun kokeen ja löysi eetterivaikutuksen 1913 jota nykyään kutsutaan Sagnac-efektiksi koska er.suht. teoria oli jo yleisesti siihen aikaan jo hyväksytty vaikka se perustuikin väärään oletukseen että eetteriä ei ole olemassa.”

        Kannattaisi lukea Wikipedian artikkeli kokonaan. Max von Laue osoitti jo vuonna 1911, että Sagnacin koe on suhteellisuusteorian mukainen. Sen sijaan se oli staattisen eetteri-hypoteesin kanssa ristiriitainen, koska pyörivä liike vaikutti eri suuntiin kulkeviin valoihin eri tavalla.

        Fysiikan historia on mielenkiintoinen, mutten ymmärrä edelleenkään, miksi haluat palata sata vuotta vanhoihin keskusteluihin, jolloin näyttö suhteellisuusteorian puolesta oli vähäisempää ja monet fyysikot vielä pitivät eetteriä varteenotettavana kilpailevana hypoteesina.

        Löydät varmasti paljon eetteri-hypoteeseja etenkin, jos tutkit lähes sata vuotta vanhaa matskua. Toki löytyy myös muutama vastavirtaan kulkija, joka tekee vielä omaa pseudotiedettään. Maallikkona kannattaa lukea tiedelehtiä eikä kuvitella, että pystyt koulukurssien pohjalta ymmärtämään julkaisuja.

        Minä olen lukenut korkeakoulun kattavimmat perusfysiikat, siis samat, jotka teoreettiset fyysikot lukevat, mutta koska en ole fyysikko, niin julkaisujen yksityiskohdat jäävät usein hämäräksi. Pystyn kuitenkin arvioimaan, miltä teoreettiselta pohjalta tutkija ponnistaa ja tuo eetteri-hypoteesi ei oikein perustu muuhun kuin jonkun yksittäisen tutkijan ideoimaan malliin, jonka hän on rukannut sopimaan joihinkin yksittäisiin ilmiöihin ja esittää näitä sitten hypoteesinsa tueksi. Oleellista on johtaa mallin pohjalta kokonaisuutta kuvaava matemaattisen tarkka malli eikä vain heilutella käsiä ja sanoa, että ”tää on niin monimutkainen, ettei sitä saa malliksi väännettyä”. Tuollaiset verbaaliset ”mallit” ovat tyypillisiä pseudotieteelle, missä terve skeptisyys omaa ideaa kohtaa puuttuu ja takerrutaan niihin harvoihin havaintoihin, jotka tukevat omaa ajatusta.

        ”Tuo ei ole edes mikään havainto vaan teoreettinen postulaatti joka on virheellinen.”

        Valon nopeus on mitattu useilla eri menetelmillä. Toki voit esittää kilpailevan hypoteesin, miksi kaikki menetelmät osoittavat valon nopeuden samaksi ja tarjota sen sijaan vaihtoehtoisen selityksen.

        https://en.wikipedia.org/wiki/Speed_of_light#Measurement

        https://www.newscientist.com/article/mg23331184-900-atomic-clocks-make-best-measurement-yet-of-relativity-of-time/

        ”Ongelma mittauksissa on myös se että ne mittaukset tulkitaan suhteellisuusteorioiden viitekehyksessä joka on jo alunperin virheellinen.”

        Se, että kaikki varmennetut mittaukset sopivat yhteen suhteellisuusteorian kanssa, ei ole ongelma. Kilpailevia hypoteeseja voi esittää, mutten ole vielä nähnyt toista teoriaa, joka selittää niin monta eri asiaa ja vieläpä tarjoaa matemaattisen tarkat kaavat, joita vastaan mittauksia voidaan verrata.

        Jos luet edellä olevat linkit, niin huomaat, että suhteellisuusteoriaa yritetään vieläkin todistaa vääriksi. Jos havaittaisiin, että suhteellisuusteorian aika-malli poikkeaisi edes hieman havaitusta, niin tämä olisi mullistava-johtolanka yrittäessä kehittää entistäkin tarkempia teorioita.

        ”Itse en ole mikään suht.teorioiden asiantuntija mutta tiedän niistä taatusti paljon enemmän kuin sinä dyn. eetterin mallista. ”

        Lienet oikeassa. Silti ymmärtänet, että jos kyseinen teoria olisi varteenotettava vaihtoehto suhteellisuusteorialle, niin siitä kirjoitettaisiin tiedelehdissä. Dynaamista eetteriä esittävillä on siis vielä edessä iso työtaakka ennen kuin heidän hypoteesinsa olisi kelvollinen.

        Esimerkiksi fyysikko Mark Barton Glasgowin yliopistosta kommentoi 2018 eetteri teorioita seuraavasti: ”Tietääkseni kukaan ei ole luonut riittävästi työstettyä eetteri teoriaa siinä mielessä, että se käynnistäisi keskustelun erityisestä suhteellisuusteoriasta.”

        "Kannattaisi lukea Wikipedian artikkeli kokonaan. ."

        Sen takia sitä kutsutaan nykyään Sagnac-efektiksi koska ei haluta myöntää että se kumosi er. suhtkiksen perustan. Joka tapauksessa Aharonov–Bohm effekti viittaa jonkinlaisen kvanttipotentiaalin tai nollapiste-energian olemassaoloon ja se eetteri ei ole mikään homogeeninen kokonaisuus vaan ne potentiaalit voivat hyvin monimutkaisia (nollasummavektoreiden rotaatio) ja monimuotoisia geometrisesti ja topologisesti ja ovat ikäänkuin reaaliaikainen hologrammi tai templaatti kaikelle mahdolliselle aineen havaittavalle rakenteelle.

        "Fysiikan historia on mielenkiintoinen, .... "

        Tässä on vähän samantapainen ilmiö kuin Lamarckismissa joka epigenetiikan muodossa tavallaan syntyi uudestaan vaikka oli hylätty jo kauan sitten.

        "Löydät varmasti paljon eetteri-hypoteeseja etenkin.... "

        Oikea tiede ei vanhene ja periaatteessa tuo dynaamisen eetterin malli tunnettiin jo muinaisissa korkeakulttureissa mutta tietenkin vähän eri muodossa ja nimellä kuin nykyään.

        "Minä olen lukenut korkeakoulun kattavimmat perusfysiikat.."

        Minä taas olen perehtynyt melkein kaikkiin vaihtoehtoisiin malleihin ja sen lisäksi hyvin monipuolisesti okkultismiin, parapsykologiaan ja ylipäätänsä sekä nykyisiin että muinaisiin esoteerisiin oppeihin ja tietenkin myös siihen mitä tieteenä tuputetaan mediassa.

        Koulutus on aina ehdollistamista kepin ja porkkanan avulla. Sinun pitää omaksua ja sisäistää oppimasi niin että läpäiset kurssit ja saat arvosanan. Näin sinut on tavallaan "rokotettu" kaikkea sellaista vastaan mikä voisi vaarantaa sen konsensusteorian perusteet.

        "Pystyn kuitenkin arvioimaan,...."

        Et pysty hyvin pinnallisen yhden ainoan muutaman kymmenen sivun julkaisun pikaisen lukemisen perusteella arvioimaan yhtään mitään sellaisesta mallista joka poikkeaa täysin siitä mihin sinut on ehdollistettu. Skeptikoiden metodi on takertua yksityiskohtiin yksi kerrallaan erotettuna kokonaisuudesta ja arvioida sitten niitä sen valtavirtamallin perusteella eli eräänlainen uuvutustaktiikka.

        "Oleellista on johtaa mallin pohjalta kokonaisuutta kuvaava matemaattisen tarkka malli ..."

        Jos haet yhtä kaavaa dynaamiselle eetterille niin sellaista ei ole eikä voi olla sellaiselle mikä periaatteessa kattaa kaiken mahdollisen ja on kaiken mahdollisen perusta ja lisäksi lounteeltaan potentiaalinen eli implisiittinen eikä eksplisiittinen. Jos haluaa LaViolettelta teknisemmän esityksen niin sellainen löytyy hänen Subquantum kinetics teoksestaan.

        Matematiikka sellaisenaan ei liity mitenkään fysikaaliseen maailmaan ja se kuva mikä tieteestä annetaan suurelle yleisölle (matemaatikko liitutaulun ääressä piirtämässä liidulla kaavoja) on lähinnä naurettava karikatyyri. Matemaatikot ovat saaneet suurinta tuhoa aikaan fysiikassa ja muissakin tieteissä (varsinkin H. Minkowski).

        "Valon nopeus on mitattu useilla eri menetelmillä. ...."

        Kaikki on tehty Maapallon pinnalla tai läheisyydessä kuten jo edellä totesin.

        Kvanttifysiikan lomittumisen voi minusta korvata myös moninkertaisella valonnopeudella mutta se koskee muita vuorovaikutuksia eikä herz siniaaltoja. Jos valonnopeus vaihtelee paikan mukaan kuten Einstein itsekin pohdiskeli yleisen suht. teorian julkistamisen jälkeen niin tuoltakin kritiikiltä putoaa pohja pois.

        Tietenkin atomikellot toimivat eri tavalla eri paikoissa (kiihtyvyys,graviaatio) ja sen takia on aika järjetöntä linkittää kelloja suhteellisuusteorioihin vaikka se olikin muotia loogis-empiirisessä tai positivistisessa tieteessä 1900-luvun alussa. Se on eräänlaista instrumentalismia joka tuottaa pääasiassa merkitysetöntä tiedettä.

        "En ole vielä nähnyt toista teoriaa, joka selittää niin monta eri asiaa ja vieläpä tarjoaa matemaattisen tarkat kaavat, joita vastaan mittauksia voidaan verrata."

        Matematiikka voi olla mielenvaltaisen tarkkaa mutta luonto ei koskaan.

        "Jos luet edellä olevat linkit, niin huomaat, että suhteellisuusteoriaa yritetään vieläkin todistaa vääriksi."

        Niissä se lähtökohta on mielestäni usein hedelmätön.

        jatkuu...


      • Anonyymi
        Anonyymi kirjoitti:

        "Kannattaisi lukea Wikipedian artikkeli kokonaan. ."

        Sen takia sitä kutsutaan nykyään Sagnac-efektiksi koska ei haluta myöntää että se kumosi er. suhtkiksen perustan. Joka tapauksessa Aharonov–Bohm effekti viittaa jonkinlaisen kvanttipotentiaalin tai nollapiste-energian olemassaoloon ja se eetteri ei ole mikään homogeeninen kokonaisuus vaan ne potentiaalit voivat hyvin monimutkaisia (nollasummavektoreiden rotaatio) ja monimuotoisia geometrisesti ja topologisesti ja ovat ikäänkuin reaaliaikainen hologrammi tai templaatti kaikelle mahdolliselle aineen havaittavalle rakenteelle.

        "Fysiikan historia on mielenkiintoinen, .... "

        Tässä on vähän samantapainen ilmiö kuin Lamarckismissa joka epigenetiikan muodossa tavallaan syntyi uudestaan vaikka oli hylätty jo kauan sitten.

        "Löydät varmasti paljon eetteri-hypoteeseja etenkin.... "

        Oikea tiede ei vanhene ja periaatteessa tuo dynaamisen eetterin malli tunnettiin jo muinaisissa korkeakulttureissa mutta tietenkin vähän eri muodossa ja nimellä kuin nykyään.

        "Minä olen lukenut korkeakoulun kattavimmat perusfysiikat.."

        Minä taas olen perehtynyt melkein kaikkiin vaihtoehtoisiin malleihin ja sen lisäksi hyvin monipuolisesti okkultismiin, parapsykologiaan ja ylipäätänsä sekä nykyisiin että muinaisiin esoteerisiin oppeihin ja tietenkin myös siihen mitä tieteenä tuputetaan mediassa.

        Koulutus on aina ehdollistamista kepin ja porkkanan avulla. Sinun pitää omaksua ja sisäistää oppimasi niin että läpäiset kurssit ja saat arvosanan. Näin sinut on tavallaan "rokotettu" kaikkea sellaista vastaan mikä voisi vaarantaa sen konsensusteorian perusteet.

        "Pystyn kuitenkin arvioimaan,...."

        Et pysty hyvin pinnallisen yhden ainoan muutaman kymmenen sivun julkaisun pikaisen lukemisen perusteella arvioimaan yhtään mitään sellaisesta mallista joka poikkeaa täysin siitä mihin sinut on ehdollistettu. Skeptikoiden metodi on takertua yksityiskohtiin yksi kerrallaan erotettuna kokonaisuudesta ja arvioida sitten niitä sen valtavirtamallin perusteella eli eräänlainen uuvutustaktiikka.

        "Oleellista on johtaa mallin pohjalta kokonaisuutta kuvaava matemaattisen tarkka malli ..."

        Jos haet yhtä kaavaa dynaamiselle eetterille niin sellaista ei ole eikä voi olla sellaiselle mikä periaatteessa kattaa kaiken mahdollisen ja on kaiken mahdollisen perusta ja lisäksi lounteeltaan potentiaalinen eli implisiittinen eikä eksplisiittinen. Jos haluaa LaViolettelta teknisemmän esityksen niin sellainen löytyy hänen Subquantum kinetics teoksestaan.

        Matematiikka sellaisenaan ei liity mitenkään fysikaaliseen maailmaan ja se kuva mikä tieteestä annetaan suurelle yleisölle (matemaatikko liitutaulun ääressä piirtämässä liidulla kaavoja) on lähinnä naurettava karikatyyri. Matemaatikot ovat saaneet suurinta tuhoa aikaan fysiikassa ja muissakin tieteissä (varsinkin H. Minkowski).

        "Valon nopeus on mitattu useilla eri menetelmillä. ...."

        Kaikki on tehty Maapallon pinnalla tai läheisyydessä kuten jo edellä totesin.

        Kvanttifysiikan lomittumisen voi minusta korvata myös moninkertaisella valonnopeudella mutta se koskee muita vuorovaikutuksia eikä herz siniaaltoja. Jos valonnopeus vaihtelee paikan mukaan kuten Einstein itsekin pohdiskeli yleisen suht. teorian julkistamisen jälkeen niin tuoltakin kritiikiltä putoaa pohja pois.

        Tietenkin atomikellot toimivat eri tavalla eri paikoissa (kiihtyvyys,graviaatio) ja sen takia on aika järjetöntä linkittää kelloja suhteellisuusteorioihin vaikka se olikin muotia loogis-empiirisessä tai positivistisessa tieteessä 1900-luvun alussa. Se on eräänlaista instrumentalismia joka tuottaa pääasiassa merkitysetöntä tiedettä.

        "En ole vielä nähnyt toista teoriaa, joka selittää niin monta eri asiaa ja vieläpä tarjoaa matemaattisen tarkat kaavat, joita vastaan mittauksia voidaan verrata."

        Matematiikka voi olla mielenvaltaisen tarkkaa mutta luonto ei koskaan.

        "Jos luet edellä olevat linkit, niin huomaat, että suhteellisuusteoriaa yritetään vieläkin todistaa vääriksi."

        Niissä se lähtökohta on mielestäni usein hedelmätön.

        jatkuu...

        "Jos havaittaisiin, että suhteellisuusteorian aika-malli poikkeaisi edes hieman havaitusta, niin tämä olisi mullistava-johtolanka yrittäessä kehittää entistäkin tarkempia teorioita."

        Aika ei ole delta-t vaan moniulotteinen (3t) ja se pohjimmiltaan seuraa eetterin ja massakeskittymien rakenteesta ja sykleistä.

        ”Itse en ole mikään suht.teorioiden asiantuntija mutta tiedän niistä taatusti paljon enemmän kuin sinä dyn. eetterin mallista. ”

        Heh. Voi olla koska tutkimukset ovat minulla vielä pahasti kesken ja tässä ketjussa en yrittänytkään käsitellä suhtiksia pääasiana.

        "Dynaamista eetteriä esittävillä on siis vielä edessä iso työtaakka ennen kuin heidän hypoteesinsa olisi kelvollinen."

        Dynaamisen eetterin mallia on sovellettu jo kauan käytännössä eikä siitä koskaan voi tulla valtavirtamallia vaikka esim. ns. nollapiste-energia voi tulla julkiseen tuotantoon lähiaikoina esim. kylmäfuusion muodossa varsinkin kun on rakennettu jo kattava kyttäyssysteemi valvomaan ettei sitä käytetä väärin ja/tai joudu vääriin käsiin.

        "Esimerkiksi fyysikko Mark Barton ....”"

        Einsteinia ja suhtiksiahan pönkitetään jatkuvasti julkisuudessa varmuuden vuoksi ettei vain kukaan vahingossakaan erehtyisi tutkimaan vääränlaista fysiikkaa:D

        Taas esitit täydellistä nollakritiikkiä jossa et taaskaan halunnut ottaa huomioon sinulle annettuja kommentteja ja vinkkejä jatkotutkimukseen. Skeptikot ovat varsinaisia ajan haaskaajia ja taidan lopettaa tähän ellet keksi jotain muuta kuin tyhjäkäyntiä. Aivan kuin seinille puhuisi ja jatkuvasti ohi maalin tulee vastaukset.

        Jatkan keskustelua vasta sitten kun huomaan että olet edes vähän vaivautunut perehtymään aiheeseen ja siihen odotan kuluvan ainakin pari päivää. Jätän nyt lukijat pohtimaan ihan itse vähäksi aikaa ja poistun lukemaan ja tutkimaan koska tällainen on verbaalipingis on lähinnä ajanhaaskausta ja pois mielenkiintoisemmista asioista.

        Belisario


      • Anonyymi
        Anonyymi kirjoitti:

        "Jos havaittaisiin, että suhteellisuusteorian aika-malli poikkeaisi edes hieman havaitusta, niin tämä olisi mullistava-johtolanka yrittäessä kehittää entistäkin tarkempia teorioita."

        Aika ei ole delta-t vaan moniulotteinen (3t) ja se pohjimmiltaan seuraa eetterin ja massakeskittymien rakenteesta ja sykleistä.

        ”Itse en ole mikään suht.teorioiden asiantuntija mutta tiedän niistä taatusti paljon enemmän kuin sinä dyn. eetterin mallista. ”

        Heh. Voi olla koska tutkimukset ovat minulla vielä pahasti kesken ja tässä ketjussa en yrittänytkään käsitellä suhtiksia pääasiana.

        "Dynaamista eetteriä esittävillä on siis vielä edessä iso työtaakka ennen kuin heidän hypoteesinsa olisi kelvollinen."

        Dynaamisen eetterin mallia on sovellettu jo kauan käytännössä eikä siitä koskaan voi tulla valtavirtamallia vaikka esim. ns. nollapiste-energia voi tulla julkiseen tuotantoon lähiaikoina esim. kylmäfuusion muodossa varsinkin kun on rakennettu jo kattava kyttäyssysteemi valvomaan ettei sitä käytetä väärin ja/tai joudu vääriin käsiin.

        "Esimerkiksi fyysikko Mark Barton ....”"

        Einsteinia ja suhtiksiahan pönkitetään jatkuvasti julkisuudessa varmuuden vuoksi ettei vain kukaan vahingossakaan erehtyisi tutkimaan vääränlaista fysiikkaa:D

        Taas esitit täydellistä nollakritiikkiä jossa et taaskaan halunnut ottaa huomioon sinulle annettuja kommentteja ja vinkkejä jatkotutkimukseen. Skeptikot ovat varsinaisia ajan haaskaajia ja taidan lopettaa tähän ellet keksi jotain muuta kuin tyhjäkäyntiä. Aivan kuin seinille puhuisi ja jatkuvasti ohi maalin tulee vastaukset.

        Jatkan keskustelua vasta sitten kun huomaan että olet edes vähän vaivautunut perehtymään aiheeseen ja siihen odotan kuluvan ainakin pari päivää. Jätän nyt lukijat pohtimaan ihan itse vähäksi aikaa ja poistun lukemaan ja tutkimaan koska tällainen on verbaalipingis on lähinnä ajanhaaskausta ja pois mielenkiintoisemmista asioista.

        Belisario

        Lisäys:

        Ei ole mikään ihme että erityinen suhtis läpäisee kaikki empiiriset testit koska se on tautologia tai truismi varsinkin sen jälkeen kuin metrin mitta määriteltiin uudestaan valonnopeuden mukaan eli sen arvo ei voi muuttua koska metrin mitta muuttuu vastaavasti samalla tavalla. Lisäksi suhtiksessa on pituuden muuttuminen mukana lähestyttäessä C:tä.


        https://en.wikipedia.org/wiki/Length_contraction

        https://fi.wikipedia.org/wiki/Metri

        Usein ihmetyttää kuinka paljon sekoilua menee läpi tiedefaneille. Kai se on se vahva usko kuten kaikilla muillakin fundamentalisteilla...:D

        Belisario


      • Anonyymi kirjoitti:

        Lisäys:

        Ei ole mikään ihme että erityinen suhtis läpäisee kaikki empiiriset testit koska se on tautologia tai truismi varsinkin sen jälkeen kuin metrin mitta määriteltiin uudestaan valonnopeuden mukaan eli sen arvo ei voi muuttua koska metrin mitta muuttuu vastaavasti samalla tavalla. Lisäksi suhtiksessa on pituuden muuttuminen mukana lähestyttäessä C:tä.


        https://en.wikipedia.org/wiki/Length_contraction

        https://fi.wikipedia.org/wiki/Metri

        Usein ihmetyttää kuinka paljon sekoilua menee läpi tiedefaneille. Kai se on se vahva usko kuten kaikilla muillakin fundamentalisteilla...:D

        Belisario

        Oleellista on, että valonnopeus tyhjiössä on vakio eikä se riipu havaitsijan liikkeestä. Jos näin, niin sitä voi ihan hyvin käyttää pituuden määrittämiseen, kun sekuntti on määritelty cecium-atomin värähtelyn avulla.

        Tämä määrittely ei mitenkään estä jotakin osoittamaan, että valonsäde voi tyhjiössä kulkea useilla eri nopeuksilla.

        ”Usein ihmetyttää kuinka paljon sekoilua menee läpi tiedefaneille. Usein ihmetyttää kuinka paljon sekoilua menee läpi tiedefaneille.”

        Tiede kehittyy ja tiede kumotaan vain paremmalla tieteellä. Tieteessä olennaista on, että useat alan asiantuntijat tarkistavat ja haastavat toistensa hypoteesit. Yksittäisten tiedemiesten hypoteeseja ei kannata lähteä ottamaan kovin vakavasti ennen kuin ne ovat isomman joukon tarkastamia ja hypoteesin hyvyyttä on tutkittu.


      • Anonyymi kirjoitti:

        "Kannattaisi lukea Wikipedian artikkeli kokonaan. ."

        Sen takia sitä kutsutaan nykyään Sagnac-efektiksi koska ei haluta myöntää että se kumosi er. suhtkiksen perustan. Joka tapauksessa Aharonov–Bohm effekti viittaa jonkinlaisen kvanttipotentiaalin tai nollapiste-energian olemassaoloon ja se eetteri ei ole mikään homogeeninen kokonaisuus vaan ne potentiaalit voivat hyvin monimutkaisia (nollasummavektoreiden rotaatio) ja monimuotoisia geometrisesti ja topologisesti ja ovat ikäänkuin reaaliaikainen hologrammi tai templaatti kaikelle mahdolliselle aineen havaittavalle rakenteelle.

        "Fysiikan historia on mielenkiintoinen, .... "

        Tässä on vähän samantapainen ilmiö kuin Lamarckismissa joka epigenetiikan muodossa tavallaan syntyi uudestaan vaikka oli hylätty jo kauan sitten.

        "Löydät varmasti paljon eetteri-hypoteeseja etenkin.... "

        Oikea tiede ei vanhene ja periaatteessa tuo dynaamisen eetterin malli tunnettiin jo muinaisissa korkeakulttureissa mutta tietenkin vähän eri muodossa ja nimellä kuin nykyään.

        "Minä olen lukenut korkeakoulun kattavimmat perusfysiikat.."

        Minä taas olen perehtynyt melkein kaikkiin vaihtoehtoisiin malleihin ja sen lisäksi hyvin monipuolisesti okkultismiin, parapsykologiaan ja ylipäätänsä sekä nykyisiin että muinaisiin esoteerisiin oppeihin ja tietenkin myös siihen mitä tieteenä tuputetaan mediassa.

        Koulutus on aina ehdollistamista kepin ja porkkanan avulla. Sinun pitää omaksua ja sisäistää oppimasi niin että läpäiset kurssit ja saat arvosanan. Näin sinut on tavallaan "rokotettu" kaikkea sellaista vastaan mikä voisi vaarantaa sen konsensusteorian perusteet.

        "Pystyn kuitenkin arvioimaan,...."

        Et pysty hyvin pinnallisen yhden ainoan muutaman kymmenen sivun julkaisun pikaisen lukemisen perusteella arvioimaan yhtään mitään sellaisesta mallista joka poikkeaa täysin siitä mihin sinut on ehdollistettu. Skeptikoiden metodi on takertua yksityiskohtiin yksi kerrallaan erotettuna kokonaisuudesta ja arvioida sitten niitä sen valtavirtamallin perusteella eli eräänlainen uuvutustaktiikka.

        "Oleellista on johtaa mallin pohjalta kokonaisuutta kuvaava matemaattisen tarkka malli ..."

        Jos haet yhtä kaavaa dynaamiselle eetterille niin sellaista ei ole eikä voi olla sellaiselle mikä periaatteessa kattaa kaiken mahdollisen ja on kaiken mahdollisen perusta ja lisäksi lounteeltaan potentiaalinen eli implisiittinen eikä eksplisiittinen. Jos haluaa LaViolettelta teknisemmän esityksen niin sellainen löytyy hänen Subquantum kinetics teoksestaan.

        Matematiikka sellaisenaan ei liity mitenkään fysikaaliseen maailmaan ja se kuva mikä tieteestä annetaan suurelle yleisölle (matemaatikko liitutaulun ääressä piirtämässä liidulla kaavoja) on lähinnä naurettava karikatyyri. Matemaatikot ovat saaneet suurinta tuhoa aikaan fysiikassa ja muissakin tieteissä (varsinkin H. Minkowski).

        "Valon nopeus on mitattu useilla eri menetelmillä. ...."

        Kaikki on tehty Maapallon pinnalla tai läheisyydessä kuten jo edellä totesin.

        Kvanttifysiikan lomittumisen voi minusta korvata myös moninkertaisella valonnopeudella mutta se koskee muita vuorovaikutuksia eikä herz siniaaltoja. Jos valonnopeus vaihtelee paikan mukaan kuten Einstein itsekin pohdiskeli yleisen suht. teorian julkistamisen jälkeen niin tuoltakin kritiikiltä putoaa pohja pois.

        Tietenkin atomikellot toimivat eri tavalla eri paikoissa (kiihtyvyys,graviaatio) ja sen takia on aika järjetöntä linkittää kelloja suhteellisuusteorioihin vaikka se olikin muotia loogis-empiirisessä tai positivistisessa tieteessä 1900-luvun alussa. Se on eräänlaista instrumentalismia joka tuottaa pääasiassa merkitysetöntä tiedettä.

        "En ole vielä nähnyt toista teoriaa, joka selittää niin monta eri asiaa ja vieläpä tarjoaa matemaattisen tarkat kaavat, joita vastaan mittauksia voidaan verrata."

        Matematiikka voi olla mielenvaltaisen tarkkaa mutta luonto ei koskaan.

        "Jos luet edellä olevat linkit, niin huomaat, että suhteellisuusteoriaa yritetään vieläkin todistaa vääriksi."

        Niissä se lähtökohta on mielestäni usein hedelmätön.

        jatkuu...

        ”Sen takia sitä kutsutaan nykyään Sagnac-efektiksi koska ei haluta myöntää että se kumosi er. suhtkiksen perustan.”

        Sagnac luuli niin, mutta hänet todistettiin erehtyneen. Löydät Sagnac:in kokeen suhteellisuusteoriaan perustuvan laskelman seuraavalta sivulta, mistä voit itse nähdä tämän. Jos olet eri mieltä laskelmasta, niin ole hyvä ja kerro missä tapahtuu laskuvirhe.

        https://www.sciencedirect.com/science/article/pii/S1631070517300907

        ”Sinun pitää omaksua ja sisäistää oppimasi niin että läpäiset kurssit ja saat arvosanan. Näin sinut on tavallaan "rokotettu" kaikkea sellaista vastaan mikä voisi vaarantaa sen konsensusteorian perusteet.”

        Tuo kuulostaa peruskoululta tai lukiolta. Korkeakoulussa asiat käydään läpi perusteista lähtien. Teoriaa ei opetella apinamaisesti vaan opetus lähtee siitä, että käydään läpi, mihin kyseinen teoria perustuu mukaan lukien sen historiaa. Toki kiistanalaisia vaihtoehtoisteorioita ei ole aikaa käydä läpi, mutta jos ymmärrät yhden teorian perustan, niin sinulla on lähtökohta arvioida myös toisen teorian perusteita.

        ”Jos haet yhtä kaavaa dynaamiselle eetterille niin sellaista ei ole eikä voi olla sellaiselle mikä periaatteessa kattaa kaiken mahdollisen”

        Et ilmeisesti ole tietoinen Schrödingerin kvanttimekaniikan yhtälöistä? Kyseinen yhtälö sisältää periaatteessa kaiken, mitä tarvitset kvanttien, atomien, molekyylien ja jopa makrotason systeemien mallintamiselle. Yhtälö on modernissa muodosta kaunis ja yksinkertaisen näköinen differentiaaliyhtälö, mutta sen matemaattinen ratkaiseminen on todella haastavaa ja monimutkaisten systeemien mallintaminen on osoittautunut todella haastavaksi.

        ”Matematiikka sellaisenaan ei liity mitenkään fysikaaliseen maailmaan”

        Matematiikka on fyysikoiden työkalu. Matemaattinen malli on armoton, koska se voidaan testata mittauksin. Jos joku esimerkiksi sanoo, että kiven lento kaareutuu, koska ”avaruudesta tulee työntävää voimaa ja maapallo estää työntävän voiman toiselta puolelta” niin väite toki kestää filosofisen tarkastelun, mutta kaatuu siinä vaiheessa, kun pyydetään esittämään matemaattisen tarkka malli, joka mallintaa aurinkokunnan planeettojen radat yhtä hyvin tai paremmin kuin painovoimamalli (tai suhteellisuusteoria). Pointtini oli, että tälläisiä pseudotieteellisiä näennäisen järkeenkäypiä, mutta vääriä, selityksiä voi helposti keksiä, mutta ne kaatuvat heti, kun niitä aletaan testata numeerisesti.


      • Anonyymi
        Järkisyitä kirjoitti:

        ”Sen takia sitä kutsutaan nykyään Sagnac-efektiksi koska ei haluta myöntää että se kumosi er. suhtkiksen perustan.”

        Sagnac luuli niin, mutta hänet todistettiin erehtyneen. Löydät Sagnac:in kokeen suhteellisuusteoriaan perustuvan laskelman seuraavalta sivulta, mistä voit itse nähdä tämän. Jos olet eri mieltä laskelmasta, niin ole hyvä ja kerro missä tapahtuu laskuvirhe.

        https://www.sciencedirect.com/science/article/pii/S1631070517300907

        ”Sinun pitää omaksua ja sisäistää oppimasi niin että läpäiset kurssit ja saat arvosanan. Näin sinut on tavallaan "rokotettu" kaikkea sellaista vastaan mikä voisi vaarantaa sen konsensusteorian perusteet.”

        Tuo kuulostaa peruskoululta tai lukiolta. Korkeakoulussa asiat käydään läpi perusteista lähtien. Teoriaa ei opetella apinamaisesti vaan opetus lähtee siitä, että käydään läpi, mihin kyseinen teoria perustuu mukaan lukien sen historiaa. Toki kiistanalaisia vaihtoehtoisteorioita ei ole aikaa käydä läpi, mutta jos ymmärrät yhden teorian perustan, niin sinulla on lähtökohta arvioida myös toisen teorian perusteita.

        ”Jos haet yhtä kaavaa dynaamiselle eetterille niin sellaista ei ole eikä voi olla sellaiselle mikä periaatteessa kattaa kaiken mahdollisen”

        Et ilmeisesti ole tietoinen Schrödingerin kvanttimekaniikan yhtälöistä? Kyseinen yhtälö sisältää periaatteessa kaiken, mitä tarvitset kvanttien, atomien, molekyylien ja jopa makrotason systeemien mallintamiselle. Yhtälö on modernissa muodosta kaunis ja yksinkertaisen näköinen differentiaaliyhtälö, mutta sen matemaattinen ratkaiseminen on todella haastavaa ja monimutkaisten systeemien mallintaminen on osoittautunut todella haastavaksi.

        ”Matematiikka sellaisenaan ei liity mitenkään fysikaaliseen maailmaan”

        Matematiikka on fyysikoiden työkalu. Matemaattinen malli on armoton, koska se voidaan testata mittauksin. Jos joku esimerkiksi sanoo, että kiven lento kaareutuu, koska ”avaruudesta tulee työntävää voimaa ja maapallo estää työntävän voiman toiselta puolelta” niin väite toki kestää filosofisen tarkastelun, mutta kaatuu siinä vaiheessa, kun pyydetään esittämään matemaattisen tarkka malli, joka mallintaa aurinkokunnan planeettojen radat yhtä hyvin tai paremmin kuin painovoimamalli (tai suhteellisuusteoria). Pointtini oli, että tälläisiä pseudotieteellisiä näennäisen järkeenkäypiä, mutta vääriä, selityksiä voi helposti keksiä, mutta ne kaatuvat heti, kun niitä aletaan testata numeerisesti.

        "sen matemaattinen ratkaiseminen on todella haastavaa .."

        Niin koska matemaattisella fysiikalla on ongelmia jo yli 2 kappaleen systeemeiden mallintamisessa.

        "Matematiikka on fyysikoiden työkalu."

        Niin se on nimenomaan työkalu eikä mikään itsetarkoitus.

        "Matemaattinen malli on armoton, .."

        Matemaattiset kaavat soveltuvat suhteellisen yksinkertaisten ilmiöiden laskemiseen ja ennustamiseen mutta eivät enää olekaan kovin helposti sovellettavissa kompleksisiin järjestelmiin. Se on yksi minkä takia ns. dynaamisen eetterin ei ole ensisijaisesti matemaattista vaikka tietysti siinäkin voi soveltaa kaavoja mutta ne kaavat todennäköisesti poikkeavat vähintään yhtä paljon tunnetun fysiikan kaavoista kuin kvanttifysiikan kaavat yleisen suhteellisuusteorian kaavoista.

        Jos haluaa jonkinlaisen holistisen mallin (erotuksena) reduktionistisista malleista niin yleinen systeemiteoria lienee ainoa toimiva lähestymistapa ja juuri sitä systeemiteoriaa LaViolette pyrkiikin soveltamaan eli ts. LaVioletten subquantum kinetics ei ole käännettävissä yleisen suhteellisuusteorian kaavoille eikä myöskään toisinpäin koska mallit eivät ole keskenään yhteismitallisia eikä niillä ole yhteistä teoreettista rajapintaa.

        "Pointtini oli, että tälläisiä pseudotieteellisiä ..."

        Minun pointtini taas on se näiden dynaamisten "tyhjiö-" tai "eetterimallien" suhteen etteivät ne tarvitse mitään kovin tarkkaa numeerista testausta esim. yli 10 desimaalin tarkkuudella vaan eetterimallien mahdollistama paikallisen aika-avaruuden ilmiöiden manipulointi on niin selkeästi havaittavissa makromaailmassa ettei sen oikeellisuuden toteamiseen tarvita minkäänlaista mittaria vaikka voi sen tietysti mitatakin.

        Viittaan tässä esim. Tesla fysiikkaan ja Tom Beardenin tutkimuksiin ja teoksiin kuten Gravitobiology ja melkein 900 sivuiseen Energy from the Vacuum (jota en ole vielä aloittanut edes lukemaan kun löysin sen vasta eilen).

        Gravitaatio on eräänlainen aineellisten makrokappaleiden eetteritemplaatti tai "hologrammi" tai "eetteriruumis" (teosofia) jota voidaan manipuloida luomalla siitä kappaleesta ensin interferenssimalli esim. phase conjugate mirror-tekniikalla ja sitten manipuloidaan sitä templaattia ja siirretään takaisin siihen makrokappaleeseen epälineaarisesti ja etäisyydestä riippumatta ja ilman minkäänlaista energiahäviötä.

        Tällä tekniikalla voidaan ladata energiaa kappaleeseen tai imeä energiaa pois kappaleesta resonanssitemplaatin avulla, manipuloida geenejä tai kudoksia etänä sekä parantamis että sairastuttamistarkoituksessa, ohjata ja manipuloida sääilmiöitä ja esim. maanjäristyksiä sekä myös ihmisten ja eläinten tajuntaa sekä yksilöinä että joukkoina.

        Beardenin mukaan tällaista tekniikkaa on ollut olemassa jo n. 100 vuotta sitä käytetty esim. Saksan ja Neuvostoliiton toimesta kuuman & kylmän sodan aikana ja että nykyään on jopa 10 valtiota jolla on valmiudet käyttää tämäntyyppistä teknologiaa mm. sotilaallisiin tarkoituksiin joten vetypommit ovat auttamatta vanhentuneita. Tekniikka on myös helpommin sovellettavissa kuin esim. vetypommien rakentaminen.

        Tämä tietysti voi kuullostaa äärimmäisen villiltä ja hullulta idealta varsinkin kun se kuulee ensimmäistä kertaa mutta toisaalta mahdollistaa sellaisen piilosodankäynnin jonka voi laittaa luonnonilmiöiden ja ilmastonmuutoksen piikkiin.

        Positiivisempia puolia ilmiöissä on käytännössä rajaton, halpa ja saastuttamaton energia vaikka kyseessä ei olekaan ikiliikkuja vaan tyhjiön stressin ja nollasummavektoreiden muutaminen energiaksi joka voi muuttua jossain vaiheessa ongelmalliseksi jos energian kulutus vielä nykyisestä kasvaa eli sellaiset muuntolaitoksen kannattaa sijoittaa kaemmaksi Maapalloa. Toinen positiivinen puoli on tietysti sairauksien ja geneettisten vikojen parantaminen ja paljon muutakin kuten esim. matkailu toisiin galakseihin ja kyseessä on nimenomaan malli joka laajentaa fysiikan kaikkiin muihinkin tieteisiin ja tietoisuuden ilmiöihin jolloin saavutetaan se todellinen fysiikkatiede jota todellisempaa ei ole enää olemassa.

        Tekniset tarkat yksityiskohdat lienevät edelleen salaisia eivätkä koskaan tule olemaankaan julkisia ymmärrettävistä syistä johtuen.

        ...

        Paljon tätä enempää en pysty asiaa tällä hetkellä vielä selventämään enkä kommentoimaan eli tutkikaa itse niiden linkkien avulla mitä antanut ja hakukone on paras ystävänne jos oikeasti haluatte perehtyä tähän aiheeseen. Kyseessä on sekä tulevaisuuden että ufojen tiede jonka avulla ihmiskunta voi päästä Kardashevin asteikon korkeimpaan sivilisaatiotasoon eli manipuloida todellisuutta galaksien tasolla.

        Pahoittelen että olin kaapannut nimimerkki järkisyitä ketjun ja niin usein tuntuu käyvän kun keskustelu alkaa rönsyillä ja on sillä rönsyilylläkin omat hyvät puolensa joskus.

        Nyt lopetan kirjoittelun tähän ketjuun.

        Belisario


      • Anonyymi
        Anonyymi kirjoitti:

        "sen matemaattinen ratkaiseminen on todella haastavaa .."

        Niin koska matemaattisella fysiikalla on ongelmia jo yli 2 kappaleen systeemeiden mallintamisessa.

        "Matematiikka on fyysikoiden työkalu."

        Niin se on nimenomaan työkalu eikä mikään itsetarkoitus.

        "Matemaattinen malli on armoton, .."

        Matemaattiset kaavat soveltuvat suhteellisen yksinkertaisten ilmiöiden laskemiseen ja ennustamiseen mutta eivät enää olekaan kovin helposti sovellettavissa kompleksisiin järjestelmiin. Se on yksi minkä takia ns. dynaamisen eetterin ei ole ensisijaisesti matemaattista vaikka tietysti siinäkin voi soveltaa kaavoja mutta ne kaavat todennäköisesti poikkeavat vähintään yhtä paljon tunnetun fysiikan kaavoista kuin kvanttifysiikan kaavat yleisen suhteellisuusteorian kaavoista.

        Jos haluaa jonkinlaisen holistisen mallin (erotuksena) reduktionistisista malleista niin yleinen systeemiteoria lienee ainoa toimiva lähestymistapa ja juuri sitä systeemiteoriaa LaViolette pyrkiikin soveltamaan eli ts. LaVioletten subquantum kinetics ei ole käännettävissä yleisen suhteellisuusteorian kaavoille eikä myöskään toisinpäin koska mallit eivät ole keskenään yhteismitallisia eikä niillä ole yhteistä teoreettista rajapintaa.

        "Pointtini oli, että tälläisiä pseudotieteellisiä ..."

        Minun pointtini taas on se näiden dynaamisten "tyhjiö-" tai "eetterimallien" suhteen etteivät ne tarvitse mitään kovin tarkkaa numeerista testausta esim. yli 10 desimaalin tarkkuudella vaan eetterimallien mahdollistama paikallisen aika-avaruuden ilmiöiden manipulointi on niin selkeästi havaittavissa makromaailmassa ettei sen oikeellisuuden toteamiseen tarvita minkäänlaista mittaria vaikka voi sen tietysti mitatakin.

        Viittaan tässä esim. Tesla fysiikkaan ja Tom Beardenin tutkimuksiin ja teoksiin kuten Gravitobiology ja melkein 900 sivuiseen Energy from the Vacuum (jota en ole vielä aloittanut edes lukemaan kun löysin sen vasta eilen).

        Gravitaatio on eräänlainen aineellisten makrokappaleiden eetteritemplaatti tai "hologrammi" tai "eetteriruumis" (teosofia) jota voidaan manipuloida luomalla siitä kappaleesta ensin interferenssimalli esim. phase conjugate mirror-tekniikalla ja sitten manipuloidaan sitä templaattia ja siirretään takaisin siihen makrokappaleeseen epälineaarisesti ja etäisyydestä riippumatta ja ilman minkäänlaista energiahäviötä.

        Tällä tekniikalla voidaan ladata energiaa kappaleeseen tai imeä energiaa pois kappaleesta resonanssitemplaatin avulla, manipuloida geenejä tai kudoksia etänä sekä parantamis että sairastuttamistarkoituksessa, ohjata ja manipuloida sääilmiöitä ja esim. maanjäristyksiä sekä myös ihmisten ja eläinten tajuntaa sekä yksilöinä että joukkoina.

        Beardenin mukaan tällaista tekniikkaa on ollut olemassa jo n. 100 vuotta sitä käytetty esim. Saksan ja Neuvostoliiton toimesta kuuman & kylmän sodan aikana ja että nykyään on jopa 10 valtiota jolla on valmiudet käyttää tämäntyyppistä teknologiaa mm. sotilaallisiin tarkoituksiin joten vetypommit ovat auttamatta vanhentuneita. Tekniikka on myös helpommin sovellettavissa kuin esim. vetypommien rakentaminen.

        Tämä tietysti voi kuullostaa äärimmäisen villiltä ja hullulta idealta varsinkin kun se kuulee ensimmäistä kertaa mutta toisaalta mahdollistaa sellaisen piilosodankäynnin jonka voi laittaa luonnonilmiöiden ja ilmastonmuutoksen piikkiin.

        Positiivisempia puolia ilmiöissä on käytännössä rajaton, halpa ja saastuttamaton energia vaikka kyseessä ei olekaan ikiliikkuja vaan tyhjiön stressin ja nollasummavektoreiden muutaminen energiaksi joka voi muuttua jossain vaiheessa ongelmalliseksi jos energian kulutus vielä nykyisestä kasvaa eli sellaiset muuntolaitoksen kannattaa sijoittaa kaemmaksi Maapalloa. Toinen positiivinen puoli on tietysti sairauksien ja geneettisten vikojen parantaminen ja paljon muutakin kuten esim. matkailu toisiin galakseihin ja kyseessä on nimenomaan malli joka laajentaa fysiikan kaikkiin muihinkin tieteisiin ja tietoisuuden ilmiöihin jolloin saavutetaan se todellinen fysiikkatiede jota todellisempaa ei ole enää olemassa.

        Tekniset tarkat yksityiskohdat lienevät edelleen salaisia eivätkä koskaan tule olemaankaan julkisia ymmärrettävistä syistä johtuen.

        ...

        Paljon tätä enempää en pysty asiaa tällä hetkellä vielä selventämään enkä kommentoimaan eli tutkikaa itse niiden linkkien avulla mitä antanut ja hakukone on paras ystävänne jos oikeasti haluatte perehtyä tähän aiheeseen. Kyseessä on sekä tulevaisuuden että ufojen tiede jonka avulla ihmiskunta voi päästä Kardashevin asteikon korkeimpaan sivilisaatiotasoon eli manipuloida todellisuutta galaksien tasolla.

        Pahoittelen että olin kaapannut nimimerkki järkisyitä ketjun ja niin usein tuntuu käyvän kun keskustelu alkaa rönsyillä ja on sillä rönsyilylläkin omat hyvät puolensa joskus.

        Nyt lopetan kirjoittelun tähän ketjuun.

        Belisario

        Mitä on olemassaoleminen kysymyksestä päästiin oikein kunnon salaliittoteorioihin!

        Hieno saavutus taas some-maailmalta ja kertoo siitä, kuinka vaikea asia olemassaoleminen on ihmislle.


      • Anonyymi
        Anonyymi kirjoitti:

        Mitä on olemassaoleminen kysymyksestä päästiin oikein kunnon salaliittoteorioihin!

        Hieno saavutus taas some-maailmalta ja kertoo siitä, kuinka vaikea asia olemassaoleminen on ihmislle.

        Näin juuri. Ihmisillä on "angst" olemassaolostaan niin kuin Heidegger asian osuvasti ilmaisee.


    • Anonyymi

      OLE, äläkä kysele, miksi !!!!!

      • Anonyymi

        Päinvastoin. Aidolle olemiselle on olennaista se, miksi olisi olemassa. Se on eksistenssin tärkein sisältö.


      • Anonyymi
        Anonyymi kirjoitti:

        Päinvastoin. Aidolle olemiselle on olennaista se, miksi olisi olemassa. Se on eksistenssin tärkein sisältö.

        Ei se ole aitoa olemista, että olemisen luonnollisuus on kyseenalaistettu kysymyksillä.


      • Anonyymi
        Anonyymi kirjoitti:

        Ei se ole aitoa olemista, että olemisen luonnollisuus on kyseenalaistettu kysymyksillä.

        Olemisen "luonnollisuudesta" olisi mieluusti käynyt enemmänkin keskustelua. Melken neljä vuotta sitten kirjoitettu kommentti on kuitenkin jo vanhentunut.


    • Anonyymi

      Hypoteettinen heitto koskien virtuaalihiukkasten putkahtelua.

      Oikeastaan totaalista kenttien nollatilannetta on kai vaikeaa löytää, ellei puhuta mahdollisen alkuhetken jostain mahdollisesta nollatilanteesta, jos edes silloin. Kentillä on omat nollatilansa, johon ne pyrkivät tasaamalla lokaalia energiamuutosta globaaliksi, sekä välittämällä kentästä pois.

      Kentät ovat jatkuvan häiriön kohteina erilaisten energisten tapahtumien takia.

      Joten jos sanotaan, että "virtuaalihiukkaset putkahtavat tyhjästä", niin ihmettelen mistä ihmeen tyhjästä?

      Tai jos sanotaan, että "virtuaalihiukkanen ei voi syntyä, koska ei ole paikkaa josta se energia tulisi", sanoisin, että energiaa on aika hyvin saatavilla näistä jatkuvan häiriön kohteina olevista kentistä. Jos ei mistään muusta.

      Jos sitten vaikka energiahäviön näkökulmasta miettii. Välimatkan kasvaessa energiahäviöitä alkaa ilmenemään, koska tasaantuminen ja välittyminen. Tulee joko tapahtumia tai tilanteita, joissa kenttä luovuttaa energiaansa pois.

      Tilanteet: Eri kentät vaikuttavat ympäröivään omalla luonnollisella tavalla pyrkimyksessään kohti minimiä, eli jakaa vastaanottamaansa energiapottia pois. Siirto kentästä muualle tapahtuu hiukkasfysiikan termein välittäjähiukkasen avustuksella (kuten sähkömagneettinen säteily fotonin absormoitumisena materiaan, jonka energialisän seurauksena materia tyypillisesti lämpövärähtelee kiivaammin, eli lämpenee). Välittäjä hiukkanen kohtaa jotain, jonka läpi ei pääse vuorovaikuttamatta.

      Hypoteettinen heitto alkaa, tapahtumat:

      Alkutapahtumassa kentällä ei ole muuta mahdollisuutta kuin imeä kaikki energisessä tapahtumassa syntyvä energia, tai sitten ehkä hypoteettisesti romahtaa liiallisen energian seurauksena (en tiedä voisiko sellainen olla mahdollista), tai hypoteettisesti tuupata ylimäärä johonkin toiseen vuorovaikuttavaan kenttään, jos sellaisen ajatellaan sellaisen olevan mahdollista.

      Jälkimmäisen voisi olla mahdollinen, jos huomataan jonkin energisen tapahtuman yhteydessä syntyvien häiriöiden saapuvan aina tietyssä järjestyksessä. Tällöin ensimmäisenä havaittu häiriö edustaisi ehkä sitä kenttää, joka oli ottamassa energisen tapahtuman vastaan, ja jälkeenpäin tuleva häiriö olisi peräisin ylimääräenergian tuuppauksesta ensimmäisestä kentästä toiseen.

      Siirto kentästä toiseen liittyisi virtuaalihiukkasten syntyyn, tuolla alempana.

      Joten normaalimpi tilanne, miksei häiriö siinä kentässä edetessään myös jonkin kynnyksen ylityksestä tai jonkin toisen kentän kanssa kombinaationa, jollain säännöillä, tupsauttaisi ulos jotain yksittäistä virtuaalihiukkasta? Ei välttämättä tarvitsisi aina olla ylivuotoa (jolloin joko virtuaalihiukkasia syntyisi joko paljon pienienergisinä tai yksi tai muutama erittäin energinen), vaan joissain tilanteessa syntyisi yksi pienienerginen. Siinä tapahtumassa syntyyn tarvittu energia kuitenkin siirtyi etenevästä häiriöstä uuteen virtuaalihiukkaseen ja matkaansa jatkava häiriö vastaavalla osalla heikkeni.

      Virtuaalihiukkanen siirtäisi siis saamansa energiapotin johonkin toiseen kenttään ja häviäisi. Tämä mekanismi siis myös ylivuodossa mittavampsna.

      • Anonyymi

        Kvanttifluktuaatiot eivät johdu häiriöistä tai energiatapahtumista, vaan energiatapahtumat ovat normaaleja hiukkasfysiikan vuorovaikutuksia.
        Fluktuaatioita on yhtä paljon, vaikka avaruuden osa olisi täydellisesti eristetty ja todella kaukana kaikista häiriönlähteistä, esim. galaksienvälisessä avaruudessa.


      • Anonyymi
        Anonyymi kirjoitti:

        Kvanttifluktuaatiot eivät johdu häiriöistä tai energiatapahtumista, vaan energiatapahtumat ovat normaaleja hiukkasfysiikan vuorovaikutuksia.
        Fluktuaatioita on yhtä paljon, vaikka avaruuden osa olisi täydellisesti eristetty ja todella kaukana kaikista häiriönlähteistä, esim. galaksienvälisessä avaruudessa.

        Mistä tällaisen täydellisesti eristetyn paikan löytää? Miten kaukaisuus (suuri välimatka) liittyy asiaan? Galaksien välinen avaruus löytyy Linnunradan ja Andromedan välistä, tarkoitatko sitä vai jotain toista galaksien välistä avaruutta?


    • Anonyymi

      "Olemassa oleminen" on luonnollinen aistivarainen kokemus siihen asti, kunnes tieteilijät alkaa rakentamaan mitä monimutkaisempaa teoreettista, mahdollista, todellisuutta. Mitä monimutkaisemmaksi pohdiskelu menee, sitä kauemmaksi ihminen liikkuu poispäin luonnollisesta luonnonkokemista, olemisesta jne. Voikin siis sanoa, että elämme maailmassa, jossa teoriat, eli tieteelliset uskomukset, muodostavat todellisuuskäsityksen. Tämän edeltäjä, yliluonnolliset uskomukset, edusti varhaisempaa todellisuuskäsitystä. Voimme siis päätellä, että ihminen ei oikeastaan tiedä mitään, mutta luo suljetussa luomisympäristössään jatkuvasti mitä erilaisempaa informaatiota, josta etsitään vastauksia suuriin kysymyksiin.

    • Anonyymi

      Ei voi lopettaa mitään asiaa ellei sitä ole ensin aloitettu.
      Ei voi aloittaakaan mitään jos sitä ei olisi joku joskus aiemmin lopettanut.

      Loppu ilman alkua ja alku ilman loppua, se on "olemista".

    • Anonyymi

      Näkemys "siiventekijöiltä" (käännä englanniksi)

      Yksi tietoisuus sisältyy kaikkeen:

      -aikaan
      -tilaan
      -materiaan
      -energiaan
      -muotoon
      -tarkoitukseen

      Huom! Sama tietoisuus sisältyy myös:

      -ei-aikaan(eli non-time englanniksi)
      -ei-tilaan
      -ei-materiaan
      -ei-energiaan
      -ei-muotoon
      -ei-tarkoitukseen

      Eli kyseessä on eräänlainen panteismi, jossa tietoisuus on "jumala".

      Ehkä tämä toi erään näkemyksen aiheeseen "olemassa oleminen"

      --
      heisielu

    • Anonyymi

      Ihmisyyden osalta:
      Olemassaolo on sitä kun itsevarma ego puhuu tai epävarma ego kysyy.
      Ja psykoosin taso määrittelee realistisen totuuden ympäröivää koskien. ("Niin metsä vastaa, kuin hallusinaatiot haluaa")
      :-)
      Joskus vastasin samantyyppisen kysymykseen. Asioiden laatu merkitsee olemassaoloon. Reaalimaailman tai käsitteellisen eroavuus.
      --planeetta

    • Anonyymi

      Me

    • Anonyymi

      Ilman tilaa, tilavuutta, pyörimisliikettä, kiertoliikettä, lomittumista, laskostumista, sisäkkäisyyttä, dimensioita ja fraktaalinomaisuutta ei ole olemassaoloa minkään laista nykyisin. Olemassaolon määrä ja pituus riippuu vapaan tilavuuden määrästä. Vapausasteiden laskiessa nollaan olemassaolo lakkaa.
      Niin saattoi käydä singulariteetille, joka lakkasi olemassaolonsa vapausasteiden loppuessa kesken ja tapahtui singulariteetin metamorfoosi tai transformaatio kaikkeudeksi. Singulariteetti levitoi, jaunttasi, räjähti olemassaolevaksi kaikkeudeksi.
      Nykyisin olemme olemassaolonsa lakkanneen yhden ainoan olemassaolleen objektin NYKYINEN OLOTILA. Tuo olotila ei ole pelkästään tilavuutta tai avaruutta tarkoittava staattinen tilanne, vaan se on dynaaminen prosessi, joka ikuisesti transformoituu jopa äärettömyydeksi.
      Olemassaolo muuttui siis olotilaksi ja katosi tuossa muutoksessa itse pelkäksi muistoksi tai ideaksi olemassa olleesta todellisuudesta.

      Olemassaolo on muisto vapausasteiden kesken loppumista edeltäneestä ajasta. Nykyisin elämme dynaamisessa olotilassa. Mutta kun tuo olotila laajenee laajenemistaan kadottavat mittabosonit lopulta kosketuksensa eri hiukkasten välillä ja silloin romahtavat tässä olotilassa olevat sisäkkäiset dimensiot monin tavoin. Kumpikin on siis pahasta: Vapausasteiden loppuminen ja toisaalta liian suuret vapausasteet?.....

      Tää saattaa kuullostaa saivartelulta tai tyhmältä. Vihkoon kirjoitin asiasta viisi sivua ja yritin puolessa tunnissa tuosta viidestä sivusta kirjastossa jonkinlaisen ytimen saada aikaan tässä viestissä tälle foorumille. Sori..... Ei tämä taida oikein toimia....

    • Anonyymi

      Voimme luoda vaikka millaisia selityksiä, mutta emme siltikään kykene luomaan yhtään sellaista, joka selittäisi asian tyydyttävästi. Oletamme, että alkuaineet ja sähköenergia välittää lukuisan määrän signaaleja lukuisassa määriä aivosoluja, mutta emme tiedä, mikä prosessi muodostaa niiden tiedostamisen. Oli kyse ideasta tai esineestä, kaikella on sama rakenne ja toiminta aivoissa. Oletetusti. Ajatukset ovat aivojen oikeaan järjestykseen järjestämiä signaaleja, jotta aivot voivat lukea niitä ja tiedostaa ne, aivan kuten nollat ja ykköset oikeassa järjestyksessä kovalevyllä, jotta niistä muodostuu jotain ymmärrettävää ruudulle.

    • Näen/kuulen mitä olen , keho on vain kulkine .

    • Anonyymi

      Soutamalla ja huopaamalla huomaa aidon airon käytön sillä , että aidolla airolla vene käänntyy ,oikealle tai vasemmalle tai sitten pysyy paikallaan!

    • Anonyymi

      Olon-Tae ja hienhaju käy ko tuoksuvesi lihapulliin, sianlihapulliin.

    • Anonyymi

      Piersisin olen siis olemassa.

    • Anonyymi

      "Olemassa olo" on älykkyyden verbaalinen huipentuma, joka saa solmuun jäykimmänkin nojatuolifilosofin.

    Ketjusta on poistettu 2 sääntöjenvastaista viestiä.

    Luetuimmat keskustelut

    1. Heikki Silvennoinen petti vaimoaan vuosien ajan

      Viiden lapsen isä Heikki kehuu kirjassaan kuinka paljon on pettänyt vaimoaan vuosien varrella.
      Kotimaiset julkkisjuorut
      141
      2092
    2. Taasko se show alkaa

      Koo osottaa taas mieltään
      Ikävä
      27
      1928
    3. Miksi ihmeessä nainen seurustelit kanssani joskus

      Olin ruma silloin ja nykyisin vielä rumempi En voi kuin miettiä että miksi Olitko vain rikki edellisestä suhteesta ja ha
      Ikävä
      23
      1878
    4. Persut nimittivät kummeli-hahmon valtiosihteeriksi!

      Persujen riveistä löytyi taas uusi törkyturpa valtiosihteeriksi! Jutun perusteella järjenjuoksu on kuin sketsihahmolla.
      Perussuomalaiset
      85
      1690
    5. Onko ministeri Juuso epäkelpo ministerin tehtäviensä hoitamiseen?

      Eikö hänellä ole kompetenttia hoitaa sosiaali- ja terveysministetin toimialalle kuuluvia ministerin tehtäviä?
      Perussuomalaiset
      62
      1488
    6. Sakarjan kirjan 6. luku

      Jolla korva on, se kuulkoon. Sain profetian 22.4.2023. Sen sisältö oli seuraava: Suomeen tulee nälänhätä niin, että se
      Profetiat
      20
      1276
    7. Avaa sydämesi mulle

      ❤ ❤❤ Tahdon pelkkää hyvää sulle Sillä ilmeisesti puhumalla Avoimesti välillämme Kaikki taas selviää Kerro kaikki, tahdo
      Ikävä
      38
      1180
    8. Söpö lutunen oot

      Kaipaan aina vaan, vaikkakin sitten yksipuolisesti.
      Ikävä
      11
      1168
    9. Elia tulee vielä

      Johannes Kastaja oli Elia, mutta Jeesus sanoi, että Elia tulee vielä. Malakian kirjan profetia Eliasta toteutuu kokonaan
      Helluntailaisuus
      37
      1163
    10. Nellietä Emmaa ja Amandaa stressaa

      Ukkii minnuu Emmaa ja Amandaa stressaa ihan sikana joten voidaanko me koko kolmikko hypätä ukin kainaloon ja syleilyyn k
      Isovanhempien jutut
      10
      1147
    Aihe